NURS3309 | Health Assessment | Exam 3

Ace your homework & exams now with Quizwiz!

Matching: What are the skeletal movement names of the movements listed below? 1. Chin to each shoulder 2. Ear to shoulder 3. Chin to chest 4. Head back Options: Elevation, Rotation, Eversion, Inversion, Dorsiflexion, Lateral Flexion, Supination, Flexion, Extension, Pronation

1. Rotation 2. Lateral Flexion 3. Flexion 4. Extension

During an examination of a 3-year-old child, the nurse notices a *bruit over the left temporal area*. The nurse should: a. Continue the examination because a bruit is a normal finding for this age. b. Check for the bruit again in 1 hour. c. Notify the parents that a bruit has been detected in their child. d. Stop the examination, and notify the physician.

A Bruits are common in the skull in children under 4 or 5 years of age and in children with anemia. They are systolic or continuous and are heard over the temporal area.

The nurse is performing an assessment on a 7-year-old child who has *symptoms of chronic watery eyes, sneezing, and clear nasal drainage*. The nurse notices the presence of a transverse line across the bridge of the nose, dark blue shadows below the eyes, and a double crease on the lower eyelids. These findings are characteristic of: a. Allergies. b. Sinus infection. c. Nasal congestion. d. Upper respiratory infection.

A Chronic allergies often develop chronic facial characteristics. These include blue shadows below the eyes, a double or single crease on the lower eyelids, open-mouth breathing, and a transverse line on the nose.

During a well-baby checkup, a mother is concerned because her *2-month-old infant cannot hold her head up* when she is pulled to a sitting position. Which response by the nurse is appropriate? a. "Head control is usually achieved by 4 months of age." b. "You shouldn't be trying to pull your baby up like that until she is older." c. "Head control should be achieved by this time." d. "This inability indicates possible nerve damage to the neck muscles."

A Head control is achieved by 4 months when the baby can hold the head erect and steady when pulled to a vertical position. The other responses are not appropriate.

When examining a patient's cranial nerve (CN) function, the nurse remembers that the muscles in the neck that are innervated by CN XI are the: a. Sternomastoid and trapezius. b. Spinal accessory and omohyoid. c. Trapezius and sternomandibular. d. Sternomandibular and spinal accessory.

A The major neck muscles are the sternomastoid and the trapezius. They are innervated by CN XI, the spinal accessory.

A patient's laboratory data reveal an elevated thyroxine level. The nurse would proceed with an examination of the _____ gland. a. Thyroid b. Parotid c. Adrenal d. Parathyroid

A The thyroid gland is a highly vascular endocrine gland that secretes thyroxine (T4) and tri-iodothyronine (T3). The other glands do not secrete thyroxine.

The nurse suspects that a patient has hyperthyroidism and laboratory data indicate that the patient's *thyroxine and tri-iodothyronine hormone levels are elevated*. Which of these findings would the nurse most likely find on examination? a. Tachycardia b. Constipation c. Rapid dyspnea d. Atrophied nodular thyroid gland

A Thyroxine and tri-iodothyronine are thyroid hormones that stimulate the rate of cellular metabolism, resulting in tachycardia. With an enlarged thyroid as in hyperthyroidism, the nurse might expect to find diffuse enlargement (goiter) or a nodular lump, but not an atrophied gland. Dyspnea and constipation are not findings associated with hyperthyroidism.

During an examination, the nurse knows that the *best way to palpate the lymph nodes* in the neck is described by which statement? a. Using gentle pressure, palpate with both hands to compare the two sides. b. Using strong pressure, palpate with both hands to compare the two sides. c. Gently pinch each node between one's thumb and forefinger, and then move down The neck muscle. d. Using the index and middle fingers, gently palpate by applying pressure in a rotating pattern.

A Use gentle pressure because strong pressure could push the nodes into the neck muscles. It is usually most efficient to palpate with both hands, to compare the two sides symmetrically.

When examining children affected with Down syndrome (trisomy 21), the nurse looks for the possible presence of: a. Ear dysplasia. b. Long, thin neck. c. Protruding thin tongue. d. Narrow and raised nasal bridge.

A With the chromosomal aberration trisomy 21, also known as Down syndrome, head and face characteristics may include upslanting eyes with inner epicanthal folds, a flat nasal bridge, a small broad flat nose, a protruding thick tongue, ear dysplasia, a short broad neck with webbing, and small hands with a single palmar crease.

37. During an oral examination of a 4-year-old American-Indian child, the nurse notices that her uvula is partially split. Which of these statements is accurate? a. A bifid uvula may occur in some American-Indian groups. b. This condition is a cleft palate and is common in American Indians. c. A bifid uvula is torus palatinus, which frequently occurs in American Indians. d. This condition is due to an injury and should be reported to the authorities.

A Bifid uvula, a condition in which the uvula is split either completely or partially, occurs in some American-Indian groups. This finding is not a cleft palate, a torus palatinus (benign bony ridge running in the middle of the hard palate), or due to injury.

26. A 40-year-old patient who has just finished chemotherapy for breast cancer tells the nurse that she is concerned about her mouth. During the assessment the nurse finds areas of buccal mucosa that are raw and red with some bleeding, as well as other areas that have a white, cheesy coating. What do these findings indicate? a. Candidiasis b. Leukoplakia c. Koplik spots d. Aphthous ulcers

A Candidiasis is a white, cheesy, curdlike patch on the buccal mucosa and tongue. It scrapes off, leaving a raw, red surface that easily bleeds. It also occurs after the use of antibiotics or corticosteroids and in people who are immunosuppressed. Leukoplakia appears as chalky white, thick, raised patches with well-defined borders on the buccal mucosa. Koplik spots are small blue-white spots with irregular red halo scattered over mucosa opposite the molars and is an early sign of measles. Aphthous ulcers, or canker sores, first appear as a vesicle and then a small, round, "punched out" ulcer with a white base surrounded by a red halo and are quite painful and last for 1-2 weeks. The findings for this patient indicate candidiasis.

17. During an assessment of a 20-year-old patient with a 3-day history of nausea and vomiting, the nurse notices dry mucosa and deep vertical fissures in the tongue. What do these findings indicate? a. Dehydration b. A normal oral assessment c. Irritation from gastric juices d. Side effects from nausea medication

A Dehydration can cause dry mouth and deep vertical fissures in the tongue (due to reduced tongue volume). These finding are not normal and are not associated with irritation from gastric juices or from nausea caused by medications.

13. The nurse is using an otoscope to assess the nasal cavity. Which of these techniques is correct? a. Avoiding touching the nasal septum with the speculum b. Inserting the speculum at least 3 cm into the vestibule c. Gently displacing the nose to the side that is being examined d. Keeping the speculum tip medial to avoid touching the floor of the nares

A The correct technique for using an otoscope to examine the nasal cavity is to insert the apparatus into the nasal vestibule, avoiding pressure on the sensitive nasal septum. The tip of the nose should be lifted up before inserting the speculum.

36. During a checkup, a 22-year-old woman tells the nurse that she uses an over-the-counter nasal spray because of her allergies. She also states that it does not work as well as it used to when she first started using it. Which is the best response by the nurse? a. "You should never use over-the-counter nasal sprays because of the risk for addiction." b. "You should try switching to another brand of medication to prevent this problem." c. "Continuing to use this spray is important to keep your allergies under control." d. "Frequent use of these nasal medications irritates the lining of the nose and may cause rebound swelling."

A The dorsal surface of the tongue is normally roughened from papillae. A thin white coating may be present. The ventral surface may show veins. Smooth, glossy areas are abnormal and may indicate atrophic glossitis.

It's been two hours since you last saw your patient and when you enter the room and ask how he is, and as he responds you suspect he is having a stroke. His speech is __________(A) and he has a/an ___________(B). You quickly assess if he has any extremity _____________(C), and after verifying a third symptom, you notify the Stroke Team because you know its important to initiate therapy within __________(D) hours. Options: Fast, Tic, Weakness, 1, Slurred, Stutter, Incontinence, 3, Raspy, Uneven smile, Atrophy, 5

A - Slurred B - Uneven smile C - Weakness D - 3

During an emergency, your_____________ (A) system helps to ____________ (B) your ____________(C), and ____________(D). Options: Parasympathetic, Maintain, Heart rate, Relaxation, Sympathetic, Decrease, Digestion, Blood pressure, Central Nervous, Increase, Calmness, Hunger

A - Sympathetic B - Increase C/D - Heart Rate/Blood Pressure

indirect vs. direct inguinal hernias

A direct inguinal hernia shows a bulge from the posterior wall of the inguinal canal, whereas an indirect inguinal hernia passes through the inguinal canal or the groin: passing lateral to inferior epigastric vessels. Indirect is more likely to descend into the scrotum.

1. During an assessment, a patient mentions that "I just can't smell like I used to. I can barely smell the roses in my garden. Why is that?" For which possible causes of changes in the sense of smell will the nurse assess? (Select all that apply.) a. Aging b. Chronic allergies c. Cigarette smoking d. Chronic alcohol use e. Herpes simplex virus I f. Frequent episodes of strep throat

A, B, C The sense of smell diminishes with cigarette smoking, chronic allergies, and aging. Chronic alcohol use, a history of strep throat, and herpes simplex virus I are not associated with changes in the sense of smell. Chronic alcohol use, herpes simplex virus I, and frequent episodes of strep throat do not common causes of a diminished sense of smell. The sense of smell diminishes with cigarette smoking, chronic allergies, and aging. Chronic alcohol use, a history of strep throat, and herpes simplex virus I are not associated with changes in the sense of smell.

The examiner notices a fine tremor when the patient sticks out his or her tongue. What disorder is consistent with this finding? Hyperthyroidism Diabetic ketoacidosis Halitosis Alcoholism

A. A fine tremor of the tongue occurs with hyperthyroidism. A patient in diabetic ketoacidosis would have a sweet, fruity breath odor. Halitosis is a term used to describe any breath odor. A coarse tremor occurs with alcoholism.

Which of the following tests provides a precise quantitative measure of hearing? Audiometer test Tuning fork tests Whispered voice test Romberg test

A. An audiometer gives a precise quantitative measure of hearing by assessing the person's ability to hear sounds of varying frequency. The tuning fork tests (Weber and Rinne) are inaccurate and should not be used for general screening. The Romberg test assesses the ability of the vestibular apparatus in the inner ear to help maintain standing balance. The whispered voice test is nonquantitative, this test documents the presence of hearing loss but does not measure the degree of loss.

On examination of the mouth of an American Indian, the examiner notices the presence of a bifid uvula. How should this finding be interpreted? This is an expected variation associated with this individual. This may indicate the presence of oral cancer. This is rare and indicates other congenital anomalies may be present. This condition is frequently associated with cleft palate.

A. Bifid uvula is a condition in which the uvula is split either completely or partially. This condition occurs in 18% of individuals in some American Indian groups. Bifid uvula may indicate a submucous cleft palate. Bifid uvula is not associated with oral cancer. The incidence of bifid uvula is common in American Indians.

The position of the tympanic membrane in the neonate is more ________________, making it more difficult to visualize with the otoscope. horizontal perpendicular oblique vertical

A. The position of the eardrum is more horizontal in the neonate, making it more difficult to see completely and harder to differentiate from the canal wall. By 1 month of age, the eardrum is in the oblique position similar to an older child, and examination is easier.

The tympanic membrane of a child with acute otitis media would be bulging with a distinct red color. flat and slightly pulled in at the center. mobile and would flutter with the Valsalva maneuver. shiny and translucent, with a pearly gray color.

A. The tympanic membrane would be bulging and red with acute otitis media. A normal tympanic membrane is flat and slightly pulled in at the center. A normal tympanic membrane is mobile and flutters with the Valsalva maneuver. A normal tympanic membrane is shiny and translucent, with a pearly gray color.

The etiology of a pilonidal cyst is a congenital disorder. a chronically inflamed gastrointestinal tract. trauma or irritant diarrheal stools. a tear in the superficial mucosa.

A. A pilonidal cyst is a congenital disorder that is first diagnosed between the ages of 15 and 30 years. An anorectal fistula is caused by a chronically inflamed gastrointestinal tract. Fissures are tears that occur in the superficial mucosa and often result from trauma (e.g., passing a large, hard stool) or from irritant diarrheal stools.

Which of the following questions would be important to include when completing a subjective assessment of the breasts for a female patient to determine whether she would be at increased risk for breast cancer? "Have you taken oral contraceptives?" "Have you ever had a rash on or around your breasts? "Have you ever been taught breast self examination?" "Have you noticed any enlargement or fullness in the breasts?"

A. A recent study observed a 20% higher risk for breast cancer among women under 50 who were current or recent users of oral contraceptives than among women who had never used oral contraceptives. These are all important questions to include in the subjective assessment, but none of these specifically tell the health care provider if the female patient is at increased risk for breast cancer.

A retention cyst in the epididymis filled with milky fluid containing sperm is called a spermatocele. a prepuce. a varicocele. Peyronie disease.

A. A spermatocele is a retention cyst in the epididymis filled with milky fluid containing sperm. A varicocele is a dilated, tortuous varicose vein in the spermatic cord. Peyronie disease is a result of hard, nontender, subcutaneous plaques on the penis that cause a painful bending of the penis during an erection. Over the glans, the skinfolds in and back on itself forming a hood or flap called the foreskin or prepuce.

A normal common breast variation is a supernumerary nipple. enlarged axillary lymph nodes. fixation of the breast. a supernumerary breast.

A. A supernumerary nipple is a normal and common variation. An extra nipple on the thorax or abdomen is a congenital finding. It is usually 5 to 6 cm below the breast near the midline and has no glandular tissue. Enlarged axillary lymph nodes indicate a local infection of the breast, arm, or hand or breast cancer metastases. A supernumerary breast is rare, additional glandular tissue is present. Fixation of the breast indicates invasive cancer that fixes the breast to the underlying pectoral muscles.

The nurse is performing a neurological exam on a 63 year old male patient after a head trauma. The nurse notes the patient is unsteady and takes staggering steps, indicating the presence of which type of gait? A. Ataxic B. Parkinsonism (festinating) C. Hemiplegic D. Spastic

A. Ataxic

Clonus that may be seen when testing deep tendon reflexes is characterized by a(n) set of rapid, rhythmic contractions of the same muscle. contraction of the muscle that appears after the tendon is hit the second time. additional contraction of the muscle that is of greater intensity than the first contraction. parallel response in the opposite extremity.

A. Clonus is a set of rapid, rhythmic contractions of the same muscle.

Crepitation is an audible sound that is produced by roughened articular surfaces moving over each other. flexion and extension of an inflamed bursa. tendons or ligaments that slip over bones during motion. joints that are stretched when placed in hyperflexion or hyperextension.

A. Crepitation is an audible and palpable crunching or grating that accompanies movement. It occurs when the articular surfaces in the joints are roughened. Crepitation is not the cracking noise heard when tendons or ligaments slip over bones during motion. Hyperflexion or hyperextension is assessed with range of motion. Bursitis is an inflamed bursa. Pain may occur with motion of the joint involved.

Dysmenorrhea is pain associated with menstruation. pain with urination. pain with defecation. painful intercourse.

A. Dysmenorrhea is the abdominal cramping and pain associated with menstruation. Dyspareunia is the term to describe painful intercourse. Dyschezia is pain with bowel movements. Dysuria describes pain or burning with urination.

Testing the deep tendon reflexes gives the examiner information regarding the intactness of the reflex arc at specific levels in the spinal cord. medulla. corticospinal tract. upper motor and lower motor neuron synaptic junction.

A. Measurement of the deep tendon reflexes reveals the intactness of the reflex arc at specific spinal levels. The corticospinal tract is the higher motor system that permits very skilled and purposeful movements such as writing. The medulla contains all ascending and descending fiber tracts, it has vital autonomic centers for respiration, heart, and gastrointestinal function as well as nuclei for cranial nerves VIII through XII. The upper motor neurons are located within the central nervous system and influence or modify the lower motor neurons and include the corticospinal, corticobulbar, and extrapyramidal tracts. The lower motor neurons are located mostly in the peripheral nervous system and extend from the spinal cord to the muscles, examples include the cranial nerves and spinal nerves.

Cessation of menses is known as menopause. menarche. adnexa. salpingitis.

A. Menopause is the cessation of menses. Menarche is the age of the first period. Salpingitis is inflammation of the fallopian tube. The adnexa of uterus (or uterine appendages) refers to the structures most closely related structurally and functionally to the uterus, these structures include the ovaries, fallopian tubes, and ligaments.

Heberden and Bouchard nodes are hard and nontender and are associated with osteoarthritis. metacarpophalangeal bursitis. Dupuytren contracture. rheumatoid arthritis.

A. Osteoarthritis is characterized by hard, nontender nodules, 2 to 3 mm or more in size. These osteophytes (bony overgrowths) of the distal interphalangeal joints are called Heberden nodes, and osteophytes of the proximal interphalangeal joints are called Bouchard nodes. Swan neck, boutonnière deformity, and ulnar deviation are conditions associated with rheumatoid arthritis. Dupuytren contracture occurs with diabetes, epilepsy, and alcoholic liver disease. Chronic hyperplasia of the palmar fascia causes flexion contractures of the digits. Bursitis is an inflammation of the bursa.

In which of the following groups does osteoporosis primarily occur? Postmenopausal white women Asian men African Americans American Indians

A. Osteoporosis primarily occurs in postmenopausal white women. Other risk factors include: smaller height and weight, younger age at menopause, lack of physical activity, and lack of estrogen in women. African American adults have a decreased risk for fractures compared with white adults, and Hispanic women have a decreased risk for fractures compared with white women. The difference in fracture rates may be traced to childhood, where African American and Hispanic children have shown significantly higher bone strength than white children show. There is greater bone density at specific bone sites in African American and Hispanic children.

A 36 year old female patient reports finding a mass on her left breast during her monthly self-breast exam. The nurse knows that which of the following exam finding may be indicative of malignancy? (Select all that apply) A. Peak d'orange B. Tenderness C. Singular dense lump with irregular borders D. Nipple deviation E. Serosanguinous nipple discharge

A. Peak d'orange C. Singular dense lump with irregular borders E. Serosanguinous nipple discharge

A 31 year old male who works in an assemble line presents with a week hand grip and a tingling sensation in his fingers. The nurse knows that these symptoms may indicate median nerve compression and knows to perform which of the following assessments? A. Phalen test B. Bulge sign C. Ballottement D. Rebound tenderness

A. Phalen test

Pruritus is the presence of an itching or burning sensation. a longitudinal tear in the superficial mucosa at the anal margin. blood in the stool. excessive fat in the stool.

A. Pruritus is an itching or burning sensation. A fissure is a painful longitudinal tear in the superficial mucosa at the anal margin. Melena, hematochezia, and occult describe blood in the stool. Steatorrhea is excessive fat in the stool.

Rome III criteria for constipation includes which of the following? Straining to have a bowel movement Pain with bowel movement Soft, firm stools Four stools per week

A. Rome III criteria for constipation include: ≤3 stools per week, lumpy or hard stools, incomplete evacuation, and sensation of blockage. Pain is not included in the Rome III criteria for constipation.

What is the most appropriate recommendation to prevent cervical cancer in females? Administration of HPV vaccine around 11 and 12 years old Start birth control if sexually active There is nothing that can prevent cervical cancer. Use barrier protection method during sexual intercourse.

A. The American Cancer Society recommends HPV vaccine for all boys and girls at 11 and 12 years as HPV causes almost all cervical cancers in addition to other types of cancer. Barrier protection is important to use during intercourse to prevent spread of sexually transmitted infections, but is not the best method to prevent cervical cancer. Use of birth control when sexually active is important to decrease the risk for pregnancy. Studies show that the HPV vaccine at 11 and 12 years old has been linked to steep declines in the presence of HPV which causes almost all cervical cancers in addition to other types of cancer.

The ejaculatory duct is the passage formed by the joining of the vas deferens and the seminal vesicle. a narrow tunnel inferior to the inguinal ligament. a narrow tunnel superior to the inguinal ligament. a muscular duct continuous with the epididymis.

A. The ejaculatory duct is the passage formed by the junction of the duct of the seminal vesicles and the vas deferens through which semen enters the urethra. The muscular duct continuous with the epididymis is the vas deferens. The femoral canal is inferior to the inguinal ligament. The inguinal canal is superior to the inguinal ligament and is a narrow tunnel passing obliquely between layers of abdominal muscle.

The first sign of puberty in boys is enlargement of the testes. pubic hair growth extending up the abdomen. an increase in penis size. the appearance of pubic hair.

A. The first sign of puberty in boys is enlargement of the testes. Following the enlargement of the testes, pubic hair appears, then penis size increases. Pubic hair growth extending up the abdomen occurs after puberty.

The first sign of puberty in girls is breast and pubic hair development. rapid increase in height. the first menstrual cycle (menarche). axillary hair development.

A. The first signs of puberty are breast and pubic hair development, beginning between 8½ and 13 years of age. These signs usually occur together, but it is not abnormal if they do not develop together. This development takes about 3 years to complete. Menarche occurs during the latter half of the sequence of breast and pubic hair development, just after the peak of growth velocity. Coarse curly hairs develop in the pubic area first and then in the axillae.

The functions of the musculoskeletal system include protection and storage. propulsion and preservation. movement and elimination. storage and control.

A. The functions of the musculoskeletal system are as follows: provide support to stand erect, allow movement, encase and protect the inner vital organs, produce the red blood cells in the bone marrow, and act as a reservoir for storage of essential minerals, such as calcium and phosphorus in the bones.

Toilet training (for bowel movements) in children may begin when the nerves in the rectal area are fully myelinated. should start after 4 years of age. can be successful as early as 12 months of age. should begin after the gastrocolic reflex disappears.

A. Voluntary control of the external anal sphincter cannot occur until the nerves supplying the area have become fully myelinated, usually around 1½ to 2 years of age. Toilet training usually starts after age 2 years. The wave of peristalsis that occurs after eating is the gastrocolic reflex. This reflex is present at birth and does not disappear.

A 2-week-old infant can fixate on an object but cannot follow a light or bright toy. The nurse would: a. Consider this a normal finding. b. Assess the pupillary light reflex for possible blindness. c. Continue with the examination, and assess visual fields. d. Expect that a 2-week-old infant should be able to fixate and follow an object.

ANS: A By 2 to 4 weeks an infant can fixate on an object. By the age of 1 month, the infant should fixate and follow a bright light or toy.

A 68-year-old woman is in the eye clinic for a checkup. She tells the nurse that she has been having trouble reading the paper, sewing, and even seeing the faces of her grandchildren. On examination, the nurse notes that she has some loss of central vision but her peripheral vision is normal. These findings suggest that she may have: a. Macular degeneration. b. Vision that is normal for someone her age. c. The beginning stages of cataract formation. d. Increased intraocular pressure or glaucoma.

ANS: A Macular degeneration is the most common cause of blindness. It is characterized by the loss of central vision. Cataracts would show lens opacity. Chronic open-angle glaucoma, the most common type of glaucoma, involves a gradual loss of peripheral vision. These findings are not consistent with vision that is considered normal at any age.

A patients vision is recorded as 20/80 in each eye. The nurse interprets this finding to mean that the patient: a. Has poor vision. b. Has acute vision. c. Has normal vision. d. Is presbyopic.

ANS: A Normal visual acuity is 20/20 in each eye; the larger the denominator, the poorer the vision.

When performing the corneal light reflex assessment, the nurse notes that the light is reflected at 2 oclock in each eye. The nurse should: a. Consider this a normal finding. b. Refer the individual for further evaluation. c. Document this finding as an asymmetric light reflex. d. Perform the confrontation test to validate the findings.

ANS: A Reflection of the light on the corneas should be in exactly the same spot on each eye, or symmetric. If asymmetry is noted, then the nurse should administer the cover test.

The nurse is testing a patients visual accommodation, which refers to which action? a. Pupillary constriction when looking at a near object b. Pupillary dilation when looking at a far object c. Changes in peripheral vision in response to light d. Involuntary blinking in the presence of bright light

ANS: A The muscle fibers of the iris contract the pupil in bright light and accommodate for near vision, which also results in pupil constriction. The other responses are not correct.

The nurse is examining a patients retina with an ophthalmoscope. Which finding is considered normal? a. Optic disc that is a yellow-orange color b. Optic disc margins that are blurred around the edges c. Presence of pigmented crescents in the macular area d. Presence of the macula located on the nasal side of the retina

ANS: A The optic disc is located on the nasal side of the retina. Its color is a creamy yellow-orange to a pink, and the edges are distinct and sharply demarcated, not blurred. A pigmented crescent is black and is due to the accumulation of pigment in the choroid.

The nurse is performing the diagnostic positions test. Normal findings would be which of these results? a. Convergence of the eyes b. Parallel movement of both eyes c. Nystagmus in extreme superior gaze d. Slight amount of lid lag when moving the eyes from a superior to an inferior position

ANS: B A normal response for the diagnostic positions test is parallel tracking of the object with both eyes. Eye movement that is not parallel indicates a weakness of an extraocular muscle or dysfunction of the CN that innervates it.

Which of these assessment findings would the nurse expect to see when examining the eyes of a black patient? a. Increased night vision b. Dark retinal background c. Increased photosensitivity d. Narrowed palpebral fissures

ANS: B An ethnically based variability in the color of the iris and in retinal pigmentation exists, with darker irides having darker retinas behind them.

During a physical education class, a student is hit in the eye with the end of a baseball bat. When examined in the emergency department, the nurse notices the presence of blood in the anterior chamber of the eye. This finding indicates the presence of: a. Hypopyon. b. Hyphema. c. Corneal abrasion. d. Pterygium.

ANS: B Hyphema is the term for blood in the anterior chamber and is a serious result of blunt trauma (a fist or a baseball) or spontaneous hemorrhage and may indicate scleral rupture or major intraocular trauma. (See Table 14-7 for descriptions of the other terms.)

The nurse is performing an eye assessment on an 80-year-old patient. Which of these findings is considered abnormal? a. Decrease in tear production b. Unequal pupillary constriction in response to light c. Presence of arcus senilis observed around the cornea d. Loss of the outer hair on the eyebrows attributable to a decrease in hair follicles

ANS: B Pupils are small in the older adult, and the pupillary light reflex may be slowed, but pupillary constriction should be symmetric. The assessment findings in the other responses are considered normal in older persons.

The nurse is reviewing in age-related changes in the eye for a class. Which of these physiologic changes is responsible for presbyopia? a. Degeneration of the cornea b. Loss of lens elasticity c. Decreased adaptation to darkness d. Decreased distance vision abilities

ANS: B The lens loses elasticity and decreases its ability to change shape to accommodate for near vision. This condition is called presbyopia.

A patients vision is recorded as 20/30 when the Snellen eye chart is used. The nurse interprets these results to indicate that: a. At 30 feet the patient can read the entire chart. b. The patient can read at 20 feet what a person with normal vision can read at 30 feet. c. The patient can read the chart from 20 feet in the left eye and 30 feet in the right eye. d. The patient can read from 30 feet what a person with normal vision can read from 20 feet.

ANS: B The top number indicates the distance the person is standing from the chart; the denominator gives the distance at which a normal eye can see.

A patient comes to the emergency department after a boxing match, and his left eye is swollen almost shut. He has bruises on his face and neck. He says he is worried because he cant see well from his left eye. The physician suspects retinal damage. The nurse recognizes that signs of retinal detachment include: a. Loss of central vision. b. Shadow or diminished vision in one quadrant or one half of the visual field. c. Loss of peripheral vision. d. Sudden loss of pupillary constriction and accommodation.

ANS: B With retinal detachment, the person has shadows or diminished vision in one quadrant or one half of the visual field. The other responses are not signs of retinal detachment.

A mother asks when her newborn infants eyesight will be developed. The nurse should reply: a. Vision is not totally developed until 2 years of age. b. Infants develop the ability to focus on an object at approximately 8 months of age. c. By approximately 3 months of age, infants develop more coordinated eye movements and can fixate on an object. d. Most infants have uncoordinated eye movements for the first year of life.

ANS: C Eye movements may be poorly coordinated at birth, but by 3 to 4 months of age, the infant should establish binocularity and should be able to fixate simultaneously on a single image with both eyes.

The nurse notices the presence of periorbital edema when performing an eye assessment on a 70-year-old patient. The nurse should: a. Check for the presence of exophthalmos. b. Suspect that the patient has hyperthyroidism. c. Ask the patient if he or she has a history of heart failure. d. Assess for blepharitis, which is often associated with periorbital edema.

ANS: C Periorbital edema occurs with local infections, crying, and systemic conditions such as heart failure, renal failure, allergy, and hypothyroidism. Periorbital edema is not associated with blepharitis.

A 60-year-old man is at the clinic for an eye examination. The nurse suspects that he has ptosis of one eye. How should the nurse check for this? a. Perform the confrontation test. b. Assess the individuals near vision. c. Observe the distance between the palpebral fissures. d. Perform the corneal light test, and look for symmetry of the light reflex.

ANS: C Ptosis is a drooping of the upper eyelid that would be apparent by observing the distance between the upper and lower eyelids. The confrontation test measures peripheral vision. Measuring near vision or the corneal light test does not check for ptosis.

The nurse is assessing color vision of a male child. Which statement is correct? The nurse should: a. Check color vision annually until the age of 18 years. b. Ask the child to identify the color of his or her clothing. c. Test for color vision once between the ages of 4 and 8 years. d. Begin color vision screening at the childs 2-year checkup.

ANS: C Test boys only once for color vision between the ages of 4 and 8 years. Color vision is not tested in girls because it is rare in girls. Testing is performed with the Ishihara test, which is a series of polychromatic cards.

The nurse is preparing to assess the visual acuity of a 16-year-old patient. How should the nurse proceed? a. Perform the confrontation test. b. Ask the patient to read the print on a handheld Jaeger card. c. Use the Snellen chart positioned 20 feet away from the patient. d. Determine the patients ability to read newsprint at a distance of 12 to 14 inches.

ANS: C The Snellen alphabet chart is the most commonly used and most accurate measure of visual acuity. The confrontation test is a gross measure of peripheral vision. The Jaeger card or newspaper tests are used to test near vision.

During an assessment, the nurse notices that an older adult patient has tears rolling down his face from his left eye. Closer examination shows that the lower lid is loose and rolling outward. The patient complains of his eye feeling dry and itchy. Which action by the nurse is correct? a. Assessing the eye for a possible foreign body b. Documenting the finding as ptosis c. Assessing for other signs of ectropion d. Contacting the prescriber; these are signs of basal cell carcinoma

ANS: C The condition described is known as ectropion, and it occurs in older adults and is attributable to atrophy of the elastic and fibrous tissues. The lower lid does not approximate to the eyeball, and, as a result, the puncta cannot effectively siphon tears; excessive tearing results. Ptosis is a drooping of the upper eyelid. These signs do not suggest the presence of a foreign body in the eye or basal cell carcinoma.

When a light is directed across the iris of a patients eye from the temporal side, the nurse is assessing for: a. Drainage from dacryocystitis. b. Presence of conjunctivitis over the iris. c. Presence of shadows, which may indicate glaucoma. d. Scattered light reflex, which may be indicative of cataracts.

ANS: C The presence of shadows in the anterior chamber may be a sign of acute angle-closure glaucoma. The normal iris is flat and creates no shadows. This method is not correct for the assessment of dacryocystitis, conjunctivitis, or cataracts.

In using the ophthalmoscope to assess a patients eyes, the nurse notices a red glow in the patients pupils. On the basis of this finding, the nurse would: a. Suspect that an opacity is present in the lens or cornea. b. Check the light source of the ophthalmoscope to verify that it is functioning. c. Consider the red glow a normal reflection of the ophthalmoscope light off the inner retina. d. Continue with the ophthalmoscopic examination, and refer the patient for further evaluation.

ANS: C The red glow filling the persons pupil is the red reflex and is a normal finding caused by the reflection of the ophthalmoscope light off the inner retina. The other responses are not correct.

When assessing the pupillary light reflex, the nurse should use which technique? a. Shine a penlight from directly in front of the patient, and inspect for pupillary constriction. b. Ask the patient to follow the penlight in eight directions, and observe for bilateral pupil constriction. c. Shine a light across the pupil from the side, and observe for direct and consensual pupillary constriction. d. Ask the patient to focus on a distant object. Then ask the patient to follow the penlight to approximately 7 cm from the nose.

ANS: C To test the pupillary light reflex, the nurse should advance a light in from the side and note the direct and consensual pupillary constriction.

A patient comes into the emergency department after an accident at work. A machine blew dust into his eyes, and he was not wearing safety glasses. The nurse examines his corneas by shining a light from the side across the cornea. What findings would suggest that he has suffered a corneal abrasion? a. Smooth and clear corneas b. Opacity of the lens behind the cornea c. Bleeding from the areas across the cornea d. Shattered look to the light rays reflecting off the cornea

ANS: D A corneal abrasion causes irregular ridges in reflected light, which produce a shattered appearance to light rays. No opacities should be observed in the cornea. The other responses are not correct.

A 52-year-old patient describes the presence of occasional floaters or spots moving in front of his eyes. The nurse should: a. Examine the retina to determine the number of floaters. b. Presume the patient has glaucoma and refer him for further testing. c. Consider these to be abnormal findings, and refer him to an ophthalmologist. d. Know that floaters are usually insignificant and are caused by condensed vitreous fibers.

ANS: D Floaters are a common sensation with myopia or after middle age and are attributable to condensed vitreous fibers. Floaters or spots are not usually significant, but the acute onset of floaters may occur with retinal detachment.

A patient is unable to read even the largest letters on the Snellen chart. The nurse should take which action next? a. Refer the patient to an ophthalmologist or optometrist for further evaluation. b. Assess whether the patient can count the nurses fingers when they are placed in front of his or her eyes. c. Ask the patient to put on his or her reading glasses and attempt to read the Snellen chart again. d. Shorten the distance between the patient and the chart until the letters are seen, and record that distance.

ANS: D If the person is unable to see even the largest letters when standing 20 feet from the chart, then the nurse should shorten the distance to the chart until the letters are seen, and record that distance (e.g., 10/200). If visual acuity is even lower, then the nurse should assess whether the person can count fingers when they are spread in front of the eyes or can distinguish light perception from a penlight. If vision is poorer than 20/30, then a referral to an ophthalmologist or optometrist is necessary, but the nurse must first assess the visual acuity.

During an examination of the eye, the nurse would expect what normal finding when assessing the lacrimal apparatus? a. Presence of tears along the inner canthus b. Blocked nasolacrimal duct in a newborn infant c. Slight swelling over the upper lid and along the bony orbit if the individual has a cold d. Absence of drainage from the puncta when pressing against the inner orbital rim

ANS: D No swelling, redness, or drainage from the puncta should be observed when it is pressed. Regurgitation of fluid from the puncta, when pressed, indicates duct blockage. The lacrimal glands are not functional at birth.

During an assessment of the sclera of a black patient, the nurse would consider which of these an expected finding? a. Yellow fatty deposits over the cornea b. Pallor near the outer canthus of the lower lid c. Yellow color of the sclera that extends up to the iris d. Presence of small brown macules on the sclera

ANS: D Normally in dark-skinned people, small brown macules may be observed in the sclera.

An ophthalmic examination reveals papilledema. The nurse is aware that this finding indicates: a. Retinal detachment. b. Diabetic retinopathy. c. Acute-angle glaucoma. d. Increased intracranial pressure.

ANS: D Papilledema, or choked disk, is a serious sign of increased intracranial pressure, which is caused by a space-occupying mass such as a brain tumor or hematoma. This pressure causes venous stasis in the globe, showing redness, congestion, and elevation of the optic disc, blurred margins, hemorrhages, and absent venous pulsations. Papilledema is not associated with the conditions in the other responses.

The nurse is performing an eye-screening clinic at a daycare center. When examining a 2-year-old child, the nurse suspects that the child has a lazy eye and should: a. Examine the external structures of the eye. b. Assess visual acuity with the Snellen eye chart. c. Assess the childs visual fields with the confrontation test. d. Test for strabismus by performing the corneal light reflex test.

ANS: D Testing for strabismus is done by performing the corneal light reflex test and the cover test. The Snellen eye chart and confrontation test are not used to test for strabismus.

The nurse is assessing a patients eyes for the accommodation response and would expect to see which normal finding? a. Dilation of the pupils b. Consensual light reflex c. Conjugate movement of the eyes d. Convergence of the axes of the eyes

ANS: D The accommodation reaction includes pupillary constriction and convergence of the axes of the eyes. The other responses are not correct.

A patient has a normal pupillary light reflex. The nurse recognizes that this reflex indicates that: a. The eyes converge to focus on the light. b. Light is reflected at the same spot in both eyes. c. The eye focuses the image in the center of the pupil. d. Constriction of both pupils occurs in response to bright light.

ANS: D The pupillary light reflex is the normal constriction of the pupils when bright light shines on the retina. The other responses are not correct.

During an examination, the nurse knows that Paget's disease would be indicated by what of these assessment findings?

ANS: Headache, vertigo, tinnitus, and deafness Paget's disease occurs more often in males and is characterized by bowed long bones, sudden fractures, and enlarging skull bones that press on cranial nerves causing symptoms of headache, vertigo, tinnitus, and progressive deafness.

A patient presents with *excruciating headache pain on one side of his head, especially around his eye, forehead, and cheek that lasts about 1/2 to 2 hours, occurring once or twice each day*. The nurse should suspect: a. Hypertension. b. Cluster headaches. c. Tension headaches. d. Migraine headaches.

B Cluster headaches produce pain around the eye, temple, forehead, and cheek and are unilateral and always on the same side of the head. They are excruciating and occur once or twice per day and last 1/2 to 2 hours each.

During an assessment of an infant, the nurse notes that the *fontanels are depressed and sunken*. The nurse suspects which condition? a. Rickets b. Dehydration c. Mental retardation d. Increased intracranial pressure

B Depressed and sunken fontanels occur with dehydration or malnutrition. Mental retardation and rickets have no effect on fontanels. Increased intracranial pressure would cause tense or bulging, and possibly pulsating fontanels.

A mother brings her newborn in for an assessment and asks, "Is there something wrong with my baby? His head seems so big." The nurse recognizes that which statement is true regarding the relative proportions of the head and trunk of the newborn? a. At birth, the head is one fifth the total length. b. Head circumference should be greater than chest circumference at birth. c. The head size reaches 90% of its final size when the child is 3 years old. d. When the anterior fontanel closes at 2 months, the head will be more proportioned to the body.

B During the fetal period, head growth predominates. Head size is greater than chest circumference at birth, and the head size grows during childhood, reaching 90% of its final size when the child is age 6 years.

The nurse has just completed a lymph node assessment on a 60-year-old healthy female patient. The nurse knows that *most lymph nodes in healthy adults* are normally: a. Shotty. b. Nonpalpable. c. Large, firm, and fixed to the tissue. d. Rubbery, discrete, and mobile.

B Most lymph nodes are not palpable in adults. The palpability of lymph nodes decreases with age. Normal nodes feel movable, discrete, soft, and nontender.

A patient says that she has recently noticed a lump in the front of her neck below her "Adam's apple" that seems to be getting bigger. During the assessment, the finding that leads the nurse to suspect that this may not be a cancerous thyroid nodule is that the lump (nodule): a. Is tender. b. Is mobile and not hard. c. Disappears when the patient smiles. d. Is hard and fixed to the surrounding structures.

B Painless, rapidly growing nodules may be cancerous, especially the appearance of a single nodule in a young person. However, cancerous nodules tend to be hard and fixed to surrounding structures, not mobile.

A patient has come in for an examination and states, "I have this spot in front of my ear lobe here on my cheek that seems to be getting bigger and is tender. What do you think it is?" The nurse notes swelling below the angle of the jaw and suspects that it could be an inflammation of his: a. Thyroid gland. b. Parotid gland. c. Occipital lymph node. d. Submental lymph node.

B Swelling with the parotid gland occurs below the angle of the jaw and is most visible when the head is extended. Painful inflammation occurs with mumps, and swelling also occurs with abscesses or tumors. Swelling occurs anterior to the lower ear lobe.

A patient comes to the clinic complaining of neck and shoulder pain and is unable to turn her head. The nurse suspects damage to cranial nerve (CN) _____ and proceeds with the examination by _____. a. XI; palpating the anterior and posterior triangles b. XI; asking the patient to shrug her shoulders against resistance c. XII; percussing the sternomastoid and submandibular neck muscles d. XII; assessing for a positive Romberg sign

B The major neck muscles are the sternomastoid and the trapezius. They are innervated by CN XI, the spinal accessory. The innervated muscles assist with head rotation and head flexion, movement of the shoulders, and extension and turning of the head.

A male patient with a history of AIDS has come in for an examination and he states, "I think that I have the mumps." The nurse would begin by examining the: a. Thyroid gland. b. Parotid gland. c. Cervical lymph nodes. d. Mouth and skin for lesions.

B The parotid gland may become swollen with the onset of mumps, and parotid enlargement has been found with HIV.

The physician reports that a patient with a neck tumor has a *tracheal shift*. The nurse is aware that this means that the patient's trachea is: a. Pulled to the affected side. b. Pushed to the unaffected side. c. Pulled downward. d. Pulled downward in a rhythmic pattern.

B The trachea is pushed to the unaffected side with an aortic aneurysm, a tumor, unilateral thyroid lobe enlargement, and pneumothorax. The trachea is pulled to the affected side with large atelectasis, pleural adhesions, or fibrosis. Tracheal tug is a rhythmic downward pull that is synchronous with systole and occurs with aortic arch aneurysm.

31. When examining the mouth of an older patient, the nurse recognizes which finding is due to the aging process? a. Teeth appearing shorter b. Tongue that looks smoother in appearance c. Buccal mucosa that is beefy red in appearance d. Small, painless lump on the dorsum of the tongue

B In the aging adult, the tongue looks smoother because of papillary atrophy. The teeth are slightly yellowed and appear longer because of the recession of gingival margins.

30. A mother is concerned because her 18-month-old toddler has 12 teeth. She is wondering if this is normal for a child of this age. Which is the best response by the nurse? a. "How many teeth did you have at this age?" b. "This is a normal number of teeth for an 18 month old." c. "Normally, by age 2 1/2 years, 16 deciduous teeth are expected." d. "All 20 deciduous teeth are expected to erupt by age 4 years."

B The guidelines for the number of teeth for children younger than 2 years old are as follows: the child's age in months minus the number 6 should be equal to the expected number of deciduous teeth. Normally all 20 teeth are in by 2 1/2 years old. In this instance, the child is 18 months old, minus 6, equals 12 deciduous teeth expected.

15. The nurse is palpating the sinus areas. If the findings are normal, then the patient should report which sensation? a. No sensation b. Firm pressure c. Pain during palpation d. Pain sensation behind eyes

B The person should feel firm pressure but no pain. Sinus areas are tender to palpation in persons with chronic allergies or an acute infection (sinusitis). A normal finding when palpating the sinus areas is for the patient to feel firm pressure, not no sensation at all, pain during palpation, or pain behind the eyes. Sinus areas that are tender to palpation may indicate chronic allergies or an acute infection (sinusitis). Feeling firm pressure but no pain is a normal finding.

5. In assessing the tonsils of a 30-year-old, the nurse notices that they are involuted, granular in appearance, and appear to have deep crypts. What is the correct response to these findings? a. Refer the patient to a throat specialist. b. No response is needed; this appearance is normal for the tonsils. c. Continue with the assessment, looking for any other abnormal findings. d. Obtain a throat culture on the patient for possible streptococcal (strep) infection

B The tonsils are the same color as the surrounding mucous membrane, although they look more granular and their surface shows deep crypts. Tonsillar tissue enlarges during childhood until puberty and then involutes. There is no need to refer the patient to a throat specialist, obtain a throat culture, or look for other abnormal findings because the findings in this question are normal. Although the tonsils look more granular and their surface shows deep crypts, they are the same color as the surrounding mucous membrane and tonsillar tissue enlarges during childhood until puberty and then involutes.

9. While obtaining a health history, a patient tells the nurse that he has frequent nosebleeds and asks the best way to get them to stop. What would be the nurse's best response? a. "While sitting up, place a cold compress over your nose." b. "Sit up with your head tilted forward and pinch your nose." c. "Allow the bleeding to stop on its own, but don't blow your nose." d. "Lie on your back with your head tilted back and pinch your nose."

B With a nosebleed, the person should sit up with the head tilted forward and pinch the nose between the thumb and forefinger for 5 to 15 minutes.

19. A 10-year-old is at the clinic for "a sore throat that has lasted 6 days." Which of these findings would be consistent with an acute infection? a. Tonsils 3+/1-4+ with pale coloring b. Tonsils 3+/1-4+ with large white spots c. Tonsils 2+/1-4+ with small plugs of white debris d. Tonsils 1+/1-4+ and pink; the same color as the oral mucosa

B With an acute infection, tonsils are bright red and swollen and may have exudate or large white spots. Tonsils are enlarged to 2+, 3+, or 4+ with an acute infection.

24. While performing an assessment of the mouth, the nurse notices that the patient has a 1-cm ulceration that is crusted with an elevated border and located on the outer third of the lower lip. What other information would be most important for the nurse to assess? a. Nutritional status b. When the patient first noticed the lesion c. Whether the patient has had a recent cold d. Whether the patient has had any recent exposure to sick animals

B With carcinoma, the initial lesion is round and indurated, but then it becomes crusted and ulcerated with an elevated border. Most cancers occur between the outer and middle thirds of the lip. Any lesion that is still unhealed after 2 weeks should be referred. Therefore, the nurse should try to establish how long the lesion has been there and ask the patient when the patient first noticed the lesion.

The nurse is performing an examination of the anus and rectum. Which of these statements is correct and important to remember during this examination? A) The rectum is about 8 cm long. B) The anorectal junction cannot be palpated. C) Above the anal canal, the rectum turns anteriorly. D) There are no sensory nerves in the anal canal or rectum.

B) The anorectal junction cannot be palpated. The anal columns are folds of mucosa that extend vertically down from the rectum and end in the anorectal junction. This junction is not palpable but is visible on proctoscopy. The rectum is 12 cm long; just above the anal canal, the rectum dilates and turns posteriorly.

Which of these statements about the sphincters is correct? A) The internal sphincter is under voluntary control. B) The external sphincter is under voluntary control. C) Both sphincters remain slightly relaxed at all times. D) The internal sphincter surrounds the external sphincter.

B) The external sphincter is under voluntary control. The external sphincter surrounds the internal sphincter but also has a small section overriding the tip of the internal sphincter at the opening. The external sphincter is under voluntary control. Except for the passing of feces and gas, the sphincters keep the anal canal tightly closed.

Which of the following questions would be appropriate for the nurse to ask when a patient complains of diarrhea? (Select all that apply.) Select all that apply. "Have you ever had clay-colored stools?" "Have you traveled recently?" "Do you eat breakfast?" "Have you eaten at a restaurant in the past few days?" Have you recently been prescribed antibiotics?"

B, D, E. Noninflammatory diarrhea is associated with travel, foodborne illness, and may be a side effect of antibiotics. Eating breakfast is a question that should be routinely asked as part of the subjective GI assessment. Clay-colored stools indicate absent bile pigment associated with liver disease or gallstones.

Which of the following pairs of sinuses is absent at birth, is fairly well developed between 7 and 8 years of age, and is fully developed after puberty? Ethmoid Frontal Maxillary Sphenoid

B. The frontal sinuses are absent at birth, are fairly well developed between 7 and 8 years of age, and reach full size after puberty. The maxillary sinuses are present at birth and reach full size after all permanent teeth have erupted. The sphenoid sinuses are minute at birth and develop after puberty. The ethmoid sinuses are present at birth and grow rapidly between 6 and 8 years of age and after puberty.

What is the major cause of decreased saliva production in older adults? Decreased fluid intake Use of anticholinergic medications A diminished sense of taste and smell Normal aging process

B. The major cause of decreased saliva flow is the use of medications that have anticholinergic effects. Normal aging is a secondary cause of decreased saliva flow. Decreased fluid intake is not the major cause of decreased saliva production in an older adult. Diminished sense of taste and smell associated with aging may decrease an older adult's interest in food and may contribute to malnutrition.

One of the purposes of the paranasal sinuses is to amplify sound. lighten the weight of the skull bones. warm and moisten the inspired air. augment the sensory sensation of smell.

B. The paranasal sinuses lighten the weight of the skull bones. Nasal mucosa and nasal turbinates warm, humidify, and filter the inhaled air. The paranasal sinuses serve as resonators for sound production. Olfactory receptors (responsible for the sensation of smell) are located in the nasal cavity and septum and merge into the olfactory nerve.

The nurse is collecting a menstrual history on a new 34 year old female patient. The patient states that she has not had a menses for the past 5 months. What term will the nurse use to describe this symptom in the patient's medical record? A. Pregnancy B. Amenorrhea C. Dysmenorrhea D. Menorrhagia

B. Amenorrhea

An anorectal fistula is usually caused by a tear in the superficial mucosa. a chronically inflamed gastrointestinal tract. daily use of laxatives. trauma from passing hard stools.

B. An anorectal fistula is caused by a chronically inflamed gastrointestinal tract. Fissures are tears that occur in the superficial mucosa and often result from trauma (e.g., passing a large, hard stool) or from irritant diarrheal stools. Long-term use of laxatives may lead to dependence.

When assessing the range of motion of a patient's hip, the nurse knows this joint should be able to perform all of the following movements except: A. Abduction/adduction B. Elevation/depression C. Rotation D. Flexion/extension

B. Elevation/depression

An adolescent male is brought to the emergency department with complaints of excruciating pain in his left testicle. Which of the following would be the nurse's most appropriate action? Perform a focused assessment. Notify the emergency department physician immediately. Document pain assessment and notify physician when he or she is available. Tell the adolescent, everything will be fine and there's nothing to worry about.

B. Excruciating unilateral testicle pain of sudden onset are signs of testicular torsion which is considered an emergency. The adolescent needs to have emergency surgery to prevent gangrene which can occur in only a few hours. Symptoms indicate an emergent condition, thus the nurse should notify the physician immediately and allow them to perform the needed assessment. Telling a patient and/or his family there is nothing to worry about is non-therapeutic communication and provides false reassurance.

An expected postmenopausal breast change is decreased fibrous connective tissue. decreased breast size. increased fatty tissue. increased glandular tissue.

B. Expected breast changes after menopause include a decrease in breast size. Expected breast changes after menopause include breast glandular tissue atrophy. Expected breast changes after menopause include increased fibrous connective tissue. Expected breast changes after menopause include fat tissue atrophy.

The divisions of the spinal vertebrae include cervical, thoracic, scaphoid, sacral, and clavicular. cervical, thoracic, lumbar, sacral, and coccygeal. cervical, lumbar, iliac, synovial, and capsular. scapular, clavicular, lumbar, scaphoid, and fasciculi.

B. Humans have 7 cervical, 12 thoracic, 5 lumbar, 5 sacral, and 3 to 4 coccygeal vertebrae.

When performing a genitourinary assessment of a male infant, the nurse notes the urethral meatus is positioned on the ventral aspect of the glans. The nurse knows that this finding is characteristic of: A. Phimosis B. Hypospadias C. Chorded D. A normal finding

B. Hypospadias

In order to obtain accurate subjective assessment data from a female adolescent, what would be the most appropriate action by the nurse? Stand when asking questions. Ask parent/caregiver to step out of the room and return to the lobby. Share what life was like for the nurse at the patient's age. Ask questions such as, "you're not sexually active, right?"

B. In order to gain trust and provide privacy for the adolescent female, it would be most appropriate to ask the parent/caregiver to step out of the room. The nurse should rarely share personal information especially r/t female genitalia and sexuality. Asking questions in a negatively worded manner does not provide an open environment for the adolescent female to share concerns. Whenever interviewing/talking with patients, the nurse should use therapeutic communication techniques which includes sitting or being at their eye level.

During the HEENT assessment of a 19-year-old Asian male patient, the nurse notices he has dry , flaky cerumen in his ear canal. The nurse understands that this finding: A. Could indicate an ear infection, and the provider should be notified. B. Is a normal finding and no further follow up is necessary. C. Represents poor hygiene. D. Is likely the result of eczema lesions in his ear.

B. Is a normal finding and no further follow up is necessary.

The nurse is reviewing the medical record of a 54-year-old female with Multiple Sclerosis and notes that nystagmus was present on the patient's last eye exam. The nurse know that nystagmus is identified by which of the following. A. Droopy eyelids that partially or completely cover the pupil B. Jerky eye movements during the 6 cardinal positions of gaze test C. One eye gazing in a different direction during the cover/uncover test D. Asymmetric pupillary response during the pupillary light reflex test

B. Jerky eye movements during the 6 cardinal positions of gaze test

The nurse is performing a physical assessment on a patient who is 37 weeks pregnant and notes that there is a forward curvature of the thoracic spine. The nurse identifies this finding as: A. Abnormal; the doctor should be notified B. Kyphosis C. Scoliosis D. Lordosis

B. Kyphosis

Which of the following documentation statements indicates a normal assessment of the perianal area? "Small round opening in the anal area." "Anus moist, color darker than adjacent tissues. No lesions or discharge. Opening tightly closed." "Anal area dark pink, moist, with 0.5-cm shiny blue skin sac at 5 o'clock." "Anus with flabby skin sac at 7 o'clock."

B. Normal findings during inspection of the perianal area include anus moist and hairless, with coarse folded skin that is more pigmented than the perianal skin, anal opening that is tightly closed, and no lesions present. A shiny blue skin sac is a thrombosed hemorrhoid. A flabby skin sac is a hemorrhoid. A small round opening in the anal area is a fistula.

The extrapyramidal system is located in the medulla. basal ganglia. hypothalamus. cerebellum.

B. The basal ganglia are large bands of gray matter buried deep within the two cerebral hemispheres that form the subcortical associated motor system (the extrapyramidal system).

The group of axillary lymph nodes that drains the other three groups of nodes is the anterior nodes. central nodes. posterior nodes. lateral nodes.

B. The central axillary nodes receive lymph from the other three groups of nodes (i.e., anterior [pectoral], posterior [subscapular], and lateral).

Cerebellar function is tested by muscle strength assessment. performance of rapid alternating movements. the Phalen maneuver. superficial pain and touch assessment.

B. The cerebellum controls motor coordination of voluntary movements, equilibrium, and muscle tone. Cerebellar function is tested by balance tests (e.g., gait, Romberg test) and coordination and skilled movements (e.g., rapid alternating movements, finger-to-finger test, finger-to-nose test, heel-to-shin test). Muscle strength assessment examines the intactness of the motor system. The Phalen maneuver reproduces numbness and burning in a patient with carpal tunnel syndrome. Superficial pain and touch assessment examines intactness of the spinothalamic tract.

When assessing for the presence of a herniated nucleus pulposus, the examiner would instruct the patient to do a knee bend. raise each of the patient's legs straight while keeping the knee extended. abduct and adduct the patient's legs while keeping the knee extended. don't raise each of the patient's legs straight while keeping the knee extended.

B. The straight leg-raising (Lasègue) test reproduces back and leg pain and helps confirm the presence of a herniated nucleus pulposus. The examiner raises each leg straight while keeping the knee in extension. To assess for a spinal curvature, the examiner has the person bend over and touch the toes with the knee in extension. Muscle extension can be assessed by instructing the person to rise from a squatting position without using the hands for support. To assess range of motion, the leg should be abducted and adducted with the knee extended.

When assessing range of motion, which of the following would be important for the nurse to do to help the patient understand what the nurse is asking? Anchor the joint with one hand while other hand slowly moves the joint to its limit. Model the movements Palpate each joint. Familiarize himself or herself with type of each joint and normal range of motion

B. When assessing active range of motion, it's helpful for the nurse to demonstrate the movements to the patient rather than just verbally trying to describe the desired movement. It's important for the nurse to understand the type of joints and expected movement, however, this is not something that will most help the patient understand the assessment techniques being performed. When the nurse notes joint limitation, he or she would anchor the joint with one hand while other hand slowly moves the joint to its limit. Palpation of joints to assess movement and for presence of crepitation is part of the assessment techniques but is not something that will help the patient understand what's asked.

The nurse notices that an infant has a large, soft lump on the side of his head* and that his mother is very concerned. She tells the nurse that she noticed the lump about 8 hours after her baby's birth, and that it seems to be getting bigger. One possible explanation for this is: a. Hydrocephalus. b. Craniosynostosis. c. Cephalhematoma. d. Caput succedaneum.

C A cephalhematoma is a subperiosteal hemorrhage that is the result of birth trauma. It is soft, fluctuant, and well defined over one cranial bone. It appears several hours after birth and gradually increases in size.

During an admission assessment, the nurse notices that a male patient has an enlarged and rather thick skull. The nurse suspects *acromegaly* and would further assess for: a. Exophthalmos. b. Bowed long bones. c. Coarse facial features. d. Acorn-shaped cranium.

C Acromegaly is excessive secretion of growth hormone that creates an enlarged skull and thickened cranial bones. Patients will have elongated heads, massive faces, prominent noses and lower jaws, heavy eyebrow ridges, and coarse facial features. Exophthalmos is associated with hyperthyroidism. Bowed long bones and an acorn-shaped cranium result from Paget's disease.

During an examination of a female patient, the nurse notes lymphadenopathy and suspects an acute infection. *Acutely infected lymph nodes* would be: a. Clumped. b. Unilateral. c. Firm but freely movable. d. Firm and nontender.

C Acutely infected lymph nodes are bilateral, enlarged, warm, tender, and firm but freely movable. Unilaterally enlarged nodes that are firm and nontender may indicate cancer.

A mother brings in her newborn infant for an assessment and tells the nurse that she has noticed that whenever her *newborn's head is turned to the right side, she straightens out the arm and leg on the same side and flexes the opposite arm and leg*. After finding this on examination, the nurse would tell her that this is: a. Abnormal and is called the atonic neck reflex. b. Normal and should disappear by the first year of life. c. Normal and is called the tonic neck reflex, which should disappear between 3 and 4 months of age. d. Abnormal. The baby should be flexing the arm and leg on the right side of his body when the head is turned to the right.

C By 2 weeks the infant shows the tonic neck reflex when supine and the head is turned to one side (extension of same arm and leg, flexion of opposite arm and leg). The tonic neck reflex disappears between 3 and 4 months of age.

A visitor from Poland who does not speak English seems to be somewhat apprehensive about the nurse examining his neck. He would probably be most comfortable with the nurse examining his thyroid from: a. Behind with the nurse's hands placed firmly around his neck. b. The side with the nurse's eyes averted toward the ceiling and thumbs on his neck. c. The front with the nurse's thumbs placed on either side of his trachea and his head tilted forward. d. The front with the nurse's thumbs placed on either side of his trachea and his head tilted backward.

C Examining this patient's thyroid from the back may be unsettling for him. It would be best to examine his thyroid using the anterior approach, asking him to tip his head forward and to the right and then the left.

The nurse notices that a patient's *palpebral fissures* are not symmetrical. On examination, the nurse may find that there has been damage to cranial nerve: a. III b. V c. VII d. VIII

C Facial muscles are mediated by cranial nerve (CN) VII; asymmetry of palpebral fissures may be due to CN VII damage (Bell's palsy).

During a well-baby check, the nurse notices that a 1-week-old *infant's face looks small compared with his cranium, which seems enlarged*. On further examination, the nurse also notices dilated scalp veins and downcast, or "setting sun," eyes. The nurse suspects which condition? a. Craniotabes b. Microcephaly c. Hydrocephalus d. Caput succedaneum

C Hydrocephalus occurs with obstruction of drainage of cerebrospinal fluid that results in excessive accumulation, increasing intracranial pressure, and enlargement of the head. The face looks small compared with the enlarged cranium, and dilated scalp veins and downcast, or "setting sun," eyes are noted. Craniotabes is a softening of the skull's outer layer. Microcephaly is an abnormally small head. A caput succedaneum is edematous swelling and ecchymosis of the presenting part of the head caused by birth trauma.

A patient's thyroid is enlarged, and the nurse is preparing to auscultate the thyroid* for the presence of a bruit. A bruit is a __________ of the stethoscope. a. Low gurgling; diaphragm b. Loud, whooshing, blowing; bell c. Soft, whooshing, pulsatile; bell d. High-pitched tinkling; diaphragm

C If the thyroid gland is enlarged, the nurse should auscultate it for the presence of a bruit, which is a soft, pulsatile, whooshing, blowing sound heard best with the bell of the stethoscope.

A woman comes to the clinic and states, "I've been sick for so long! My *eyes have gotten so puffy, and my eyebrows and hair have become coarse and dry*." The nurse will assess for other signs and symptoms of: a. Cachexia. b. Parkinson syndrome. c. Myxedema. d. Scleroderma.

C Myxedema (hypothyroidism) is a deficiency of thyroid hormone that, when severe, causes a nonpitting edema or myxedema. The patient will have a puffy edematous face especially around eyes (periorbital edema), coarse facial features, dry skin, and dry, coarse hair and eyebrows. See Table 13-4, Abnormal Facial Appearances with Chronic Illnesses, for descriptions of the other responses.

While performing a well-child assessment on a 5 year old, the nurse notes the *presence of palpable, bilateral, cervical, and inguinal lymph nodes*. They are approximately 0.5 cm in size, round, mobile, and nontender. The nurse suspects that this: a. Has chronic allergies. b. May have an infection. c. Is exhibiting a normal finding for a well child of this age. d. Should be referred for additional evaluation.

C Palpable lymph nodes are normal in children until puberty when the lymphoid tissue begins to atrophy. Lymph nodes may be up to 1 cm in size in the cervical and inguinal areas, but are discrete, movable, and nontender.

During an examination, the nurse finds that a patient's *left temporal artery is tortuous and feels hardened and tender compared with the right temporal artery*. The nurse suspects which condition? a. Crepitation b. Mastoiditis c. Temporal arteritis d. Bell palsy

C The artery looks more tortuous and feels hardened and tender with temporal arteritis. These assessment findings are not consistent with the other responses.

A patient, an 85-year-old woman, is complaining about the fact that the bones in her face have become more noticeable. What explanation should the nurse give to her? a. Diets low in protein and high in carbohydrates may cause enhanced facial bones. b. Bones can become more noticeable if the person does not use a dermatologically approved moisturizer. c. More noticeable facial bones are probably due to a combination of factors related to aging, such as decreased elasticity, subcutaneous fat, and moisture in her skin. d. Facial skin becomes more elastic with age. This increased elasticity causes the skin to be more taught, drawing attention to the facial bones.

C The facial bones and orbits appear more prominent in the aging adult, and the facial skin sags owing to decreased elasticity, decreased subcutaneous fat, and decreased moisture in the skin.

The nurse needs to palpate the temporomandibular joint for crepitation. This joint is located just below the temporal artery and anterior to the: a. Hyoid bone. b. Vagus nerve. c. Tragus. d. Mandible.

C The temporomandibular joint is just below the temporal artery and anterior to the tragus.

During an examination of a patient in her third trimester of pregnancy, the nurse notices that the *patient's thyroid gland is slightly enlarged*. No enlargement had been noticed previously. The nurse suspects that: a. Has an iodine deficiency. b. Is exhibiting early signs of goiter. c. Is exhibiting a normal enlargement of the thyroid gland during pregnancy. d. Needs further testing for possible thyroid cancer.

C The thyroid gland enlarges slightly during pregnancy because of hyperplasia of the tissue and increased vascularity.

When examining the face, the nurse is aware that the *two pairs of salivary glands* that are accessible to examination are the _____ glands. a. Occipital; submental b. Parotid; jugulodigastric c. Parotid; submandibular d. Submandibular; occipital

C Two pairs of salivary glands accessible to examination on the face are the parotid glands, which are in the cheeks over the mandible, anterior to and below the ear; and the submandibular glands, which are beneath the mandible at the angle of the jaw. The parotid glands are not normally palpable.

A physician tells the nurse that a patient's *vertebra prominens* is tender and asks the nurse to reevaluate the area in 1 hour. The area of the body the nurse will assess is the area: a. Just above the diaphragm. b. Just lateral to the knee cap. c. At the level of the C7 vertebra. d. At the level of the T11 vertebra.

C The C7 vertebra has a long spinous process, called the vertebra prominens, that is palpable when the head is flexed.

27. The nurse is assessing a patient in the hospital who has received numerous antibiotics for a lung infection and notices that his tongue appears to be black and hairy. In response to his concern, what would the nurse say? a. "We will need to get a biopsy to determine the cause." b. "This is an overgrowth of hair and will go away in a few days." c. "Black, hairy tongue is a fungal infection caused by all the antibiotics you have received." d. "This is probably caused by the same bacteria you had in your lungs."

C A black, hairy tongue is not really hair but the elongation of filiform papillae and painless overgrowth of mycelial threads of fungus infection on the tongue. It occurs after the use of antibiotics, which inhibit normal bacteria and allow a proliferation of fungus. It is not caused by the same bacteria as his lung infection but occurred after the use of antibiotics, which inhibit normal bacteria and allow a proliferation of fungus. There is no need to get a biopsy.

22. The nurse is assessing a 3-year-old for "drainage from the nose." On assessment, a purulent drainage that has a very foul odor is noted from the left naris and no drainage is observed from the right naris. The child is afebrile with no other symptoms. What should the nurse do next? a. Refer to the physician for an antibiotic order. b. Have the mother bring the child back in 1 week. c. Perform an otoscopic examination of the left nares. d. Tell the mother that this drainage is normal for a child of this age.

C Children are prone to put an object up the nose, producing unilateral purulent drainage with a foul odor. Because some risk for aspiration exists, removal should be prompt.

23. During an assessment of a 26-year-old for "a spot on my lip I think is cancer," the clinic nurse notices a group of clear vesicles with an erythematous base around them located at the lip-skin border. The patient mentions that she just returned from Hawaii. What is the most appropriate action by the nurse? a. Tell the patient she needs to see a skin specialist. b. Discuss the benefits of having a biopsy performed on any unusual lesion. c. Tell the patient that these vesicles are indicative of herpes simplex I or cold sores and that they will heal in 4 to 10 days. d. Tell the patient that these vesicles are most likely the result of a riboflavin deficiency and discuss nutrition.

C Cold sores are groups of clear vesicles with a surrounding erythematous base. These evolve into pustules or crusts and heal in 4 to 10 days. The most likely site is the lip-skin junction. Infection often recurs in the same site. Recurrent herpes infections may be precipitated by sunlight, fever, colds, or allergy.

20. Immediately after birth, the nurse is unable to suction the nares of a crying newborn. An attempt is made to pass a catheter through both nasal cavities with no success. What should the nurse do next? a. Attempt to suction again with a bulb syringe. b. Wait a few minutes, and try again once the infant stops crying. c. Recognize that this situation requires immediate intervention. d. Contact the physician to schedule an appointment for the infant at his or her next hospital visit.

C Determining the patency of the nares in the immediate newborn period is essential because most newborns are obligate nose breathers. Nares blocked with amniotic fluid are gently suctioned with a bulb syringe. If obstruction is suspected, then a small lumen (5 to 10 Fr) catheter is passed down each naris to confirm patency. The inability to pass a catheter through the nasal cavity indicates choanal atresia, which requires immediate intervention.

7. The nurse is assessing an 80-year-old patient. Which of these findings would be expected for this patient? a. Hypertrophy of the gums b. Increased production of saliva c. Decreased ability to identify odors d. Finer and less prominent nasal hair

C Dysphagia is difficulty with swallowing and may occur with a variety of disorders, including stroke and other neurologic diseases. Rhinorrhea is a runny nose, epistaxis is a bloody nose, and xerostomia is a dry mouth. Rhinorrhea is a runny nose, epistaxis is a bloody nose, and xerostomia is a dry mouth; none of which are expected findings in a patient who had a stroke with drooping on the right side of the face. Dysphagia is difficulty with swallowing and may occur with a variety of disorders, including stroke and other neurologic diseases.

16. During an oral assessment of a 30-year-old black patient, the nurse notices bluish lips and a dark line along the gingival margin. What action would the nurse perform in response to this finding? a. Check the patient's Hb for anemia. b. Assess for other signs of insufficient oxygen supply. c. Proceed with the assessment, this appearance is a normal finding. d. Ask if he has been exposed to an excessive amount of carbon monoxide.

C Some blacks may have bluish lips and a dark line on the gingival margin; this appearance is a normal finding so the nurse should proceed with the assessment. Some blacks may have bluish lips and a dark line on the gingival margin, so this is a normal finding and there is no need to check the Hb for anemia, assess for other signs of insufficient oxygen supply, or ask if he has been exposed to an excessive amount of carbon monoxide. Instead, the nurse should continue with the assessment.

1. What is the primary purpose of the ciliated mucous membrane in the nose? a. To warm the inhaled air b. To filter out dust and bacteria c. To filter coarse particles from inhaled air d. To facilitate the movement of air through the nares

C The nasal hairs, or cilia, filter the coarsest matter from inhaled air, whereas the mucous blanket filters out dust and bacteria. The rich blood supply of the nasal mucosa warms the inhaled air. The rich blood supply of the nasal mucosa warms the inhaled air, not the ciliated mucous membrane. The mucous blanket, not the cilia, filters out dust and bacteria. The cilia in the nose do not facilitate the movement of air through the nares. Instead, the nasal hairs, or cilia, filter the coarsest matter from inhaled air.

21. The nurse notices that the mother of a 2-year-old boy brings him into the clinic quite frequently for various injuries and suspects there may be some child abuse involved. What should the nurse look for during an inspection of this child's mouth? a. Swollen, red tonsils b. Ulcerations on the hard palate c. Bruising on the buccal mucosa or gums d. Small yellow papules along the hard palate

C The nurse should notice any bruising or laceration on the buccal mucosa or gums of an infant or young child. Trauma may indicate child abuse from a forced feeding of a bottle or spoon.

2. What are the projections in the nasal cavity that increase the surface area are called? a. Meatus b. Septum c. Turbinates d. Kiesselbach plexus

C The projections in the nasal cavity that increases the surface area are called turbinates. The lateral walls of each nasal cavity contain three parallel bony projections: the superior, middle, and inferior turbinates. These increase the surface area, making more blood vessels and mucous membrane available to warm, humidify, and filter the inhaled air. A meatus is the passageway or canal underlying each turbinate that collects drainage. The septum is what divides the nasal cavity into two slitlike air passages. The Kiesselbach plexus is a rich vascular network in the anterior part of the septum.

14. The nurse is performing an assessment on a 21-year-old patient and notices that his nasal mucosa appears pale, gray, and swollen. What would be the most appropriate question to ask the patient? a. "Have you had any symptoms of a cold?" b. "Do you have an elevated temperature?" c. "Are you aware of having any allergies?" d. "Have you been having frequent nosebleeds?"

C With chronic allergies, the mucosa looks swollen, boggy, pale, and gray. Elevated body temperature, colds, and nosebleeds do not cause these mucosal changes. Elevated body temperature, colds, and nosebleeds do not cause the nasal mucosa to appear pale, gray, and swollen. Chronic allergies do cause the mucosa to look swollen, boggy, pale, and gray.

If the tympanic membrane has white dense areas, the examiner suspects a fungal infection. serous fluid from serous otitis media. scarring from recurrent ear infections. perforation from a ruptured membrane.

C. White dense areas indicate scarring on the tympanic membrane from recurrent ear infections. Dark oval areas indicate perforation from a ruptured tympanic membrane. Air or fluid levels or air bubbles indicate serous fluid from serous otitis media. Black or white dots indicate a fungal infection.

When performing a muscular strength exam on a geriatric patient, the nurse notes that the patient is only able to perform range of motion against gravity without resistance. The nurse would document this patient's strength score as a: A. 0 B. 1 C. 3 D. 4

C. 3

What term is used to describe slow, twisting muscle movements that resemble a snake or worm? Ataxia Vestibular function Athetosis Flaccid

C. Athetosis is slow, writhing, continuous, and involuntary movements of the extremities. Ataxia is an impaired ability to coordinate movement, often characterized by a staggering gait and postural imbalance. Flaccid is weak, soft, and flabby, lacking normal muscle tone. Vestibular function is the sense of balance.

Which of the following are findings r/t testosterone deficiency? Increased libido Scrotal sac pendulous with less rugae Increased fatigue Decreased penis size

C. Depression, fatigue, loss of muscle mass or strength, and decreased libido are common findings in males with testosterone deficiency. Decreased libido is expected with testosterone deficiency. Pendulous scrotal sac with less rugae is a normal expected finding in an older male. Decreased penis size is a normal expected finding in an older male.

A 58 year old patient diagnosed with prostatitis is complaining of burning pain during urination. The nurse would document this symptom as: A. Dystrophy B. Stress incontinence C. Dysuria D. Anuria

C. Dysuria

The nurse is caring for a patient with Bell's Palsy. Knowing that this disorder affects the facial nerve (CN VII), the nurse expects the patient to have which of the following exam findings? A. Impaired hearing B. Extraocular eye movements C. Facial paralysis D. Asymmetric gag reflex

C. Facial paralysis

Soft, pointed, fleshy papules that occur on the genitalia caused by human papillomavirus (HPV) are known as chancres. urethritis. genital warts. varicoceles.

C. Genital warts are soft, pointed, fleshy papules that occur on the genitalia and are caused by HPV. Syphilitic chancres are small, solitary, silvery papules that erode to a red, round or oval, superficial ulcer with a yellowish serous discharge. Urethritis is an infection of the urethra, the meatus edges are reddened, everted, and swollen. A varicocele is a dilated, tortuous varicose vein in the spermatic cord.

Hematuria is a term used for bleeding after intercourse. bloody discharge. blood in the urine. urine in the blood.

C. Hematuria is the term used to describe blood in the urine.

A 25 year old college student presents tot he clinic for her first annual pelvic examination and Pap smear. For this examination, the nurse will place the patient in which of the following positions? A. Supine position B. Trendelenburg position C. Lithotomy position D. Sim's position

C. Lithotomy position

Orchitis is (are) hard, subcutaneous plaques associated with painful bending of the erect penis. a circumscribed collection of serous fluid in the tunica vaginalis surrounding the testes. an acute inflammation of the testes. a meatus opening on the dorsal side of the glans or shaft.

C. Orchitis is an acute inflammation of the testes. Epispadias is a meatus opening on the dorsal side of the glans or shaft. Peyronie disease is a result of hard, nontender, subcutaneous plaques on the penis that cause a painful bending of the penis during an erection. A hydrocele is a circumscribed collection of serous fluid in the tunica vaginalis surrounding the testes.

The _____________ coordinates movement, maintains equilibrium, and helps maintain posture. basal ganglia extrapyramidal system cerebellum upper and lower motor neurons

C. The cerebellum controls motor coordination of voluntary movements, equilibrium (i.e., posture balance of the body), and muscle tone. The extrapyramidal system maintains muscle tone and controls body movements, especially gross automatic movements such as walking. The upper motor neurons are located within the central nervous system, influence or modify the lower motor neurons, and include the corticospinal, corticobulbar, and extrapyramidal tracts. The lower motor neurons are located mostly in the peripheral nervous system and extend from the spinal cord to the muscles, examples include the cranial nerves and spinal nerves. The basal ganglia control automatic associated movements of the body.

An older woman is having an annual mammogram. Before the mammogram, the nurse does a breast examination. Expected normal findings would include increased glandularity. a unilateral venous pattern. palpable, firm, stringy lactiferous ducts. yellow colostrum expressed from the nipple.

C. The lactiferous ducts of an older woman (eighth to ninth decades) are more palpable and feel firm and stringy because of fibrosis and calcification. The glandular breast tissue in an older woman atrophies. Yellow colostrum may be expressed from the nipple of a pregnant woman after the first trimester. A venous pattern is prominent over the skin surface of pregnant women.

The presence of primitive reflexes in a newborn infant is indicative of prematurity of the infant. mental retardation. immaturity of the nervous system. spinal cord alterations.

C. The nervous system is not completely developed at birth, and motor activity in the newborn is under the control of the spinal cord and medulla. The neurons are not yet myelinated. Movements are directed primarily by primitive reflexes. As the cerebral cortex develops during the first year, it inhibits these reflexes, and they disappear at predictable times. Persistence of the primitive reflexes is an indication of central nervous system dysfunction.

A deep recess formed by the peritoneum between the rectum and the cervix is called a cystocele. the Chadwick sign. a rectouterine pouch. a rectocele.

C. The rectouterine pouch (or cul-de-sac of Douglas) is the deep recess between the rectum and the cervix. The cervical mucosa during the second month of pregnancy is blue, which is termed the Chadwick sign. The cervix may also turn blue in any condition causing hypoxia or venous congestion. A cystocele is an abnormality of the pelvic musculature in which the bladder prolapses into the vagina. A rectocele is an abnormality of the pelvic musculature in which the rectum prolapses into the vagina.

Vaginal lubrication during intercourse is produced by sebaceous glands. adrenal glands. Bartholin glands. Skene glands.

C. The vestibular (Bartholin) glands secrete clear lubricating mucus during intercourse. Paraurethral (Skene) glands are tiny, multiple glands that surround the urethral meatus. Sebaceous glands are microscopic glands in the skin that secrete an oily/waxy matter, called sebum, to lubricate the skin and hair. The adrenal glands are endocrine glands responsible for releasing hormones in conjunction with stress through the synthesis of corticosteroids and catecholamines.

When reviewing the medical record of an 82 year old patient with dementia, the NURS notes the patient has aphasia. Which behavior support this finding. A. Difficulty swallowing B. Mumbled speech C. Unable to speak D. Unable to recognize objects

C. Unable to speak

A 23 year old female presents with diminished hearing and difficulty with balance. Based on these symptoms the nurse know to assess which of the following cranial nerves? A. Vagus (CNX) B. Tochlear (CN IV) C. Vestibulocochlear (CN VIII) D. Accessory (CN XI)

C. Vestibulocochlear (CN VIII)

colorectal cancer

Cancer of the colon and rectum, S&S- Abdominal pain, change in bowel habits; bloody stools; weight loss; cachexia (wasting disorder linked with malnutrition); lymphadenopathy (disease of the lymph nodes). Most common symptomatic presentation is unexplained iron deficiency anemia. Risk increases with age; adenomatous polyps.

A 19-year-old college student is brought to the emergency department with a severe headache he describes as "Like nothing I've ever had before." His temperature is 104° F, and he has a stiff neck. The nurse looks for other signs and symptoms of which problem? a. Head injury b. Cluster headache c. Migraine headache d. Meningeal inflammation

D Acute onset of neck stiffness and pain along with headache and fever occurs with meningeal inflammation. A severe headache in an adult or child who has never had it before is a red flag. Head injury and cluster or migraine headaches are not associated with a fever or stiff neck.

A patient is unable to differentiate between *sharp and dull stimulation to both sides of her face*. The nurse suspects: a. Bell palsy. b. Damage to the trigeminal nerve. c. Frostbite with resultant paresthesia to the cheeks. d. Scleroderma.

D Facial sensations of pain or touch are mediated by cranial nerve (CN) V, which is the trigeminal nerve. Bell's palsy is associated with CN VII damage. Frostbite and scleroderma are not associated this problem.

A mother brings her 2-month-old daughter in for an examination and says, "My daughter rolled over against the wall and now I have noticed that she has this *spot that is soft* on the top of her head. Is there something terribly wrong?" The nurse's best response would be: a. "Perhaps that could be a result of your dietary intake during pregnancy." b. "Your baby may have craniosynostosis, a disease of the sutures of the brain." c. "That 'soft spot' may be an indication of cretinism or congenital hypothyroidism." d. "That 'soft spot' is normal, and actually allows for growth of the brain during the first year of your baby's life."

D Membrane-covered "soft spots" allow for growth of the brain during the first year. They gradually ossify; the triangular-shaped posterior fontanel is closed by 1 to 2 months, and the diamond-shaped anterior fontanel closes between 9 months and 2 years.

A patient complains that while studying for an examination he began to notice a *severe headache in the frontotemporal area of his head that is throbbing and is somewhat relieved when he lies down. He tells the nurse that his mother also had these headaches. The nurse suspects that he may be suffering from: a. Hypertension. b. Cluster headaches. c. Tension headaches. d. Migraine headaches.

D Migraine headaches tend to be supraorbital, retro-orbital, or frontotemporal with a throbbing quality. They are of a severe quality and are relieved by lying down. Migraines are associated with family history of migraines.

The nurse is aware that the four areas in the body where lymph nodes are accessible* are the: a. Head, breasts, groin, and abdomen. b. Arms, breasts, inguinal area, and legs. c. Head and neck, arms, breasts, and axillae. d. Head and neck, arms, inguinal area, and axillae.

D Nodes are located throughout the body, but they are accessible to examination only in four areas: head and neck, arms, inguinal region, and axillae.

The nurse notices that a patient's submental lymph nodes are enlarged. In an effort to identify the cause of the node enlargement, the nurse would assess the patient's: a. Infraclavicular area. b. Supraclavicular area. c. Area distal to the enlarged node. d. Area proximal to the enlarged node.

D When nodes are abnormal, the nurse should check the area they drain for the source of the problem. Explore the area proximal (upstream) to the location of the abnormal node.

A patient visits the clinic because he has recently noticed that the left side of his mouth is paralyzed. He states that he cannot raise his eyebrow or whistle. The nurse suspects that he has: a. Cushing syndrome. b. Parkinson disease. c. Bell palsy. d. Experienced a cerebrovascular accident (CVA) or stroke.

D With an upper motor neuron lesion (as with CVA) the patient will have paralysis of lower facial muscles, but the upper half of the face is not affected owing to the intact nerve from the unaffected hemisphere. The person is still able to wrinkle the forehead and close the eyes. See Table 13-4, Abnormal Facial Appearances with Chronic Illnesses, for descriptions of the other responses.

29. A mother brings her 4-month-old infant to the clinic with concerns regarding a small pad in the middle of the upper lip that has been there since 1 month of age. The infant has no health problems. On physical examination, the nurse notices a 0.5-cm, fleshy, elevated area in the middle of the upper lip. No evidence of inflammation or drainage is observed. What would the nurse tell this mother? a. "This area of irritation is caused from teething and is nothing to worry about." b. "This finding is abnormal and should be evaluated by another health care provider." c. "This area of irritation is the result of chronic drooling and should resolve within the next month or two." d. "This elevated area is a sucking tubercle caused from the friction of breastfeeding or bottle-feeding and is normal."

D A normal finding in infants is the sucking tubercle, a small pad in the middle of the upper lip from the friction of breastfeeding or bottle-feeding. This condition is not caused by irritation, teething, or excessive drooling, and evaluation by another health care provider is not warranted.

25. A pregnant woman states that she is concerned about her gums because she has noticed they are swollen and have started to bleed. What would be an appropriate response by the nurse? a. "Your condition is probably due to a vitamin C deficiency." b. "I'm not sure what causes swollen and bleeding gums, but let me know if it's not better in a few weeks." c. "You need to make an appointment with your dentist as soon as possible to have this checked." d. "Swollen and bleeding gums can be caused by a change in hormonal balance during pregnancy."

D Although with gingivitis (which can be caused by a vitamin C deficiency) gum margins are red and swollen and easily bleed, a changing hormonal balance during puberty or pregnancy may also cause these symptoms. Since this patient is pregnant, a change in hormonal balance is likely the cause.

18. A 32-year-old woman is at the clinic for "little white bumps in my mouth." During the assessment, the nurse notes that she has a 0.5-cm white, nontender papule under her tongue and one on the mucosa of her right cheek. What would the nurse tell the patient? a. "These spots indicate an infection such as strep throat." b. "These bumps could be indicative of a serious lesion, so I will refer you to a specialist." c. "This condition is called leukoplakia and can be caused by chronic irritation such as with smoking." d. "These bumps are Fordyce granules, which are sebaceous cysts and are not a serious condition."

D Fordyce granules are small, isolated white or yellow papules on the mucosa of the cheek, tongue, and lips. These little sebaceous cysts are painless and are not significant. Chalky white, thick, raised patches would indicate leukoplakia. In strep throat, the examiner would see tonsils that are bright red, swollen, and may have exudates or white spots. In strep throat, the examiner would see tonsils that are bright red, swollen, and may have exudates or white spots and leukoplakia would appear as chalky white, thick, raised patches. These findings are not indicative of a serious lesion but are fordyce granules. Fordyce granules are small, isolated white or yellow papules on the mucosa of the cheek, tongue, and lips. These little sebaceous cysts are painless and are not significant.

6. The nurse is obtaining a health history on a 3-month-old infant. During the interview, the mother states, "I think she is getting her first tooth because she has started drooling a lot." What is the best response by the nurse? a. "You're right, drooling is usually a sign of the first tooth." b. "It would be unusual for a 3-month-old to be getting her first tooth." c. "This could be the sign of a problem with the salivary glands." d. "She is just starting to salivate and hasn't learned to swallow the saliva."

D In the infant, salivation starts at 3 months. The baby will drool for a few months before learning to swallow the saliva. This drooling does not herald the eruption of the first tooth, although many parents think it does. Although many parents think the start of drooling signals the eruption of the first tooth, it does not. Although teeth usually erupt between 6 and 24 months, the nurse should not just say it would be unusual for a 3-month-old to be getting her first tooth as that does not address the issue of the drooling. It is also not a sign of a problem.

39. A woman who is in the second trimester of pregnancy mentions that she has had "more nosebleeds than ever" since she became pregnant. What is the likely reason for this? a. Inappropriate use of nasal sprays b. A problem with the patient's coagulation system c. Increased susceptibility to colds and nasal irritation d. Increased vascularity in the upper respiratory tract as a result of the pregnancy

D Nasal stuffiness and epistaxis may occur during pregnancy as a result of increased vascularity in the upper respiratory tract. Inappropriate use of nasal sprays often causes rebound congestion or swelling, but not usually nosebleeds. Nasal stuffiness and epistaxis may occur during pregnancy as a result of increased vascularity in the upper respiratory tract so this patient's nose bleeds are more likely to be due to the increased vascularity in the upper respiratory tract than to a problem with the coagulation system or an increased susceptibility to colds and nasal irritation.

3. The nurse is reviewing the development of the newborn infant. Regarding the sinuses, which statement is true in relation to a newborn infant? a. Sphenoid sinuses are full size at birth. b. Maxillary sinuses reach full size after puberty. c. Frontal sinuses are fairly well developed at birth. d. Maxillary and ethmoid sinuses are the only sinuses present at birth.

D Only the maxillary and ethmoid sinuses are present at birth. The sphenoid sinuses are minute at birth and develop after puberty. The frontal sinuses are absent at birth, are fairly well developed at age 7 to 8 years, and reach full size after puberty. The sphenoid sinuses are minute at birth and develop after puberty. The frontal sinuses are absent at birth, are fairly well developed at age 7 to 8 years, and reach full size after puberty. Only the maxillary and ethmoid sinuses are present at birth but the maxillary sinus does not reach full size until all permanent teeth have erupted (not after puberty).

11. While obtaining a health history from the mother of a 1-year-old child, the nurse notices that the baby has had a bottle in his mouth the entire time. The mother states, "It makes a great pacifier." What is the best response by the nurse? a. "You're right. Bottles make very good pacifiers." b. "Using a bottle as a pacifier is better for the teeth than thumb-sucking." c. "It's okay to use a bottle as long as it contains milk and not juice." d. "Prolonged use of a bottle can increase the risk for tooth decay and ear infections."

D Prolonged bottle use during the day or when going to sleep places the infant at risk for tooth decay and middle ear infections.

32. When examining the nares of a 45-year-old patient who is experiencing rhinorrhea, itching of the nose and eyes, and sneezing, the nurse notices the following: pale turbinates, swelling of the turbinates, and clear rhinorrhea. Which of these conditions is most likely the cause? a. Nasal polyps b. Acute rhinitis c. Acute sinusitis d. Allergic rhinitis

D Rhinorrhea, itching of the nose and eyes, and sneezing are manifestations of allergic rhinitis. On physical examination, serous edema is noted, and the turbinates usually appear pale with a smooth, glistening surface. Nasal polyps appear as smooth, pale gray nodules which are overgrowths of mucosa most commonly caused by chronic allergic rhinitis and often cause absence of sense of smell and a sensation of a "valve that moves" in the nose when breathing. Acute rhinitis initially presents with clear, watery discharge (rhinorrhea) which later become purulent, with sneezing nasal itching, stimulation of cough reflex, and inflamed mucosa with dark red and swollen turbinates which cause nasal obstruction. With sinusitis, there is usually mucopurulent drainage, nasal obstruction, facial pain or pressure, and may have fever, chills, and malaise. This patient's symptoms of rhinorrhea, itching of the nose and eyes, and sneezing are manifestations of allergic rhinitis. On physical examination, serous edema is noted, and the turbinates usually appear pale with a smooth, glistening surface.

38. A patient comes into the clinic complaining of facial pain, fever, and malaise. On examination, the nurse notes swollen turbinates and purulent discharge from the nose. The patient also complains of a dull, throbbing pain in his cheeks and teeth on the right side and pain when the nurse palpates the areas. What do these findings indicate? a. Nasal polyps b. Frontal sinusitis c. Posterior epistaxis d. Maxillary sinusitis

D Signs of maxillary sinusitis include facial pain after upper respiratory infection, red swollen nasal mucosa, swollen turbinates, and purulent discharge. The person also has fever, chills, and malaise. With maxillary sinusitis, dull throbbing pain occurs in the cheeks and teeth on the same side, and pain with palpation is present. With frontal sinusitis, pain is above the supraorbital ridge. Nasal polyps appear as smooth, pale gray nodules which are overgrowths of mucosa most commonly caused by chronic allergic rhinitis and often cause absence of sense of smell and a sensation of a "valve that moves" in the nose when breathing. Epistaxis is a nosebleed and the most common site of bleeding is the Kiesselbach plexus in the anterior septum. With frontal sinusitis, pain is above the supraorbital ridge. This patient's signs and symptoms are indicative of maxillary sinusitis. Signs of maxillary sinusitis include facial pain after upper respiratory infection, red swollen nasal mucosa, swollen turbinates, and purulent discharge. The person also has fever, chills, and malaise. With maxillary sinusitis, dull throbbing pain occurs in the cheeks and teeth on the same side, and pain with palpation is present.

4. What is the tissue that connects the tongue to the floor of the mouth called? a. Uvula b. Palate c. Papillae d. Frenulum

D The frenulum is a midline fold of tissue that connects the tongue to the floor of the mouth. The uvula is the free projection hanging down from the middle of the soft palate. The palate is the arching roof of the mouth. Papillae are the rough, bumpy elevations on the tongue's dorsal surface. The uvula is the free projection hanging down from the middle of the soft palate. The palate is the arching roof of the mouth. Papillae are the rough, bumpy elevations on the tongue's dorsal surface. The frenulum is a midline fold of tissue that connects the tongue to the floor of the mouth

28. The nurse is assessing a patient with a history of intravenous drug abuse. In assessing his mouth, the nurse notices a dark red confluent macule on the hard palate. This could be an early sign of what disease or disorder? a. Measles b. Leukemia c. A carcinoma d. Acquired immunodeficiency syndrome (AIDS)

D This dark red confluent macule on the hard palate is an oral Kaposi's sarcoma. An oral Kaposi's sarcoma is a bruiselike, dark red or violet, confluent macule that usually occurs on the hard palate but may also appear on the soft palate or gingival margin. Oral lesions such as a Kaposi's sarcoma are among the earliest lesions to develop with AIDS.

34. The nurse is performing an assessment. Which of these findings would cause the greatest concern? a. A painful vesicle inside the cheek for 2 days b. The presence of moist, nontender Stensen's ducts c. Stippled gingival margins that snugly adhere to the teeth d. An ulceration on the side of the tongue with rolled edges

D Ulceration on the side or base of the tongue or under the tongue raises the suspicion of cancer and must be investigated. The risk for early metastasis is present because of rich lymphatic drainage. The vesicle may be an aphthous ulcer, which is painful but not dangerous. The other responses are normal findings. The presence of moist, nontender Stensen's ducts and stippled gingival margins that snugly adhere to the teeth are normal findings. Although a painful vesicle inside the cheek for 2 days is not that uncommon or concerning, but an ulceration on the side, base, or under the tongue raises the suspicion of cancer and must be investigated. The risk for early metastasis is present because of rich lymphatic drainage. The vesicle may be an aphthous ulcer, which is painful but not dangerous.

12. A 72-year-old patient has a history of hypertension and chronic lung disease. Which is an important question for the nurse to include in this patient's health history? a. "Do you use a fluoride supplement?" b. "Have you had tonsillitis in the last year?" c. "At what age did you get your first tooth?" d. "Have you noticed any dryness in your mouth?"

D With a history of hypertension and chronic lung disease, this patient is likely on medications and a side effect of antihypertensive and bronchodilator medication (and many other drugs such as antidepressants, anticholinergics, antispasmodics, and antipsychotics) is dry mouth, or xerostomia. The nurse should ask the patient if they've noticed dryness in their mouth.

Which of these statements about the anal canal is true? The anal canal: A) is about 2 cm long in the adult. B) slants backward toward the sacrum. C) contains hair and sebaceous glands. D) is the outlet for the gastrointestinal tract.

D) is the outlet for the gastrointestinal tract. The anal canal is the outlet for the gastrointestinal tract and is approximately 3.8 cm long in the adult. It is lined with a modified skin that does not contain hair or sebaceous glands, and it slants forward toward the umbilicus.

Which of the following questions would the examiner ask to determine whether an individual has epistaxis? "Have you ever noticed any unusual lesions on the inside of your mouth?" "Do you have any difficulty with swallowing?" "Do you experience a runny nose frequently?" "Do you experience nosebleeds?"

D. Epistaxis is the medical term for a nosebleed. Dysphagia is the medical term for difficulty swallowing. Rhinorrhea is the medical term for a runny nose.

An enlarged tongue (macroglossia) may accompany cleft palate. hairy tongue. fissured tongue. Down syndrome.

D. Macroglossia occurs with Down syndrome, it also occurs with cretinism, myxedema, and acromegaly. A transient swelling also occurs with local infections.

In addition to initiating digestion of food, saliva protects the mucosa from caustic substances. augments taste sensation. inhibits overgrowth of bacteria in the mouth. cleans and protects the mucosa.

D. Saliva moistens and lubricates the food bolus, starts digestion, and cleans and protects the mucosa.

The duct in the parotid gland that opens into the mouth opposite the second molar is the salivary duct. the Wharton's duct. the sublingual duct. the Stensen duct.

D. The duct in the parotid gland is the Stensen duct, it runs forward to open on the buccal mucosa opposite the second molar. The Wharton's duct (for the submandibular gland) runs up and forward to the floor of the mouth and opens at either side of the frenulum. The mouth contains three pairs of salivary glands, which are the parotid gland, the submandibular gland, and the sublingual gland. The sublingual gland lies within the floor of the mouth under the tongue.

The nasal mucosa of an individual with rhinitis would be moist and pink. swollen, boggy, and gray. pale with bright red bleeding. bright red and swollen.

D. The nasal mucosa is bright red and swollen with rhinitis. Normally, the nasal mucosa is red with a smooth and moist surface. The nasal mucosa is swollen, boggy, pale, and gray with chronic allergies. Bright red bleeding occurs with epistaxis (bleeding from the nose).

A caruncle is a(n) hard, painless nodule in the uterine wall. aberrant growth of endometrial tissue. vestibular gland located on either side of the vaginal orifice. small, red mass protruding from the urethral meatus.

D. A caruncle is a small, deep red mass protruding from the urinary meatus. Bartholin glands are vestibular glands located on either side of and posterior to the vaginal orifice. Endometriosis is a disorder caused by aberrant growths of endometrial tissue scattered throughout the pelvis. Myomas (leiomyomas or uterine fibroids) is a disorder in which the uterus is irregularly enlarged, firm, mobile, and nodular with hard, painless nodules in the uterine wall.

A known risk factor for breast cancer is low breast tissue density. low fat, low cholesterol diet. breastfeeding an infant for more than 6 months. early menarche or late menopause.

D. A risk factor for breast cancer is early menarche (before age 12 years) or late menopause (after age 55 years). A risk factor for breast cancer is high breast tissue density. A risk factor for breast cancer is never breastfeeding a child. A risk factor for breast cancer is a high fat diet.

Breast development in an adolescent girl usually begins between 12 and 13 years of age. occurs after the beginning of menstruation. takes an average of 5 years. precedes menstruation by about 2 years.

D. Breast development usually precedes menstruation by about 2 years. Breast development usually begins between 8 and 9 years of age for African American girls and by 10 years for white girls. Full breast development from Tanner stage 2 to 5 takes an average of 3 years, although the range is 1.5 to 6 years.

The most common sexually transmitted infection in the United States is trichomoniasis. syphilis. gonorrhea. chlamydia.

D. Chlamydia is the most common sexually transmitted infection in the United States.

A patient tells the examiner that passing stools is painful. What term would the examiner use to document painful bowel movements? Flatulence Encopresis Occult Dyschezia

D. Dyschezia is the painful passage of stool secondary to a local condition (e.g., hemorrhoids, fissure) or constipation. Occult is a term used to describe blood in the stools that is not visible. Flatulence is the passage of gas from the anus. Encopresis is persistent passing of stools into clothing in a child older than age 4 years, at which age continence would be expected.

Gynecomastia occurs with calcium channel blockers. Addison disease. hypothyroidism. liver cirrhosis.

D. Gynecomastia occurs in liver cirrhosis because the liver is unable to metabolize estrogens. Gynecomastia may occur in Cushing syndrome, not Addison disease. Gynecomastia may occur in hyperthyroidism. Gynecomastia may occur as an adverse effect of certain medications (e.g., metronidazole, isoniazid, digoxin, angiotensin-converting enzyme inhibitors, diazepam, and tricyclic antidepressants).

The nurse is conducting an admission assessment on a 41 year old patient who reports "when i had a bowel movement, i had paint in my bottom." The nurse should assess for which of the following problems? A. Fecal incontinence B. Colon cancer C. Imperforate anus D. Hemorrhoids

D. Hemorrhoids

On palpation the prostate gland is enlarged, nontender, firm, and smooth with a palpable central groove. This assessment finding indicates a normal prostate gland. prostatitis. prostate carcinoma. benign prostatic hypertrophy.

D. In benign prostatic hypertrophy, the prostate gland is enlarged, nontender, firm, and smooth with a palpable central groove. In prostatitis, the prostate gland is swollen and exquisitely tender. In prostatic carcinoma, the prostate gland is stone-hard and irregular with fixed nodules. A normal prostate gland does not protrude more than 1 cm into the rectum, is heart-shaped, with a palpable central groove, is smooth, is elastic, rubbery, and slightly movable, and is nontender to palpation.

A patient is taking iron supplements. The patient should expect the stools to be frothy. clay-colored. tarry and black. nontarry and black.

D. Iron medication supplements cause stools to be nontarry and black. Clay-colored stools are caused by the absence of bile pigment. Melena is the term to describe tarry, black stools, this indicates the presence of blood in the stool. Frothy stools have excessive fat from a malabsorption of fat, the term to describe fatty stools is steatorrhea.

During assessment of extraocular movements, two back-and-forth oscillations of the eyes in the extreme lateral gaze occur. This response indicates that the patient needs to be referred for a more complete eye examination. this assessment should be repeated in 15 minutes to allow the eyes to rest. a disease of the vestibular system, further evaluation is needed. an expected movement of the eyes during this procedure.

D. Nystagmus is a back-and-forth oscillation of the eyes. End-point nystagmus, a few beats of horizontal nystagmus at extreme lateral gaze, occurs normally.

During a routine examination of a 13 year old male patient, the nurse notes bilateral breast enlargement. Which of the following actions should the nurse take first? A. Explain this is a temporary condition caused by hormonal changes B. Inform the parents that a mammogram and breast biopsy must be performed C. Refer the patient to a cosmetic surgeon for excess breast tissue removal D. Palpate the breasts to assess for the presence of lumps

D. Palpate the breasts to assess for the presence of lumps

An abnormal sensation of burning or tingling is best described as paralysis. paraphasia. paresis. paresthesia.

D. Paresthesia is an abnormal sensation such as burning or tingling. Paralysis is a loss of motor function as a result of a lesion in the neurologic or muscular system or loss of sensory innervation. Paresis is a partial or incomplete paralysis. Paraphasia is a condition in which a person hears and comprehends words but is unable to speak correctly, incoherent words are substituted for intended words.

The nurse is assessing a patient who experience a thrombotic stroke. Which of these is the priority assessment? A. Electrocardiography (ECG) B. Lipid profile C. Swallow evaluation D. Pupillary response

D. Pupillary response

The nurse is performing a digital rectal examination on a 55 year old patient and finds a firm, irregularly shaped mass upon palpating. Which action should the nurse take? A. Instruct the patient to return in 4 weeks for a follow up examination B. Tell the patient that a mass was felt, but reassure that it is nothing to worry about C. Continue with the exam and document finding int he patient's medical record D. Report the finding to the provider.

D. Report the finding to the provider.

The American Academy of Pediatrics recommends newborn male circumcision because of which of the following benefits? Increased risk for HIV Increased risk for sexually transmitted infections Transmission of trichomoniasis Decreased risk for urinary tract infections

D. The American Academy of Pediatrics recommends newborn male circumcision as it shows decreased risk for urinary tract infections. Newborn male circumcision decreases the risk for sexually transmitted infection, transmission of trichomoniasis, and decreased risk for HIV transmission among heterosexual partners.

Which of the following is an expected finding of assessment of the male genitourinary system? a. Right testes 1.5 cm, Left testes 3 cm. b. Patient complains of urinary frequency c. Scrotum enlarged, rugae flattened. d. Patient states that he urinates 4 to 5 times per day.

D. The adult male typically urinates at least 4 to 5 times per day. An enlarged scrotum with flattened rugae is a sign of edema. Urinary frequency is usually associated with a urinary tract infection. Testes should be equal bilaterally and are freely movable.

Automatic associated movements of the body are under the control and regulation of the thalamus. the hypothalamus. Wernicke's area. the basal ganglia.

D. The basal ganglia controls automatic associated movements of the body. The thalamus is where sensory pathways of the spinal cord, cerebellum, and brainstem form synapses on their way to the cerebral cortex. The hypothalamus is a major respiratory center with basic vital functions: temperature, appetite, sex drive, heart rate, and blood pressure control, sleep center, anterior and posterior pituitary gland regulation, and coordination of autonomic nervous system activity and stress response. Wernicke's area in the temporal lobe is associated with language comprehension.

The knee joint is the articulation of which three bones? Femur, fibula, and patella. Femur, radius, and olecranon process. Fibula, tibia, and patella. Femur, tibia, and patella.

D. The knee joint is the articulation of the femur, the tibia, and the patella. The fibula is not involved in articulation of the knee joint. The radius and ulna are bones in the lower part of the upper extremity, the olecranon process is located on the proximal end of the ulna.

The most common site of cancerous breast tumors is in the upper inner quadrant. lower outer quadrant. lower inner quadrant. upper outer quadrant.

D. The most common site of breast cancer is in the upper outer quadrant.

Which of the following is the most reliable assessment method to assess for developmental dysplasia of the hip in an infant? Allis test Gluteal fold comparison Tibial torsion Ortolani maneuver

D. The most reliable method to check an infant's hips for developmental dysplasia of the hip is the Ortolani maneuver. Tibial torsion is a twisting of the tibia that may originate from intrauterine positioning. Unequal gluteal folds may be an indicator for developmental dysplasia of the hip, however, some asymmetry may occur in healthy children. The Allis test is also used to check for developmental dysplasia of the hip, but the Ortolani maneuver is the most reliable method.

When testing for muscle strength, the examiner should observe muscles for the degree of contraction when the individual lifts a heavy object. measure the degree of force that it takes to overcome joint flexion or extension. estimate the degree of flexion and extension in each joint. apply an opposing force when the individual puts a joint in flexion or extension.

D. The person should flex or extend muscle groups for each joint while the examiner applies an opposing force. Range of motion can be described by estimating (or measuring) the degree of flexion and extension of a joint.

The relative proportion of glandular, fibrous, and adipose breast tissue depends on genetics. sex. environmental factors. nutritional state.

D. The relative proportion of glandular, fibrous, and fatty tissue varies depending on age, cycle, pregnancy, lactation, and general nutritional state.

The term rugae refers to an acute inflammation of the testes. a muscle that controls the size of the scrotum. a corpus spongiosum cone of erectile tissue. folds of thin skin of the scrotal wall.

D. The scrotum wall consists of thin skin lying in folds, or rugae, and the underlying cremaster muscle. The penis is composed of three cylindric columns of erectile tissue: two corpora cavernosa on the dorsal side and the corpus spongiosum ventrally. The cremaster muscle controls the size of the scrotum. Orchitis is an acute inflammation of the testes.

The nurse is performing an assessment on a patient experiencing headache. Which of the following symptoms reported by the patient with indicate a cluster headache? A. Nuchal rigidity B. Projectile vomiting C. Photophobia and phonophobia D. Unilateral eyelid swelling

D. Unilateral eyelid swelling

epididymitis

Epididymitis is where a tube (the epididymis) at the back of the testicles becomes swollen and painful. It's often caused by an infection and is usually treated with antibiotics. If the testicles are also affected, it may be called epididymo-orchitis.

Prostate carcinoma

Most common tumor in men. Firm to hard nodules, Irregularities, unequal lobes, Induration, Stony hard prostate, elevated PSA, driven by testosterone

Which nervous system involves muscle movement?

Peripheral nervous system

Early testicular tumor

Solitary firm, harder-than-normal nodule; painless, found on examination, may have hx of undescended testicle or familial testicular cancer

testicular torsion

Testicular torsion occurs when a testicle rotates, twisting the spermatic cord that brings blood to the scrotum. The reduced blood flow causes sudden and often severe pain and swelling. Testicular torsion is most common between ages 12 and 18, but it can occur at any age, even before birth.

While gathering equipment after an injection, a nurse accidentally received a prick from an improperly capped needle. To interpret this sensation, which of these areas must be intact? a. Corticospinal tract, medulla, and basal ganglia b. Pyramidal tract, hypothalamus, and sensory cortex c. Lateral spinothalamic tract, thalamus, and sensory cortex d. Anterior spinothalamic tract, basal ganglia, and sensory cortex c. Lateral spinothalamic tract, thalamus, and sensory cortex

The spinothalamic tract contains sensory fibers that transmit the sensations of pain, temperature, and crude or light touch. Fibers carrying pain and temperature sensations ascend the lateral spinothalamic tract, whereas the sensations of crude touch form the anterior spinothalamic tract. At the thalamus, the fibers synapse with another sensory neuron, which carries the message to the sensory cortex for full interpretation. The other options are not correct. A patient with a lack of oxygen to his heart will have pain in his chest and possibly in the shoulder, arms, or jaw. The nurse knows that the best explanation why this occurs is which one of these statements?

anal fissure

a small crack-like sore in the skin of the anus that can cause severe pain during a bowel movement

While discussing the history of a 6-month-old infant, the mother tells the nurse that she took a great deal of aspirin while she was pregnant. What question would the nurse want to include in the history? a. "Does your baby seem to startle with loud noise?" b. "Has the baby had any surgeries on the ears?" c. "Have you noticed any drainage from her ears?" d. "How many ear infections has your baby had since birth?"

a. "Does your baby seem to startle with loud noise?" Children at risk for hearing deficit include those exposed in utero to a variety of conditions, such as maternal rubella, or to maternal ototoxic drugs.

During a well-baby checkup, a mother is concerned because her 2-month-old infant cannot hold her head up when she is pulled to a sitting position. Which response by the nurse is appropriate? a. "Head control is usually achieved by 4 months of age." b. "You shouldn't be trying to pull your baby up like that until she is older." c. "Head control should be achieved by this time." d. "This inability indicates possible nerve damage to the neck muscles."

a. "Head control is usually achieved by 4 months of age."

Which of the following statements describing a headache would warrant an immediate referral? a. "I have never had a headache like this before, it is so bad I can't function." b. "This is the worst headache I've had since puberty." c. "This is the worst migraine of my life." d. "I have had daily headaches for years."

a. "I have never had a headache like this before, it is so bad I can't function." A sudden severe headache in an adult or child who has never had it before warrants an immediate referral. A sudden severe headache could indicate a subarachnoid hemorrhage.

When performing an external genitalia examination of a 10-year-old girl, the nurse notices that no pubic hair has grown in and the mons and the labia are covered with fine vellus hair. These findings are consistent with stage of sexual maturity, according to the Sexual Maturity Rating scale. a. 1 b.2 c. 3 d.4

a. 1 Sexual Maturity Rating stage 1 is the preadolescent stage. There is no pubic hair, and the mons and labia are covered with fine, vellus hair as on the abdomen.

The nurse is performing a digital examination of a patient's prostate gland and notices that a normal prostate gland includes which of the following characteristics? Select all that apply. a. 1 cm protrusion into the rectum b. Heart-shaped with a palpable central groove c. Flat shape with no palpable groove d. Boggy with a soft consistency e. Smooth surface, elastic, and rubbery consistency f. Fixed mobility

a. 1 cm protrusion into the rectum b. Heart-shaped with a palpable central groove e. Smooth surface, elastic, and rubbery consistency The size of a normal prostate gland should be 2.5 cm long by 4 cm wide and should not protrude more than 1 cm into the rectum. The prostate should be heart-shaped, with a palpable central groove, a smooth surface, and elastic with a rubbery consistency. Abnormal findings include a flat shape with no palpable groove, boggy with a soft consistency, and fixed mobility.

Of the 33 vertebrae in the spinal column, there are: a. 5 lumbar. b. 5 thoracic. c. 7 sacral. d. 12 cervical.

a. 5 lumbar. There are 7 cervical, 12 thoracic, 5 lumbar, 5 sacral, and 3 to 4 coccygeal vertebrae in the spinal column

The nurse is reviewing a patients medical record and notes that he is in a coma. Using the Glasgow Coma Scale, which number indicates that the patient is in a coma? a. 6 b. 12 c. 15 d. 24

a. 6 A fully alert, normal person has a score of 15, whereas a score of 7 or less reflects coma on the Glasgow Coma Scale

During the ear examination of an 80-year-old patient, which of the following would be a normal finding? a. A high-tone frequency loss b. Increased elasticity of the pinna c. A thin, translucent membrane d. A shiny, pink tympanic membrane

a. A high-tone frequency loss A high-tone frequency hearing loss is apparent for those affected with presbycusis, the hearing loss that occurs with aging.

The uterus is usually positioned tilting forward and superior to the bladder. This position is known as: a. Anteverted and anteflexed. b. Retroverted and anteflexed. c. Retroverted and retroflexed. d. Superiorverted andanteflexed

a. Anteverted and anteflexed. The uterus is freely movable, not fixed, and usually tilts forward and superior to the bladder (a position labeled as anteverted and anteflexed).

The nurse is testing the deep tendon reflexes of a 30-year-old woman who is in the clinic for an annual physical examination. When striking the Achilles heel and quadriceps muscle, the nurse is unable to elicit a reflex. The nurses next response should be to: a. Ask the patient to lock her fingers and pull. b. Complete the examination, and then test these reflexes again. c. Refer the patient to a specialist for further testing. d. Document these reflexes as 0 on a scale of 0 to 4+.

a. Ask the patient to lock her fingers and pull. Sometimes the reflex response fails to appear. Documenting the reflexes as absent is inappropriate this soon in the examination. The nurse should try to further encourage relaxation, varying the persons position or increasing the strength of the blow. Reinforcement is another technique to relax the muscles and enhance the response. The person should be asked to perform an isometric exercise in a muscle group somewhat away from the one being tested. For example, to enhance a patellar reflex, the person should be asked to lock the fingers together and pull.

A 70-year-old man is visiting the clinic for difficulty in passing urine. In the health history, he indicates that he has to urinate frequently, especially at night. He has burning when he urinates and has noticed pain in his back. Considering this history, what might the nurse expect to find during the physical assessment? a. Asymmetric, hard, and fixed prostate gland b. Occult blood and perianal pain to palpation c. Symmetrically enlarged, soft prostate gland d. Soft nodule protruding from the rectal mucosa

a. Asymmetric, hard, and fixed prostate gland Subjective symptoms of carcinoma of the prostate include frequency, nocturia, hematuria, weak stream, hesitancy, pain or burning on urination, and continuous pain in lower back, pelvis, and thighs. Objective symptoms of carcinoma of the prostate include a malignant neoplasm that often starts as a single hard nodule on the posterior surface, producing asymmetry and a change in consistency. As it invades normal tissue, multiple hard nodules appear, or the entire gland feels stone hard and fixed.

A 70-year-old man is visiting the clinic for difficulty in passing urine. In the health history, he indicates that he has to urinate frequently, especially at night. He has burning when he urinates and has noticed pain in his back. Considering this history, what might the nurse expect to find during the physical assessment? a. Asymmetric, hard, and fixed prostate gland b. Occult blood and perianal pain to palpation c. Symmetrically enlarged, soft prostate gland d. Soft nodule protruding from the rectal mucosa

a. Asymmetric, hard, and fixed prostate gland Subjective symptoms of carcinoma of the prostate include frequency, nocturia, hematuria, weak stream, hesitancy, pain or burning on urination, and continuous pain in lower back, pelvis, and thighs. Objective symptoms of carcinoma of the prostate include a malignant neoplasm that often starts as a single hard nodule on the posterior surface, producing asymmetry and a change in consistency. As it invades normal tissue, multiple hard nodules appear, or the entire gland feels stone hard and fixed.

A physician tells the nurse that a patient's vertebra prominens is tender and asks the nurse to reevaluate the area in 1 hour. What area of the body will the nurse assess? a. At the level of the C7 vertebra b. At the level of the T11 vertebra c. At the level of the L5 vertebra d. At the level of the S3 vertebra

a. At the level of the C7 vertebra

During an annual physical examination, a 43-year-old patient states that she does not perform monthly breast self-examinations (BSEs). She tells the nurse that she believes that mammograms do a much better job than I ever could to find a lump. The nurse should explain to her that: a. BSEs may detect lumps that appear between mammograms. b. BSEs are unnecessary until the age of 50 years. c. She is correct mammography is a good replacement for BSE. d. She does not need to perform BSEs as long as a physician checks her breasts annually.

a. BSEs may detect lumps that appear between mammograms. The monthly practice of BSE, along with clinical breast examination and mammograms, are complementary screening measures. Mammography can reveal cancers too small to be detected by the woman or by the most experienced examiner. However, interval lumps may become palpable between mammograms.

The nurse should use which test to check for large amounts of fluid around the patella? a. Ballottement b. Tinel sign c. Phalen test d. McMurray test

a. Ballottement Ballottement of the patella is reliable when large amounts of fluid are present. The Tinel sign and the Phalen test are used to check for carpal tunnel syndrome. The McMurray test is used to test the knee for a torn meniscus

A 43-year-old woman is at the clinic for a routine examination. She reports that she has had a breast lump in her right breast for years. Recently, it has begun to change in consistency and is becoming harder. She reports that 5 years ago her physician evaluated the lump and determined that it was nothing to worry about. The examination validates the presence of a mass in the right upper outer quadrant at 1 oclock, approximately 5 cm from the nipple. It is firm, mobile, and nontender, with borders that are not well defined. The nurse replies: a. Because of the change in consistency of the lump, it should be further evaluated by a physician. b. The changes could be related to your menstrual cycles. Keep track of the changes in the mass each month. c. The lump is probably nothing to worry about because it has been present for years and was determined to be noncancerous 5 years ago. d. Because you are experiencing no pain and the size has not changed, you should continue to monitor the lump and return to the clinic in 3 months.

a. Because of the change in consistency of the lump, it should be further evaluated by a physician. A lump that has been present for years and is not exhibiting changes may not be serious but should still be explored. Any recent change or a new lump should be evaluated. The other responses are not correct.

The nurse is reviewing statistics regarding breast cancer. Which woman, aged 40 years in the United States has the highest risk for developing breast cancer? a. Black b. White c. Asian d. American Indian

a. Black The incidence of breast cancer varies within different cultural groups. White women have a higher incidence of breast cancer than black women starting at age 45 years; but black women have a higher incidence before age 45 years. Asian, Hispanic, and American Indian women have a lower risk for development of breast cancer

During a health history of a patient who complains of chronic constipation, the patient asks the nurse about high-fiber foods. The nurse relates that an example of a high-fiber food would be: a. Broccoli. b. Hamburger. c. Iceberg lettuce. d. Yogurt.

a. Broccoli. High-fiber foods are either soluble type (e.g., beans, prunes, barley, broccoli) or insoluble type (e.g., cereals, wheat germ). The other examples are not considered high-fiber foods.

During a health history of a patient who complains of chronic constipation, the patient asks the nurse about high-fiber foods. The nurse relates that an example of a high-fiber food would be: a. Broccoli. b. Hamburger. c. Iceberg lettuce. d. Yogurt.

a. Broccoli. High-fiber foods are either soluble type (e.g., beans, prunes, barley, broccoli) or insoluble type (e.g., cereals, wheatgerm). The other examples are not considered high-fiber foods.

When performing a genitourinary assessment, the nurse notices that the urethral meatus is ventrally positioned. This finding is: a. Called hypospadias. b. A result of phimosis. c. Probably due to a stricture. d. Often associated with aging.

a. Called hypospadias. Normally, the urethral meatus is positioned just about centrally. Hypospadias is the ventral location of the urethral meatus. The position of the meatus does not change with aging. Phimosis is the inability to retract the foreskin. A stricture is a narrow opening of the meatus.

During a vaginal examination of a 38-year-old woman, the nurse notices that the vulva and vagina are erythematous and edematous with thick, white, curdlike discharge adhering to the vaginal walls. The woman reports intense pruritus and thick white discharge from her vagina. The nurse knows that these history and physical examination findings are most consistent with which condition? a. Candidiasis b. Trichomoniasis c. Atrophic vaginitis d. Bacterial vaginosis

a. Candidiasis The woman with candidiasis often reports intense pruritus and thick white discharge. The vulva and vagina are erythematous and edematous. The discharge is usually thick, white, and curdlike. Infection with trichomoniasis causes a profuse, watery, gray-green, and frothy discharge. Bacterial vaginosis causes a profuse discharge that has a foul, fishy, rotten odor. Atrophic vaginitis may have a mucoid discharge.

The nurse knows that determining whether a person is oriented to his or her surroundings will test the functioning of which structure(s)? a. Cerebrum b. Cerebellum c. CNs d. Medulla oblongata

a. Cerebrum The cerebral cortex is responsible for thought, memory, reasoning, sensation, and voluntary movement. The other structures are not responsible for a persons level of consciousness.

During a speculum inspection of the vagina, the nurse would expect to see what at the end of the vaginal canal? a. Cervix b. Uterus c. Ovaries d. Fallopian tubes

a. Cervix At the end of the canal, the uterine cervix projects into the vagina.

The nurse is performing an assessment on a 7-year-old child who has symptoms of chronic watery eyes, sneezing, and clear nasal drainage. The nurse notices the presence of a transverse line across the bridge of the nose, dark blue shadows below the eyes, and a double crease on the lower eyelids. What does the nurse suspect is the cause of these signs and symptoms? a. Chronic allergies b. Lymphadenopathy c. Nasal congestion d. Upper respiratory infection

a. Chronic allergies

A patient states, I can hear a crunching or grating sound when I kneel. She also states that it is very difficult to get out of bed in the morning because of stiffness and pain in my joints. The nurse should assess for signs of what problem? a. Crepitation b. Bone spur c. Loose tendon d. Fluid in the knee joint

a. Crepitation Crepitation is an audible and palpable crunching or grating that accompanies movement and occurs when articular surfaces in the joints are roughened, as with rheumatoid arthritis. The other options are not correct.

Which of these tests would the nurse use to check the motor coordination of an 11-month-old infant? a. Denver II b. Stereognosis c. Deep tendon reflexes d. Rapid alternating movements

a. Denver II To screen gross and fine motor coordination, the nurse should use the Denver II with its age-specific developmental milestones. Stereognosis tests a persons ability to recognize objects by feeling them and is not appropriate for an 11-month-old infant. Testing the deep tendon reflexes is not appropriate for checking motor coordination. Testing rapid alternating movements is appropriate for testing coordination in adults.

Which of these statements is most appropriate when the nurse is obtaining a genitourinary history from an older man? a. Do you need to get up at night to urinate? b. Do you experience nocturnal emissions, or wet dreams? c. Do you know how to perform a testicular self-examination? d. Has anyone ever touched your genitals when you did not want them to?

a. Do you need to get up at night to urinate? The older male patient should be asked about the presence of nocturia. Awaking at night to urinate may be attributable to a diuretic medication, fluid retention from mild heart failure or varicose veins, or fluid ingestion 3 hours before bedtime, especially coffee and alcohol. The other questions are more appropriate for younger men.

A 59-year-old patient has been diagnosed with prostatitis and is being seen at the clinic for complaints of burning and pain during urination. He is experiencing: a. Dysuria. b. Nocturia. c. Polyuria. d. Hematuria.

a. Dysuria. Dysuria (burning with urination) is common with acute cystitis, prostatitis, and urethritis. Nocturia is voiding during the night. Polyuria is voiding in excessive quantities. Hematuria is voiding with blood in the urine.

When examining children affected with Down syndrome (trisomy 21), what should the nurse look for r/t this disorder? a. Ear dysplasia b. Long, thin neck c. Protruding thin tongue d. Narrow and raised nasal bridge

a. Ear dysplasia

During an examination, which tests will the nurse collect to screen for cervical cancer? a. Endocervical specimen, cervical scrape, and vaginal pool b. Endocervical specimen, vaginal pool, and acetic acid wash c. Endocervical specimen, potassium hydroxide (KOH) preparation, and acetic acid wash d. Cervical scrape, acetic acid wash, saline mount (wet prep)

a. Endocervical specimen, cervical scrape, and vaginal pool Laboratories may vary in method, but usually the test consists of three specimens: endocervical specimen, cervical scrape, and vaginal pool. The other tests (acetic acid wash, KOH preparation, and saline mount) are used to test for sexually transmitted infections.

The nurse is preparing to palpate the rectum and should use which of these techniques? The nurse should: a. Flex the finger, and slowly insert it toward the umbilicus. b. First instruct the patient that this procedure will be painful. c. Insert an extended index finger at a right angle to the anus. d. Place the finger directly into the anus to overcome the tight sphincter.

a. Flex the finger, and slowly insert it toward the umbilicus. The nurse should gently place the pad of the index finger against the anal verge. The nurse will feel the sphincter tighten and then relax. As it relaxes, the nurse should flex the tip of the finger and slowly insert it into the anal canal in a direction toward the umbilicus. The nurse should never approach the anus at right angles with the index finger extended; doing so would cause pain. The nurse should instruct the patient that palpation is not painful but may feel like needing to move the bowels.

The nurse is preparing to palpate the rectum and should use which of these techniques? The nurse should: a. Flex the finger, and slowly insert it toward the umbilicus. b. First instruct the patient that this procedure will be painful. c. Insert an extended index finger at a right angle to the anus. d. Place the finger directly into the anus to overcome the tight sphincter.

a. Flex the finger, and slowly insert it toward the umbilicus. The nurse should gently place the pad of the index finger against the anal verge. The nurse will feel the sphincter tighten and then relax. As it relaxes, the nurse should flex the tip of the finger and slowly insert it into the anal canal in a direction toward the umbilicus. The nurse should never approach the anus at right angles with the index finger extended; doing so would cause pain. The nurse should instruct the patient that palpation is not painful but may feel like needing to move the bowels.

A patient tells the nurse that she is having a hard time bringing her hand to her mouth when she eats or tries to brush her teeth. The nurse knows that for her to move her hand to her mouth, she must perform which movement? a. Flexion b. Abduction c. Adduction d. Extension

a. Flexion Flexion, or bending a limb at a joint, is required to move the hand to the mouth. Extension is straightening a limb at a joint. Moving a limb toward the midline of the body is called adduction; abduction is moving a limb away from the midline of the body.

The nurse is checking the range of motion in a patients knee and knows that the knee is capable of which movement(s)? a. Flexion and extension b. Supination and pronation c. Circumduction d. Inversion and eversion

a. Flexion and extension The knee is a hinge joint, permitting flexion and extension of the lower leg on a single plane. The knee is not capable of the other movements listed.

The wife of a 65-year-old man tells the nurse that she is concerned because she has noticed a change in her husbands personality and ability to understand. He also cries very easily and becomes angry. The nurse recalls that the cerebral lobe responsible for these behaviors is the lobe. a. Frontal b. Parietal c. Occipital d. Temporal

a. Frontal The frontal lobe has areas responsible for personality, behavior, emotions, and intellectual function. The parietal lobe has areas responsible for sensation; the occipital lobe is responsible for visual reception; and the temporal lobe is responsible for hearing, taste, and smell.

To test for gross motor skill and coordination of a 6-year-old child, which of these techniques would be appropriate? Ask the child to: a. Hop on one foot. b. Stand on his head. c. Touch his finger to his nose. d. Make funny faces at the nurse.

a. Hop on one foot. Normally, a child can hop on one foot and can balance on one foot for approximately 5 seconds by 4 years of age and can balance on one foot for 8 to 10 seconds at 5 years of age. Children enjoy performing these tests. Failure to hop after 5 years of age indicates incoordination of gross motor skills. Asking the child to touch his or her finger to the nose checks fine motor coordination; and asking the child to make funny faces tests CN VII. Asking a child to stand on his or her head is not appropriate.

In a person with an upper motor neuron lesion such as a cerebrovascular accident, which of these physical assessment findings should the nurse expect? a. Hyperreflexia b. Fasciculations c. Loss of muscle tone and flaccidity d. Atrophy and wasting of the muscles

a. Hyperreflexia Hyperreflexia, diminished or absent superficial reflexes, and increased muscle tone or spasticity can be expected with upper motor neuron lesions. The other options reflect a lesion of lower motor neurons.

A 59-year-old patient has a herniated intervertebral disk. Which of the following findings should the nurse expect to see on physical assessment of this individual? a. Hyporeflexia b. Increased muscle tone c. Positive Babinski sign d. Presence of pathologic reflexes

a. Hyporeflexia With a herniated intervertebral disk or lower motor neuron lesion, loss of tone, flaccidity, atrophy, fasciculations, and hyporeflexia or areflexia are demonstrated. No Babinski sign or pathologic reflexes would be observed. The other options reflect a lesion of upper motor neurons.

When examining the eye, the nurse notices that the patient's eyelid margins approximate completely. The nurse recognizes that this assessment finding: a. Is expected. b. May indicate a problem with extraocular muscles. c. May result in problems with tearing. d. Indicates increased intraocular pressure.

a. Is expected. The palpebral fissure is the elliptical open space between the eyelids, and, when closed, the lid margins approximate completely, which is a normal finding.

Which of the following statements is true concerning air conduction? a. It is the most efficient pathway for hearing. b. It is caused by the vibrations of bones in the skull. c. The amplitude of sound determines the pitch that is heard. d. A loss of air conduction is called a conductive hearing loss.

a. It is the most efficient pathway for hearing. The normal pathway of hearing is air conduction, and it is the most efficient.

The functional units of the musculoskeletal system are the: a. Joints. b. Bones. d. Muscles. d. Tendons.

a. Joints. Joints are the functional units of the musculoskeletal system because they permit the mobility needed to perform the activities of daily living. The skeleton (bones) is the framework of the body. The other options are not correct.

A woman who is 8 months pregnant comments that she has noticed a change in her posture and is having lower back pain. The nurse tells her that during pregnancy, women have a posture shift to compensate for the enlarging fetus. This shift in posture is known as: a. Lordosis. b. Scoliosis. c. Ankylosis. d. Kyphosis.

a. Lordosis. Lordosis compensates for the enlarging fetus, which would shift the center of balance forward. This shift in balance, in turn, creates a strain on the low back muscles, felt as low back pain during late pregnancy by some women. Scoliosis is lateral curvature of portions of the spine; ankylosis is extreme flexion of the wrist, as observed with severe rheumatoid arthritis; and kyphosis is an enhanced thoracic curvature of the spine.

Decreased vision in an older patient may be due to which of the following conditions? a. Macular degeneration b. Presbyopia c. Fixation d. Retinoblastoma

a. Macular degeneration Decreased vision in older patients is most commonly caused by cataracts, glaucoma, or macular degeneration. Retinoblastoma is a malignant tumor of the retina that usually affects children younger than 6 years old. Fixation is a reflex direction of the eye toward an object attracting a person's attention, fixation is impaired by drugs, alcohol, fatigue, and inattention. The lens in an older adult loses elasticity and becomes hard and glasslike, this decreases the lens' ability to change shape to accommodate for near vision and is called presbyopia.

A patient is newly diagnosed with benign breast disease. The nurse recognizes which statement about benign breast disease to be true? The presence of benign breast disease: a. Makes it hard to examine the breasts. b. Frequently turns into cancer in a womans later years. c. Is easily reduced with hormone replacement therapy. d. Is usually diagnosed before a woman reaches childbearing age.

a. Makes it hard to examine the breasts. The presence of benign breast disease (formerly fibrocystic breast disease) makes it hard to examine the breasts; the general lumpiness of the breast conceals a new lump. The other statements are not true.

A new mother calls the clinic to report that part of her left breast is red, swollen, tender, very hot, and hard. She has a fever of 38.3 C. She also has had symptoms of influenza, such as chills, sweating, and feeling tired. The nurse notices that she has been breastfeeding for 1 month. From her description, what condition does the nurse suspect? a. Mastitis b. Paget disease c. Plugged milk duct d. Mammary ductectasia

a. Mastitis The symptoms describe mastitis, which stems from an infection or stasis caused by a plugged duct. A plugged duct does not have infection present.

A woman is in the clinic for an annual gynecologic examination. The nurse should plan to begin the interview with the: a. Menstrual history, because it is generally nonthreatening. b. Obstetric history, because it includes the most important information. c. Urinary system history, because problems may develop in this area as well. d. Sexual history, because discussing it first will build rapport.

a. Menstrual history, because it is generally nonthreatening. Menstrual history is usually nonthreatening and therefore a good topic with which to begin the interview. Obstetric, urinary, and sexual histories are also part of the interview but not necessarily the best topics with which to start.

The nurse is discussing BSEs with a postmenopausal woman. The best time for postmenopausal women to perform BSEs is: a. On the same day every month. b. Daily, during the shower or bath. c. One week after her menstrual period. d. Every year with her annual gynecologic examination.

a. On the same day every month. Postmenopausal women are no longer experiencing regular menstrual cycles but need to continue to perform BSEs on a monthly basis. Choosing the same day of the month is a helpful reminder to perform the examination.

During an assessment of a 62-year-old man, the nurse notices the patient has a stooped posture, shuffling walk with short steps, flat facial expression, and pill-rolling finger movements. These findings would be consistent with: a. Parkinsonism. b. Cerebral palsy. c. Cerebellar ataxia. d. Muscular dystrophy.

a. Parkinsonism. The stooped posture, shuffling walk, short steps, flat facial expression, and pill-rolling finger movements are all found in parkinsonism

During a physical examination, the nurse finds that a male patients foreskin is fixed and tight and will not retract over the glans. The nurse recognizes that this condition is: a. Phimosis. b. Epispadias. c. Urethral stricture. d. Peyronie disease.

a. Phimosis. With phimosis, the foreskin is nonretractable, forming a pointy tip of the penis with a tiny orifice at the end of the glans. The foreskin is advanced and so tight that it is impossible to retract over the glans. This condition may be congenital or acquired from adhesions related to infection.

The mother of a 5-year-old girl tells the nurse that she has noticed her daughter "scratching at her bottom a lot the last few days." During the assessment, the nurse finds redness and raised skin in the anal area. This finding most likely indicates: a. Pinworms. b. Chickenpox. c. Constipation. d. Bacterial infection.

a. Pinworms. In children, pinworms are a common cause of intense itching and irritated anal skin.

The mother of a 5-year-old girl tells the nurse that she has noticed her daughter scratching at her bottom a lot the last few days. During the assessment, the nurse finds redness and raised skin in the anal area. This finding most likely indicates: a. Pinworms. b. Chickenpox. c. Constipation. d. Bacterial infection.

a. Pinworms. In children, pinworms are a common cause of intense itching and irritated anal skin. The other options are not correct.

A 62-year-old man is experiencing fever, chills, malaise, urinary frequency, and urgency. He also reports urethral discharge and a dull aching pain in the perineal and rectal area. These symptoms are most consistent with which condition? a. Prostatitis b. Polyps c. Carcinoma of the prostate d. BPH

a. Prostatitis The common presenting symptoms of prostatitis are fever, chills, malaise, and urinary frequency and urgency. The individual may also have dysuria, urethral discharge, and a dull aching pain in the perineal and rectal area. These symptoms are not consistent with polyps.

A 62-year-old man is experiencing fever, chills, malaise, urinary frequency, and urgency. He also reports urethral discharge and a dull aching pain in the perineal and rectal area. These symptoms are most consistent with which condition? a. Prostatitis b. Polyps c. Carcinoma of the prostate d. BPH

a. Prostatitis The common presenting symptoms of prostatitis are fever, chills, malaise, and urinary frequency and urgency. The individual may also have dysuria, urethral discharge, and a dull aching pain in the perineal and rectal area. These symptoms are not consistent with polyps.

When performing a musculoskeletal assessment, the nurse knows that the correct approach for the examination should be: a. Proximal todistal b. Distal to proximal. c. Posterior to anterior. d. Anterior to posterior.

a. Proximal todistal The musculoskeletal assessment should be performed in an orderly approach, head to toe, proximal to distal, from the midline outward. The other options are not correct.

The nurse is preparing to do an otoscopic examination on a 2-year-old child. Which of the following reflects correct procedure? a. Pull the pinna down. b. Pull the pinna up and back. c. Tilt the child's head slightly toward the examiner. d. Have the child touch his chin to his chest.

a. Pull the pinna down. Pull the pinna down on an infant and a child under 3 years of age.

To assess the head control of a 4-month-old infant, the nurse lifts up the infant in a prone position while supporting his chest. The nurse looks for what normal response? The infant: a. Raises the head, and arches the back. b. Extends the arms, and drops down the head. c. Flexes the knees and elbows with the back straight. d. Holds the head at 45 degrees, and keeps the back straight.

a. Raises the head, and arches the back. At 3 months of age, the infant raises the head and arches the back as if in a swan dive. This response is the Landau reflex, which persists until 1 years of age. The other responses are incorrect.

In the assessment of a 1-month-old infant, the nurse notices a lack of response to noise or stimulation. The mother reports that in the last week he has been sleeping all of the time, and when he is awake all he does is cry. The nurse hears that the infants cries are very high pitched and shrill. What should be the nurses appropriate response to these findings? a. Refer the infant for further testing. b. Talk with the mother about eating habits. c. Do nothing; these are expected findings for an infant this age. d. Tell the mother to bring the baby back in 1 week for a recheck.

a. Refer the infant for further testing. A high-pitched, shrill cry or cat-sounding screech occurs with central nervous system damage. Lethargy, hyporeactivity, and hyperirritability, as well as the parents report of significant changes in behavior all warrant referral. The other options are not correct responses.

While obtaining a health history of a 3-month-old infant from the mother, the nurse asks about the infants ability to suck and grasp the mothers finger. What is the nurse assessing? a. Reflexes b. Intelligence c. CNs d. Cerebral cortex function

a. Reflexes Questions regarding reflexes include such questions as, What have you noticed about the infants behavior, Are the infants sucking and swallowing seem coordinated, and Does the infant grasp your finger? The other responses are incorrect.

A 21-year-old patient has a head injury resulting from trauma and is unconscious. There are no other injuries. During the assessment what would the nurse expect to find when testing the patients deep tendon reflexes? a. Reflexes will be normal. b. Reflexes cannot be elicited. c. All reflexes will be diminished but present. d. Some reflexes will be present, depending on the area of injury.

a. Reflexes will be normal. A reflex is a defense mechanism of the nervous system. It operates below the level of conscious control and permits a quick reaction to potentially painful or damaging situations.

A woman has just learned that she is pregnant. What are some things the nurse should teach her about her breasts? a. She can expect her areolae to become larger and darker in color. b. Breasts may begin secreting milk after the fourth month of pregnancy. c. She should inspect her breasts for visible veins and immediately report these. d. During pregnancy, breast changes are fairly uncommon; most of the changes occur after the birth

a. She can expect her areolae to become larger and darker in color. The areolae become larger and grow a darker brown as pregnancy progresses, and the tubercles become more prominent. (The brown color fades after lactation, but the areolae never return to their original color). A venous pattern is an expected finding and prominent over the skin surface and does not need to be reported. After the fourth month of pregnancy, colostrum, a thick, yellow fluid (precursor to milk), may be expressed from the breasts.

An older man is concerned about his sexual performance. The nurse knows that in the absence of disease, a withdrawal from sexual activity later in life may be attributable to: a. Side effects of medications. b. Decreased libido with aging. c. Decreased sperm production. d. Decreased pleasure from sexual intercourse.

a. Side effects of medications. In the absence of disease, a withdrawal from sexual activity may be attributable to side effects of medications such as antihypertensives, antidepressants, sedatives, psychotropics, antispasmotics, tranquilizers or narcotics, and estrogens. The other options are not correct.

The nurse is inspecting the scrotum and testes of a 43-year-old man. Which finding would require additional follow-up and evaluation? a. Skin on the scrotum is taut. b. Left testicle hangs lower than the right testicle. c. Scrotal skin has yellowish 1-cm nodules that are firm and nontender. d. Testes move closer to the body in response to cold temperatures.

a. Skin on the scrotum is taut. Scrotal swelling may cause the skin to be taut and to display pitting edema. Normal scrotal skin is rugae, and asymmetry is normal with the left scrotal half usually lower than the right. The testes may move closer to the body in response to cold temperatures.

When inspecting the eyeballs of an African American individual, which of the following might the examiner expect to observe? a. Small brown macules on the sclera b. A slight amount of drainage around the lacrimal apparatus c. A slight yellow discoloration of the sclera d. A slight misalignment of the eyeballs

a. Small brown macules on the sclera Dark-skinned people may normally have small brown macules on the sclera.

When examining a patient's CN function, what muscles should the nurse assess to assess the function of CN XI? a. Sternomastoid and trapezius b. Spinal accessory and omohyoid c. Trapezius and sternomandibular d. Sternomandibular and spinal accessory

a. Sternomastoid and trapezius

The nurse is caring for a newborn infant. Thirty hours after birth, the infant passes a dark green meconium stool. The nurse recognizes this is important because the: a. Stool indicates anal patency. b. Dark green color indicates occult blood in the stool. c. Meconium stool can be reflective of distress in the newborn. d. Newborn should have passed the first stool within 12 hours after birth.

a. Stool indicates anal patency. The first stool passed by the newborn is dark green meconium and occurs within 24 to 48 hours of birth, indicating anal patency.

The nurse is caring for a newborn infant. Thirty hours after birth, the infant passes a dark green meconium stool. The nurse recognizes this is important because the: a. Stool indicates anal patency. b. Dark green color indicates occult blood in the stool. c. Meconium stool can be reflective of distress in the newborn. d. Newborn should have passed the first stool within 12 hours after birth.

a. Stool indicates anal patency. The first stool passed by the newborn is dark green meconium and occurs within 24 to 48 hours of birth, indicating anal patency. The other responses are not correct.

The nurse is palpating a female patients breasts during an examination. Which of these positions is most likely to make significant lumps more distinct during breast palpation? a. Supine with the arms raised over her head b. Sitting with the arms relaxed at her sides c. Supine with the arms relaxed at her sides d. Sitting with the arms flexed and fingertips touching her shoulders

a. Supine with the arms raised over her head The nurse should help the woman to a supine position, tuck a small pad under the side to be palpated, and help the woman raise her arm over her head. These maneuvers will flatten the breast tissue and medially displace it. Any significant lumps will then feel more distinct.

The ankle joint is the articulation of the tibia, fibula, and: a. Talus. b. Cuboid. b. Calcaneus. d. Cuneiform bones.

a. Talus. The ankle or tibiotalar joint is the articulation of the tibia, fibula, and talus. The other bones listed are foot bones and not part of the ankle joint.

A male patient with possible fertility problems asks the nurse where sperm is produced. The nurse knows that sperm production occurs in the: a. Testes. b. Prostate. c. Epididymis. d. Vas deferens.

a. Testes. Sperm production occurs in the testes, not in the other structures listed

A 14-year-old girl is anxious about not having reached menarche. When taking the health history, the nurse should ascertain which of the following? The age that: a. The girl began to develop breasts. b. Her mother developed breasts. c. She began to develop pubic hair. d. She began to develop axillary hair.

a. The girl began to develop breasts. Full development from stage 2 to stage 5 takes an average of 3 years, although the range is 1 to 6 years. Pubic hair develops during this time, and axillary hair appears 2 years after the onset of pubic hair. The beginning of breast development precedes menarche by approximately 2 years. Menarche occurs in breast development stage 3 or 4, usually just after the peak of the adolescent growth spurt, which occurs around age 12 years.

The nurse assesses the hearing of a 7-month-old by clapping hands. What is the expected response? a. The infant turns the head to localize sound. b. No obvious response to noise c. A startle and acoustic blink reflex d. The infant stops movement and appears to listen.

a. The infant turns the head to localize sound. With a loud sudden noise, you should note these responses: 6 to 8 months—infant turns head to localize sound, responds to own name.

The nurse is preparing to interview a postmenopausal woman. Which of these statements is true as it applies to obtaining the health history of a postmenopausal woman? a. The nurse should ask a postmenopausal woman if she has ever had vaginal bleeding. b. Once a woman reaches menopause, the nurse does not need to ask any history questions. c. The nurse should screen for monthly breast tenderness d. Postmenopausal women are not at risk for contracting STIs; therefore, these questions can be omitted.

a. The nurse should ask a postmenopausal woman if she has ever had vaginal bleeding. Postmenopausal bleeding warrants further workup and referral. The other statements are not true.

The nurse is performing an external eye examination. Which statement regarding the outer layer of the eye is true? a. The outer layer of the eye is very sensitive to touch. b. The outer layer of the eye is darkly pigmented to prevent light from reflecting internally. c. The trigeminal nerve (CN V) and the trochlear nerve (CN IV) are stimulated when the outer surface of the eye is stimulated. d. The visual receptive layer of the eye in which light waves are changed into nerve impulses is located in the outer layer of the eye.

a. The outer layer of the eye is very sensitive to touch. The cornea and the sclera make up the outer layer of the eye. The cornea is very sensitive to touch. The middle layer, the choroid, has dark pigmentation to prevent light from reflecting internally. The trigeminal nerve (CN V) and the facial nerve (CN VII) are stimulated when the outer surface of the eye is stimulated. The retina, in the inner layer of the eye, is where light waves are changed into nerve impulses.

During the assessment of an 80-year-old patient, the nurse notices that his hands show tremors when he reaches for something and his head is always nodding. No associated rigidity is observed with movement. Which of these statements is most accurate? a. These findings are normal, resulting from aging. b. These findings could be related to hyperthyroidism. c. These findings are the result of Parkinson disease. d. This patient should be evaluated for a cerebellar lesion.

a. These findings are normal, resulting from aging. Senile tremors occasionally occur. These benign tremors include an intention tremor of the hands, head nodding (as if saying yes or no), and tongue protrusion. Tremors associated with Parkinson disease include rigidity, slowness, and a weakness of voluntary movement. The other responses are incorrect.

The nurse is examining a 6-month-old infant and places the infants feet flat on the table and flexes his knees up. The nurse notes that the right knee is significantly lower than the left. Which of these statements is true of this finding? a. This finding is a positive Allis sign and suggests hip dislocation. b. The infant probably has a dislocated patella on the right knee. c. This finding is a negative Allis sign and normal for an infant of this age. d. The infant should return to the clinic in 2 weeks to see if his condition has changed.

a. This finding is a positive Allis sign and suggests hip dislocation. Finding one knee significantly lower than the other is a positive Allis sign and suggests hip dislocation. Normally, the tops of the knees are at the same elevation. The other statements are not correct.

When assessing a newborn infants genitalia, the nurse notices that the genitalia are somewhat engorged. The labia majora are swollen, the clitoris looks large, and the hymen is thick. The vaginal opening is difficult to visualize. The infants mother states that she is worried about the labia being swollen. The nurse should reply: a. This is a normal finding in newborns and should resolve within a few weeks. b. This finding could indicate an abnormality and may need to be evaluated by a physician. c. We will need to have estrogen levels evaluated to ensure that they are within normal limits. d. We will need to keep close watch over the next few days to see if the genitalia decrease in size.

a. This is a normal finding in newborns and should resolve within a few weeks. It is normal for a newborns genitalia to be somewhat engorged. A sanguineous vaginal discharge or leukorrhea is normal during the first few weeks because of the maternal estrogen effect. During the early weeks, the genital engorgement resolves, and the labia minora atrophy and remain small until puberty.

A colleague is assessing an 80-year-old patient who has ear pain and asks him to hold his nose and swallow. The nurse knows that which of the following is true concerning this technique? a. This should not be used in an 80-year-old patient. b. This technique is helpful in assessing for otitis media. c. This is especially useful in assessing a patient with an upper respiratory infection. d. This will cause the eardrum to bulge slightly and make landmarks more visible.

a. This should not be used in an 80-year-old patient. The eardrum is flat, slightly pulled in at the center, and flutters when the person performs the Valsalva maneuver or holds the nose and swallows (insufflation). One may elicit these maneuvers to assess drum mobility. Avoid these with an aging person because they may disrupt equilibrium.

During an examination, the nurse notes a supernumerary nipple just under the patients left breast. The patient tells the nurse that she always thought it was a mole. Which statement about this finding is correct? a. This variation is normal and not a significant finding. b. This finding is significant and needs further investigation. c. A supernumerary nipple also contains glandular tissue and may leak milk during pregnancy and lactation. d. The patient is correct a supernumerary nipple is actually a mole that happens to be located under the breast.

a. This variation is normal and not a significant finding. A supernumerary nipple looks like a mole, but close examination reveals a tiny nipple and areola; it is not a significant finding.

A patient's laboratory data reveal an elevated thyroxine (T4) level. What gland should the nurse assess?' a. Thyroid b. Parotid c. Adrenal d. Parathyroid

a. Thyroid THINK: T fo(u)r Thyroid

During an examination, the nurse observes a female patient vestibule and expects to see the: a. Urethral meatus and vaginal orifice. b. Vaginal orifice and vestibular (Bartholin) glands. c. Urethral meatus and paraurethral (Skene) glands. d. Paraurethral (Skene) and vestibular (Bartholin) glands.

a. Urethral meatus and vaginal orifice. The labial structures encircle a boat-shaped space, or cleft, termed the vestibule. Within the vestibule are numerous openings. The urethral meatus and vaginal orifice are visible. The ducts of the paraurethral (Skene) glands and the vestibular (Bartholin) glands are present but not visible.

An 11-year-old girl is in the clinic for a sports physical examination. The nurse notices that she has begun to develop breasts, and during the conversation the girl reveals that she is worried about her development. The nurse should use which of these techniques to best assist the young girl in understanding the expected sequence for development? The nurse should: a. Use the Tanner scale on the five stages of sexual development. b. Describe her development and compare it with that of other girls her age. c. Use the Jacobsen table on expected development on the basis of height and weight data. d. Reassure her that her development is within normal limits and tell her not to worry about the next step.

a. Use the Tanner scale on the five stages of sexual development. The Tanner scale on the five stages of pubic hair development is helpful in teaching girls the expected sequence of sexual development. The other responses are not appropriate.

During an examination, the nurse knows that the best way to palpate the lymph nodes in the neck is described by which statement? a. Using gentle pressure, palpate with both hands to compare the two sides. b. Using strong pressure, palpate with both hands to compare the two sides. c. Gently pinch each node between one's thumb and forefinger, and then move down the neck muscle. d. Using the index and middle fingers, gently palpate by applying pressure in a rotating pattern.

a. Using gentle pressure, palpate with both hands to compare the two sides.

The nurse is reviewing the changes that occur with menopause. Which changes are associated with menopause? a. Uterine and ovarian atrophy, along with a thinning of the vaginal epithelium b. Ovarian atrophy, increased vaginal secretions, and increasing clitoral size c. Cervical hypertrophy, ovarian atrophy, and increased acidity of vaginal secretions d. Vaginal mucosa fragility, increased acidity of vaginal secretions, and uterine hypertrophy

a. Uterine and ovarian atrophy, along with a thinning of the vaginal epithelium The uterus shrinks because of its decreased myometrium. The ovaries atrophy to 1 to 2 cm and are not palpable after menopause. The sacral ligaments relax, and the pelvic musculature weakens; consequently, the uterus droops. The cervix shrinks and looks paler with a thick glistening epithelium. The vaginal epithelium atrophies, becoming thinner, drier, and itchy. The vaginal pH becomes more alkaline, and secretions are decreased, which results in a fragile mucosal surface that is at risk for vaginitis.

During the taking of the health history, a patient tells the nurse that it feels like the room is spinning around me. The nurse would document this finding as: a. Vertigo. b. Syncope. c. Dizziness. d. Seizure activity

a. Vertigo. True vertigo is rotational spinning caused by a neurologic dysfunction or a problem in the vestibular apparatus or the vestibular nuclei in the brainstem. Syncope is a sudden loss of strength or a temporary loss of consciousness. Dizziness is a lightheaded, swimming sensation. Seizure activity is characterized by altered or loss of consciousness, involuntary muscle movements, and sensory disturbances.

The nurse is assisting with a BSE clinic. Which of these women reflect abnormal findings during the inspection phase of breast examination? a. Woman whose nipples are in different planes (deviated). b. Woman whose left breast is slightly larger than her right. c. Nonpregnant woman whose skin is marked with linear striae. d. Pregnant woman whose breasts have a fine blue network of veins visible under the skin.

a. Woman whose nipples are in different planes (deviated). The nipples should be symmetrically placed on the same plane on the two breasts. With deviation in pointing, an underlying cancer may cause fibrosis in the mammary ducts, which pulls the nipple angle toward it. The other examples are normal findings.

The external structure of the ear is identified as the a. auricle. b. auriga. c. atrium. d. aureole.

a. auricle. The auricle or pinna is the external structure of the ear. The atrium is the upper chamber of the heart.

The portion of the ear that consists of movable cartilage and skin is called the: a. auricle. b. concha. c. outer meatus. d. mastoid process.

a. auricle. The external ear is called the auricle or pinna and consists of movable cartilage and skin.

Kyphosis of the spine is common with aging. To compensate, older adults a. extend their heads and jaws forward. b. shuffle. c. stiffen their gait. d. increase their center of gravity.

a. extend their heads and jaws forward. An older adult may show an increased anterior cervical (concave or inward) curve when the head and jaw are extended forward to compensate for kyphosis of the spine.

A 31-year-old patient tells the nurse that he has noticed a progressive loss in his hearing. He says that it does seem to help when people speak louder or if he turns up the volume. The most likely cause of his hearing loss is: a. otosclerosis. b. presbycusis. c. trauma to the bones. d. frequent ear infections.

a. otosclerosis. Otosclerosis is a common cause of conductive hearing loss in young adults between the ages of 20 and 40 years.

The bulge sign is a test for: a. swelling in the suprapatellar pouch b. kyphosis c. carpal runnel syndrome d. scoliosis

a. swelling in the suprapatellar pouch

When performing an otoscopic examination of a 5-year-old child with a history of chronic ear infections, the nurse sees that his right tympanic membrane is amber- yellow in color and there are air bubbles behind the tympanic membrane. The child reports occasional hearing loss and a popping sound with swallowing. The preliminary analysis based on this information is that: a. this is most likely a serous otitis media. b. the child has an acute purulent otitis media. c. there is evidence of a resolving cholesteatoma. d. the child is experiencing the early stages of perforation.

a. this is most likely a serous otitis media. An amber-yellow color to the tympanic membrane suggests serum in the middle ear. Often an air/fluid level or bubbles behind the tympanic membrane are visible. The patient may have feelings of fullness, transient hearing loss, and a popping sound with swallowing.

scrotal hernia

an inguinal hernia that has descended into the scrotum

An 80-year-old woman is visiting the clinic for a checkup. She states, I cant walk as much as I used to. The nurse is observing for motor dysfunction in her hip and should ask her to: a. Internally rotate her hip while she is sitting. b. Abduct her hip while she is lying on her back. c. Adduct her hip while she is lying on her back. d. Externally rotate her hip while she is standing.

b. Abduct her hip while she is lying on her back. Limited abduction of the hip while supine is the most common motion dysfunction found in hip disease. The other options are not correct.

During an interview the patient states, I can feel this bump on the top of both of my shoulders it doesnt hurt but I am curious about what it might be. The nurse should tell the patient that it is his: a. Subacromial bursa. b. Acromion process. c. Glenohumeral joint. d. Greater tubercle of the humerus.

b. Acromion process. The bump of the scapulas acromion process is felt at the very top of the shoulder. The other options are not correct.

A 40-year-old man has come into the clinic with complaints of extreme pain in his toes. The nurse notices that his toes are slightly swollen, reddened, and warm to the touch. His complaints would suggest: a. Osteoporosis. b. Acute gout. c. Ankylosing spondylitis. d. Degenerative joint disease.

b. Acute gout. Clinical findings for acute gout consist of redness, swelling, heat, and extreme pain like a continuous throbbing. Gout is a metabolic disorder of disturbed purine metabolism, associated with elevated serum uric

Which of the following statements r/t aggravating symptoms or triggers of headaches is true? a. Poor posture may trigger a migraine headache. b. Alcohol consumption may precipitate the onset of cluster or migraine headaches. c. Premenstrual hormonal fluctuations may precipitate the onset of cluster headaches. d. Certain foods such as chocolate or cheese may precipitate the onset of tension headaches.

b. Alcohol consumption may precipitate the onset of cluster or migraine headaches. Aggravating symptoms or triggers for cluster headaches include alcohol consumption, stress, or wind or heat exposure. Aggravating symptoms or triggers for migraines include hormonal fluctuations, certain foods, letdown after stress, changes in sleep pattern, sensory stimuli, and changes in weather or physical activity. Aggravating symptoms or triggers for tension headaches include stress anxiety, depression, and poor posture.

Which of the following children is at risk for recurrent otitis media (OM)? a. A 2 year old child who has had two ear infections in the past 6 months b. An 18 month old infant who has had three episodes of ear infections in a 5 month period c. An 18 month old infant who lives with a smoker d. A 6 month old infant who has a sibling who had tubes inserted at 3 years of age

b. An 18 month old infant who has had three episodes of ear infections in a 5 month period A first episode of OM that occurs within 3 months of life increases risk for recurrent OM. Recurrent OM is three episodes within the past 3 months or four episodes within the past year.

During an examination of a 7-year-old girl, the nurse notices that the girl is showing breast budding. What should the nurse do next? a. Ask the young girl if her periods have started. b. Assess the girls weight and body mass index (BMI). c. Ask the girls mother at what age she started to develop breasts. d. Nothing; breast budding is a normal finding

b. Assess the girls weight and body mass index (BMI). Research has shown that girls with overweight or obese BMI levels have a higher occurrence of early onset of breast budding (before age 8 years for black girls and age 10 years for white girls) and early menarche.

The nurse places a key in the hand of a patient and he identifies it as a penny. What term would the nurse use to describe this finding? a. Extinction b. Astereognosis c. Graphesthesia d. Tactile discrimination

b. Astereognosis Stereognosis is the persons ability to recognize objects by feeling their forms, sizes, and weights. Astereognosis is an inability to identify objects correctly, and it occurs in sensory cortex lesions. Tactile discrimination tests fine touch. Extinction tests the persons ability to feel sensations on both sides of the body at the same point.

A patient's thyroid gland is enlarged, and the nurse is preparing to auscultate the thyroid gland for the presence of a bruit. What technique should the nurse use to assess for a bruit? a. Palpate the thyroid while the patient is swallowing. b. Auscultate the thyroid with the bell of the stethoscope. c. Palpate the thyroid while the patient holds their breath. d. Auscultate the thyroid with the diaphragm of the stethoscope.

b. Auscultate the thyroid with the bell of the stethoscope. THINK: Bell is for Bruits

A patient calls the clinic for instructions before having a Papanicolaou (Pap) smear. The most appropriate instructions from the nurse are: a. If you are menstruating, please use pads to avoid placing anything into the vagina. b. Avoid intercourse, inserting anything into the vagina, or douching within 24 hours of your appointment. c. If you suspect that you have a vaginal infection, please gather a sample of the discharge to bring with you. d. We would like you to use a mild saline douche before your examination. You may pick this up in our office.

b. Avoid intercourse, inserting anything into the vagina, or douching within 24 hours of your appointment. When instructing a patient before Pap smear is obtained, the nurse should follow these guidelines: Do not obtain during the womans menses or if a heavy infectious discharge is present. Instruct the woman not to douche, have intercourse, or put anything into the vagina within 24 hours before collecting the specimens. Any specimens will be obtained during the visit, not beforehand.

During a discussion for a men's health group, the nurse relates that the group with the highest incidence of prostate cancer is: a. Asian Americans. b. Blacks. c. American Indians. d. Hispanics.

b. Blacks. According to the American Cancer Society (2010), black men have a higher rate of prostate cancer than other racial groups.

During a discussion for a mens health group, the nurse relates that the group with the highest incidence of prostate cancer is: a. Asian Americans. b. Blacks. c. American Indians. d. Hispanics.

b. Blacks. According to the American Cancer Society (2010), black men have a higher rate of prostate cancer than other racial groups.

The nurse is assessing a 16-year-old patient with head injuries from a recent motor vehicle accident. Which of the following statements indicates the most important reason for assessing for any drainage from the canal? a. If the drum has ruptured, there will be purulent drainage. b. Bloody or clear watery drainage can indicate a basal skull fracture. c. The auditory canal many be occluded from increased cerumen. d. There may be occlusion of the canal caused by foreign bodies from the accident.

b. Bloody or clear watery drainage can indicate a basal skull fracture. Frank blood or clear watery drainage (cerebrospinal leak) after trauma suggests a basal skull fracture and warrants immediate referral.

During an interview, a patient reveals that she is pregnant. She states that she is not sure whether she will breastfeed her baby and asks for some information about this. Which of these statements by the nurse is accurate? a. Breastfed babies tend to be more colicky. b. Breastfeeding provides the perfect food and antibodies for your baby. c. Breastfed babies eat more often than infants on formula. d. Breastfeeding is second nature, and every woman can do it.

b. Breastfeeding provides the perfect food and antibodies for your baby. Exclusively breastfeeding for 6 months provides the perfect food and antibodies for the baby, decreases the risk of ear infections, promotes bonding, and provides relaxation.

The two parts of the nervous system are the: a. Motor and sensory. b. Central and peripheral. c. Peripheral and autonomic. d. Hypothalamus and cerebral.

b. Central and peripheral. The nervous system can be divided into two parts central and peripheral. The central nervous system includes the brain and spinal cord. The peripheral nervous system includes the 12 pairs of cranial nerves (CNs), the 31 pairs of spinal nerves, and all of their branches.

A woman has just been diagnosed with HPV or genital warts. The nurse should counsel her to receive regular examinations because this virus makes her at a higher risk for cancer. a. Uterine b. Cervical c. Ovarian d. Endometrial

b. Cervical HPV is the virus responsible for most cases of cervical cancer, not the other options.

A patient reports excruciating headache pain on one side of his head, especially around his eye, forehead, and cheek that has lasted approximately to 2 hours, occurring once or twice each day. What should the nurse suspect?a. Hypertension b. Cluster headaches c. Tension headaches d. Migraine headaches

b. Cluster headaches NOTE: See Table 14.1 for comparisons of headaches

After completing an assessment of a 60-year-old man with a family history of colon cancer, the nurse discusses with him early detection measures for colon cancer. The nurse should mention the need for a(n): a. Annual proctoscopy. b. Colonoscopy every 10 years. c. Fecal test for blood every 6 months. d. DREs every 2 years.

b. Colonoscopy every 10 years. NOTE: Beginning at 50 years old (45 for black men) early detection for colon cancer includes: - DRE - 1x per year - fecal occult blood test - 1x per year - PSA blood test - 1x per year - sigmoidoscopic examination every 5 years or colonoscopy every 10 years

After completing an assessment of a 60-year-old man with a family history of colon cancer, the nurse discusses with him early detection measures for colon cancer. The nurse should mention the need for a(n): a. Annual proctoscopy. b. Colonoscopy every 10 years. c. Fecal test for blood every 6 months. d. DREs every 2 years.

b. Colonoscopy every 10 years. Early detection measures for colon cancer include a DRE performed annually after age 50 years, an annual fecal occult blood test after age 50 years, a sigmoidoscopic examination every 5 years or a colonoscopy every 10 years after age 50 years, and a PSA blood test annually for men over 50 years old, except beginning at age 45 years for black men (American Cancer Society, 2006).

While inspecting a patients breasts, the nurse finds that the left breast is slightly larger than the right with the bilateral presence of Montgomery glands. The nurse should: a. Palpate over the Montgomery glands, checking for drainage. b. Consider these findings as normal, and proceed with the examination. c. Ask extensive health history questions regarding the womans breast asymmetry. d. Continue with the examination, and then refer the patient for further evaluation of the Montgomery glands.

b. Consider these findings as normal, and proceed with the examination. Normal findings of the breast include one breast (most often the left) slightly larger than the other and the presence of Montgomery glands across the areola.

When performing a genital examination on a 25-year-old man, the nurse notices deeply pigmented, wrinkled scrotal skin with large sebaceous follicles. On the basis of this information, the nurse would: a. Squeeze the glans to check for the presence of discharge. b. Consider this finding as normal, and proceed with the examination. c. Assess the testicles for the presence of masses or painless lumps. d. Obtain a more detailed history, focusing on any scrotal abnormalities the patient has noticed.

b. Consider this finding as normal, and proceed with the examination. After adolescence, the scrotal skin is deeply pigmented and large sebaceous follicles and appears corrugated.

The ability that humans have to perform very skilled movements such as writing is controlled by the: a. Basal ganglia. b. Corticospinal tract. c. Spinothalamic tract. d. Extrapyramidal tract.

b. Corticospinal tract. Corticospinal fibers mediate voluntary movement, particularly very skilled, discrete, and purposeful movements, such as writing. The corticospinal tract, also known as the pyramidal tract, is a newer, higher motor system that humans have that permits very skilled and purposeful movements. The other responses are not related to skilled movements.

During an examination of an aging man, the nurse recognizes that normal changes to expect would be: a. Change in scrotal color. b. Decrease in the size of the penis. c. Enlargement of the testes and scrotum. d. Increase in the number of rugae over the scrotal sac.

b. Decrease in the size of the penis. When assessing the genitals of an older man, the nurse may notice thinner, graying pubic hair and a decrease in the size of the penis. The size of the testes may be decreased, they may feel less firm, and the scrotal sac is pendulous with less rugae. No change in scrotal color is observed.

During an assessment of an infant, the nurse notes that the fontanels are depressed and sunken. The nurse suspects which condition? a. Rickets b. Dehydration c. Mental retardation d. Increased intracranial pressure

b. Dehydration THINK: there's not enough fluid so everything shrinks in on itself.

A nurse is assessing a patients risk of contracting a sexually transmitted infection (STI). An appropriate question to ask would be: a. You know that its important to use condoms for protection, right? b. Do you use a condom with each episode of sexual intercourse? c. Do you have a sexually transmitted infection? d. You are aware of the dangers of unprotected sex, arent you?

b. Do you use a condom with each episode of sexual intercourse? In reviewing a patients risk for STIs, the nurse should ask in a nonconfrontational manner whether condoms are being used during each episode of sexual intercourse. Asking a person whether he or she has an infection does not address the risk.

In obtaining a health history on a 74-year-old patient, the nurse notes that he drinks alcohol daily and that he has noticed a tremor in his hands that affects his ability to hold things. With this information, what response should the nurse make? a. Does your family know you are drinking every day? b. Does the tremor change when you drink alcohol? c. Well do some tests to see what is causing the tremor. d. You really shouldnt drink so much alcohol; it may be causing your tremor.

b. Does the tremor change when you drink alcohol? Senile tremor is relieved by alcohol, although not a recommended treatment. The nurse should assess whether the person is abusing alcohol in an effort to relieve the tremor.

During the examination portion of a patients visit, she will be in lithotomy position. Which statement reflects some things that the nurse can do to make this position more comfortable for her? a. Ask her to place her hands and arms over her head. b. Elevate her head and shoulders to maintain eye contact. c. Allow her to choose to have her feet in the stirrups or have them resting side by side on the edge of the table. d. Allow her to keep her buttocks approximately 6 inches from the edge of the table to prevent her from feeling as if she will fall off.

b. Elevate her head and shoulders to maintain eye contact. The nurse should elevate her head and shoulders to maintain eye contact. The patients arms should be placed at her sides or across the chest. Placing her hands and arms over her head only tightens the abdominal muscles. The feet should be placed into the stirrups, knees apart, and buttocks at the edge of the examining table. The stirrups are placed so that the legs are not abducted too far.

A 55-year-old man is experiencing severe pain of sudden onset in the scrotal area. It is somewhat relieved by elevation. On examination the nurse notices an enlarged, red scrotum that is very tender to palpation. Distinguishing the epididymis from the testis is difficult, and the scrotal skin is thick and edematous. This description is consistent with which of these? a. Varicocele b. Epididymitis c. Spermatocele d. Testicular torsion

b. Epididymitis Epididymitis presents as severe pain of sudden onset in the scrotum that is somewhat relieved by elevation. On examination, the scrotum is enlarged, reddened, and exquisitely tender. The epididymis is enlarged and indurated and may be hard to distinguish from the testis. The overlying scrotal skin may be thick and

A married couple has come to the clinic seeking advice on pregnancy. They have been trying to conceive for 4 months and have not been successful. What should the nurse do first? a. Ascertain whether either of them has been using broad-spectrum antibiotics. b. Explain that couples are considered infertile after 1 year of unprotected intercourse. c. Immediately refer the woman to an expert in pelvic inflammatory disease the most common cause of infertility. d. Explain that couples are considered infertile after 3 months of engaging in unprotected intercourse and that they will need a referral to a fertility expert.

b. Explain that couples are considered infertile after 1 year of unprotected intercourse. Infertility is considered after 1 year of engaging in unprotected sexual intercourse without conceiving. The other actions are not appropriate.

During a digital examination of the rectum, the nurse notices that the patient has hard feces in the rectum. The patient complains of feeling "full," has a distended abdomen, and states that she has not had a bowel movement "for several days." The nurse suspects which condition? a. Rectal polyp b. Fecal impaction c. Rectal abscess d. Rectal prolapse

b. Fecal impaction A fecal impaction is a collection of hard, desiccated feces in the rectum. The obstruction often results from decreased bowel motility, in which more water is reabsorbed from the stool.

During a digital examination of the rectum, the nurse notices that the patient has hard feces in the rectum. The patient complains of feeling full, has a distended abdomen, and states that she has not had a bowel movement for several days. The nurse suspects which condition? a. Rectal polyp b. Fecal impaction c. Rectal abscess d. Rectal prolapse

b. Fecal impaction A fecal impaction is a collection of hard, desiccated feces in the rectum. The obstruction often results from decreased bowel motility, in which more water is reabsorbed from the stool.

Which of the following statements is true regarding the internal structures of the breast? The breast is made up of: a. Primarily muscle with very little fibrous tissue. b. Fibrous, glandular, and adipose tissues. c. Primarily milk ducts, known as lactiferous ducts. d. Glandular tissue, which supports the breast by attaching to the chest wall.

b. Fibrous, glandular, and adipose tissues. The breast is made up of glandular, fibrous (including the suspensory ligaments), and adipose tissues.

The nurse is assessing a patients ischial tuberosity. To palpate the ischial tuberosity, the nurse knows that it is best to have the patient: a. Standing. b. Flexing the hip. c. Flexing the knee. d. Lying in the supine position.

b. Flexing the hip. The ischial tuberosity lies under the gluteus maximus muscle and is palpable when the hip is flexed. The other options are not correct.

An imaginary line connecting the highest point on each iliac crest would cross the vertebra. a. First sacral b. Fourth lumbar c. Seventh cervical d. Twelfth thoracic

b. Fourth lumbar An imaginary line connecting the highest point on each iliac crest crosses the fourth lumbar vertebra. The other options are not correct.

A patients annual physical examination reveals a lateral curvature of the thoracic and lumbar segments of his spine; however, this curvature disappears with forward bending. The nurse knows that this abnormality of the spine is called: a. Structural scoliosis. b. Functional scoliosis. c. Herniated nucleus pulposus. d. Dislocated hip.

b. Functional scoliosis. Functional scoliosis is flexible and apparent with standing but disappears with forward bending. Structural scoliosis is fixed; the curvature shows both when standing and when bending forward. These findings are not indicative of a dislocated hip

A patient tells the nurse that, All my life Ive been called knock knees. The nurse knows that another term for knock knees is: a. Genu varum. b. Genu valgum. c. Pes planus. d. Metatarsus adductus.

b. Genu valgum. Genu valgum is also known as knock knees and is present when more than 2.5 cm is between the medial malleoli when the knees are together.

When the nurse is discussing sexuality and sexual issues with an adolescent, a permission statement helps convey that it is normal to think or feel a certain way. Which statement is the best example of a permission statement? a. It is okay that you have become sexually active. b. Girls your age often have questions about sexual activity. Do you have any questions? c. If it is okay with you, Id like to ask you some questions about your sexual history. d. Girls your age often engage in sexual activities. It is okay to tell me if you have had intercourse.

b. Girls your age often have questions about sexual activity. Do you have any questions? The examiner should start with a permission statement such as, Girls your age often experience A permission statement conveys the idea that it is normal to think or feel a certain way, and implying that the topic is normal and unexceptional is important.

The nurse is palpating a female patients adnexa. The findings include a firm, smooth uterine wall; the ovaries are palpable and feel smooth and firm. The fallopian tube is firm and pulsating. The nurses most appropriate course of action would be to: a. Tell the patient that her examination is normal. b. Give her an immediate referral to a gynecologist. c. Suggest that she return in a month for a recheck to verify the findings. d. Tell the patient that she may have an ovarian cyst that should be evaluated further.

b. Give her an immediate referral to a gynecologist. Normally, the uterine wall feels firm and smooth, with the contour of the fundus rounded. Ovaries are not often palpable, but when they are, they normally feel smooth, firm, and almond shaped and are highly movable, sliding through the fingers. The fallopian tube is not normally palpable. No other mass or pulsation should be felt. Pulsation or palpable fallopian tube suggests ectopic pregnancy, which warrants immediate referral.

The nurse is examining the hip area of a patient and palpates a flat depression on the upper, lateral side of the thigh when the patient is standing. The nurse interprets this finding as the: a. Ischial tuberosity. b. Greater trochanter. c. Iliac crest. d. Gluteus maximus muscle

b. Greater trochanter. The greater trochanter of the femur is palpated when the person is standing, and it appears as a flat depression on the upper lateral side of the thigh. The iliac crest is the upper part of the hip bone; the ischial tuberosity lies under the gluteus maximus muscle and is palpable when the hip is flexed; and the gluteus muscle is part of the buttocks.

A 50-year-old woman calls the clinic because she has noticed some changes in her body and breasts and wonders if these changes could be attributable to the hormone replacement therapy (HRT) she started 3 months earlier. The nurse should tell her: a. HRT is at such a low dose that side effects are very unusual. b. HRT has several side effects, including fluid retention, breast tenderness, and vaginal bleeding. c. Vaginal bleeding with HRT is very unusual; I suggest you come into the clinic immediately to have this evaluated. d. It sounds as if your dose of estrogen is too high; I think you may need to decrease the amount you are taking and then call back in a week.

b. HRT has several side effects, including fluid retention, breast tenderness, and vaginal bleeding. Side effects of HRT include fluid retention, breast pain, and vaginal bleeding. The other responses are not correct.

The nurse is assessing a 1-month-old infant at his well-baby checkup. Which assessment findings are appropriate for this age? Select all that apply. a. Head circumference equal to chest circumference b. Head circumference greater than chest circumference c. Head circumference less than chest circumference d. Fontanels firm and slightly concave e. Absent tonic neck reflex f. Non-palpable cervical lymph nodes

b. Head circumference greater than chest circumference d. Fontanels firm and slightly concave f. Non-palpable cervical lymph nodes

A mother brings her newborn in for an assessment and asks, "Is there something wrong with my baby? His head seems so big." Which statement is true regarding the relative proportions of the head and trunk of the newborn? a. At birth, the head is one fifth the total length. b. Head circumference should be greater than chest circumference at birth. c. The head size reaches 90% of its final size when the child is 3 years old. d. When the anterior fontanel closes at 2 months, the head will be more proportioned to the body.

b. Head circumference should be greater than chest circumference at birth.

A 30-year-old woman is visiting the clinic because of "pain in my bottom when I have a bowel movement." The nurse should assess for which problem? a. Pinworms b. Hemorrhoids c. Colon cancer d. Fecal incontinence

b. Hemorrhoids Having painful bowel movements, known as dyschezia, may be attributable to a local condition (hemorrhoid or fissure) or constipation. The other responses are not correct.

A 30-year-old woman is visiting the clinic because of pain in my bottom when I have a bowel movement. The nurse should assess for which problem? a. Pinworms b. Hemorrhoids c. Colon cancer d. Fecal incontinence

b. Hemorrhoids Having painful bowel movements, known as dyschezia, may be attributable to a local condition (hemorrhoid or fissure) or constipation. The other responses are not correct.

The nurse suspects that a patient has otitis media. Early signs of otitis media include which of the following findings of the tympanic membrane? a. Red and bulging b. Hypomobility c. Retraction with landmarks clearly visible d. Flat, slightly pulled in at the center, and moves with insufflation

b. Hypomobility An early sign of otitis media is hypomobility of the tympanic membrane.

During a breast health interview, a patient states that she has noticed pain in her left breast. The nurses most appropriate response to this would be: a. Dont worry about the pain; breast cancer is not painful. b. I would like some more information about the pain in your left breast. c. Oh, I had pain like that after my son was born; it turned out to be a blocked milk duct. d. Breast pain is almost always the result of benign breast disease.

b. I would like some more information about the pain in your left breast. Breast pain occurs with trauma, inflammation, infection, or benign breast disease. The nurse will need to gather more information about the patients pain rather than make statements that ignore the patients concerns.

The nurse is describing how to perform a testicular self-examination to a patient. Which statement is most appropriate? a. A good time to examine your testicles is just before you take a shower. b. If you notice an enlarged testicle or a painless lump, call your health care provider. c. The testicle is egg shaped and movable. It feels firm and has a lumpy consistency. d. Perform a testicular examination at least once a week to detect the early stages of testicular cancer.

b. If you notice an enlarged testicle or a painless lump, call your health care provider. If the patient notices a firm painless lump, a hard area, or an overall enlarged testicle, then he should call his health care provider for further evaluation. The testicle normally feels rubbery with a smooth surface. A good time to examine the testicles is during the shower or bath, when ones hands are warm and soapy and the scrotum is warm. Testicular self-examination should be performed once a month.

The nurse is preparing to examine the external genitalia of a school-age girl. Which position would be most appropriate in this situation? a. In the parents lap b. In a frog-leg position on the examining table c. In the lithotomy position with the feet in stirrups d. Lying flat on the examining table with legs extended

b. In a frog-leg position on the examining table For school-age children, placing them on the examining table in a frog-leg position is best. With toddlers and preschoolers, having the child on the parents lap in a frog-leg position is best.

During an assessment of a 22-year-old woman who sustained a head injury from an automobile accident 4 hours earlier, the nurse notices the following changes: pupils were equal, but now the right pupil is fully dilated and nonreactive, and the left pupil is 4 mm and reacts to light. What do these findings suggest? a. Injury to the right eye b. Increased intracranial pressure c. Test inaccurately performed d. Normal response after a head injury

b. Increased intracranial pressure In a person with a brain injury, a sudden, unilateral, dilated, and nonreactive pupil is ominous. CN III runs parallel to the brainstem. When increasing intracranial pressure pushes down the brainstem (uncal herniation), it puts pressure on CN III, causing pupil dilation. The other responses are incorrect.

A patient has come in for an examination and states, "I have this spot in front of my ear lobe on my cheek that seems to be getting bigger and is tender. What do you think it is?" The nurse notes swelling below the angle of the jaw. What does the nurse suspect? a. Inflammation of the thyroid gland b. Inflammation of the parotid gland c. Infection in the occipital lymph node d. Infection in the submental lymph node

b. Inflammation of the parotid gland

If a patient reports a recent breast infection, then the nurse should expect to find node enlargement. a. Nonspecific b. Ipsilateral axillary c. Contralateral axillary d. Inguinal and cervical

b. Ipsilateral axillary The breast has extensive lymphatic drainage. Most of the lymph, more than 75%, drains into the ipsilateral, or same side, axillary nodes.

A patient says that she has recently noticed a lump in the front of her neck below her "Adam's apple" that seems to be getting bigger. During the assessment, what finding would lead the nurse to suspect that this may not be a cancerous thyroid nodule? a. It is tender. b. It is mobile and soft. c. It disappears when the patient smiles. d. It is hard and fixed to the surrounding structures.

b. It is mobile and soft.

Subcutaneous fat is not lost but is redistributed to the abdomen and hips. A patient has been diagnosed with osteoporosis and asks the nurse, What is osteoporosis? The nurse explains that osteoporosis is defined as: a. Increased bone matrix. b. Loss of bone density. c. New, weaker bone growth. d. Increased phagocytic activity.

b. Loss of bone density. After age 40 years, a loss of bone matrix (resorption) occurs more rapidly than new bone formation. The net effect is a gradual loss of bone density, or osteoporosis. The other options are not correct.

During an examination, the nurse notices severe nystagmus in both eyes of a patient. Which conclusion by the nurse is correct? Severe nystagmus in both eyes: a. Is a normal occurrence. b. May indicate disease of the cerebellum or brainstem. c. Is a sign that the patient is nervous about the examination. d. Indicates a visual problem, and a referral to an ophthalmologist is indicated.

b. May indicate disease of the cerebellum or brainstem. End-point nystagmus at an extreme lateral gaze normally occurs; however, the nurse should carefully assess any other nystagmuses. Severe nystagmus occurs with disease of the vestibular system, cerebellum, or brainstem.

During the neurologic assessment of a healthy 35-year-old patient, the nurse asks him to relax his muscles completely. The nurse then moves each extremity through full range of motion. Which of these results would the nurse expect to find? a. Firm, rigid resistance to movement b. Mild, even resistance to movement c. Hypotonic muscles as a result of total relaxation d. Slight pain with some directions of movement

b. Mild, even resistance to movement Tone is the normal degree of tension (contraction) in voluntarily relaxed muscles. It shows a mild resistance to passive stretching. Normally, the nurse will notice a mild, even resistance to movement. The other responses are not correct.

During an assessment of the CNs, the nurse finds the following: asymmetry when the patient smiles or frowns, uneven lifting of the eyebrows, sagging of the lower eyelids, and escape of air when the nurse presses against the right puffed cheek. This would indicate dysfunction of which of these CNs? a. Motor component of CN IV b. Motor component of CN VII c. Motor and sensory components of CN XI d. Motor component of CN X and sensory component of CNVII

b. Motor component of CN VII The findings listed reflect a dysfunction of the motor component of the facial nerve (CN VII).

The nurse has just completed an inspection of a nulliparous womans external genitalia. Which of these would be a description of a finding within normal limits? a. Redness of the labia majora b. Multiple nontender sebaceous cysts c. Discharge that is foul smelling and irritating d. Gaping and slightly shriveled labia majora

b. Multiple nontender sebaceous cysts No lesions should be noted, except for the occasional sebaceous cysts, which are yellowish 1-cm nodules that are firm, nontender, and often multiple. The labia majora are dark pink, moist, and symmetric; redness indicates inflammation or lesions. Discharge that is foul smelling and irritating may indicate infection. In the nulliparous woman, the labia majora meet in the midline, are symmetric and plump.

A mother of a 1-month-old infant asks the nurse why it takes so long for infants to learn to roll over. The nurse knows that the reason for this is: a. A demyelinating process must be occurring with her infant. b. Myelin is needed to conduct the impulses, and the neurons of a newborn are not yet myelinated. c. The cerebral cortex is not fully developed; therefore, control over motor function gradually occurs. d. The spinal cord is controlling the movement because the cerebellum is not yet fully developed.

b. Myelin is needed to conduct the impulses, and the neurons of a newborn are not yet myelinated. The infants sensory and motor development proceeds along with the gradual acquisition of myelin, which is needed to conduct most impulses. Very little cortical control exists, and the neurons are not yet myelinated. The other responses are not correct.

A woman comes to the clinic and states, "I've been sick for so long! My eyes have gotten so puffy, and my eyebrows and hair have become coarse and dry." For what condition should the nurse assess for other signs and symptoms? a. Cachexia b. Myxedema c. Graves disease d. Parkinson syndrome

b. Myxedema NOTE: A deficiency of thyroid hormone reduces the metabolic rate and can causes a non-pitting edema or myxedema. Symptoms include fatigue and cold intolerance. Signs include puffy, edematous face, especially around eyes (periorbital edema); puffy hands and feet; coarse facial features; cool, dry skin; dry, coarse hair and eyebrows; slow reflexes; and sometimes thick speech.

The nurse has just completed a lymph node assessment on a 60-year-old healthy female patient. When palpating the nodes on this healthy 60-year-old adult, how did the lymph nodes feel? a. Fixed b. Non-palpable c. Rubbery, discrete, and mobile d. Large, firm, and fixed to the tissue

b. Non-palpable NOTE: Small, soft, non-tender, moveable, non-palpable lymph nodes are normal findings. Large, firm, tender, rubbery, fixed are abnormal findings. (Moveable can be abnormal when combined with other abnormal findings).

A patient states during the interview that she noticed a new lump in the shower a few days ago. It was on her left breast near her axilla. The nurse should plan to: a. Palpate the lump first. b. Palpate the unaffected breast first. c. Avoid palpating the lump because it could be a cyst, which might rupture. d. Palpate the breast with the lump first but plan to palpate the axilla last.

b. Palpate the unaffected breast first. If the woman mentions a breast lump she has discovered herself, then the nurse should examine the unaffected breast first to learn a baseline of normal consistency for this individual.

A male patient with a history of acquired immunodeficiency syndrome (AIDS) has come in for an examination and states, "I think that I have the mumps." What should the nurse examine first? a. Thyroid gland b. Parotid gland c. Cervical lymph nodes d. Mouth and skin for lesions

b. Parotid gland

When examining the face of a patient, what are the two pairs of salivary glands that are accessible for examination? a. Occipital; submental b. Parotid; submandibular c. Submandibular; occipital d. Sublingual; parotid

b. Parotid; submandibular

The nurse knows that testing kinesthesia is a test of a persons: a. Fine touch. b. Position sense. c. Motor coordination. d. Perception of vibration.

b. Position sense. Kinesthesia, or position sense, is the persons ability to perceive passive movements of the extremities. The other options are incorrect.

The nurse is assessing the neurologic status of a patient who has a late-stage brain tumor. With the reflex hammer, the nurse draws a light stroke up the lateral side of the sole of the foot and inward, across the ball of the foot. In response, the patients toes fan out, and the big toe shows dorsiflexion. The nurse interprets this result as: a. Negative Babinski sign, which is normal for adults. b. Positive Babinski sign, which is abnormal for adults. c. Clonus, which is a hyperactive response. d. Achilles reflex, which is an expected response.

b. Positive Babinski sign, which is abnormal for adults. Dorsiflexion of the big toe and fanning of all toes is a positive Babinski sign, also called up-going toes. This response occurs with upper motor neuron disease of the corticospinal (or pyramidal) tract and is an abnormal finding for adults.

The structure that secretes a thin, milky alkaline fluid to enhance the viability of sperm is the: a. Cowper gland b. Prostate gland c. Median sulcus d. Bulbourethral gland

b. Prostate gland In men, the prostate gland secretes a thin milky alkaline fluid that enhances sperm viability. The Cowper glands (also known as bulbourethral glands) secrete a clear, viscid mucus. The median sulcus is a groove that divides the lobes of the prostate gland and does not secrete fluid.

The structure that secretes a thin, milky alkaline fluid to enhance the viability of sperm is the: a. Cowper gland. b. Prostate gland. c. Median sulcus. d. Bulbourethral gland.

b. Prostate gland. In men, the prostate gland secretes a thin milky alkaline fluid that enhances sperm viability. The Cowper glands (also known as bulbourethral glands) secrete a clear, viscid mucus. The median sulcus is a groove that divides the lobes of the prostate gland and does not secrete fluid.

During a breast examination on a female patient, the nurse notices that the nipple is flat, broad, and fixed. The patient states it started doing that a few months ago. This finding suggests: a. Dimpling. b. Retracted nipple. c. Nipple inversion. d. Deviation in nipple pointing.

b. Retracted nipple. The retracted nipple looks flatter and broader, similar to an underlying crater. A recent retraction suggests cancer, which causes fibrosis of the whole duct system and pulls in the nipple. It also may occur with benign lesions such as ectasia of the ducts. The nurse should not confuse retraction with the normal long-standing type of nipple inversion, which has no broadening and is not fixed.

A patient is able to flex his right arm forward without difficulty or pain but is unable to abduct his arm because of pain and muscle spasms. The nurse should suspect: a. Crepitation. b. Rotator cuff lesions. c. Dislocated shoulder. d. Rheumatoid arthritis.

b. Rotator cuff lesions. Rotator cuff lesions may limit range of motion and cause pain and muscle spasms during abduction, whereas forward flexion remains fairly normal. The other options are not correct.

A patient in her first trimester of pregnancy is diagnosed with rubella. The nurse recognizes that the significance of this in relation to the infant's hearing is which of the following? a. Rubella may affect the mother's hearing but not the infant's. b. Rubella can damage the infant's organ of Corti, which will impair hearing. c. Rubella is only dangerous to the infant in the second trimester of pregnancy. d. Rubella can impair the development of CN VIII and thus affect hearing.

b. Rubella can damage the infant's organ of Corti, which will impair hearing. If maternal rubella infection occurs during the first trimester, it can damage the organ of Corti and impair hearing.

The external male genital structures include the: a. Testis. b. Scrotum. c. Epididymis. d. Vas deferens.

b. Scrotum. The external male genital structures include the penis and scrotum. The testis, epididymis, and vas deferens are internal structures.

When assessing the scrotum of a male patient, the nurse notices the presence of multiple firm, nontender, yellow 1-cm nodules. The nurse knows that these nodules are most likely: a. From urethritis. b. Sebaceous cysts. c. Subcutaneous plaques. d. From an inflammation of the epididymis.

b. Sebaceous cysts. Sebaceous cysts are commonly found on the scrotum. These yellowish 1-cm nodules are firm, nontender, and often multiple. The other options are not correct.

A 54-year-old woman who has just completed menopause is in the clinic today for a yearly physical examination. Which of these statements should the nurse include in patient education? A postmenopausal woman: a. Is not at any greater risk for heart disease than a younger woman. b. Should be aware that she is at increased risk for dyspareunia because of decreased vaginal secretions. c. Has only stopped menstruating; there really are no other significant changes with which she should be concerned. d. Is likely to have difficulty with sexual pleasure as a result of drastic changes in the female sexual response cycle.

b. Should be aware that she is at increased risk for dyspareunia because of decreased vaginal secretions. Decreased vaginal secretions leave the vagina dry and at risk for irritation and pain with intercourse (dyspareunia). The other statements are incorrect.

A 40-year-old black man is in the office for his annual physical examination. Which statement regarding the PSA blood test is true, according to the American Cancer Society? The PSA: a. Should be performed with this visit. b. Should be performed at age 45 years. c. Should be performed at age 50 years. d. Is only necessary if a family history of prostate cancer exists.

b. Should be performed at age 45 years. According to the American Cancer Society (2006), the PSA blood test should be performed annually for black men beginning at age 45 years and annually for all other men over age 50 years.

A 40-year-old black man is in the office for his annual physical examination. Which statement regarding the PSA blood test is true, according to the American Cancer Society? The PSA: a. Should be performed with this visit. b. Should be performed at age 45 years. c. Should be performed at age 50 years. d. Is only necessary if a family history of prostate cancer exists.

b. Should be performed at age 45 years. According to the American Cancer Society (2006), the PSA blood test should be performed annually for black men beginning at age 45 years and annually for all other men over age 50 years.

During an examination, the nurse would expect the cervical os of a woman who has never had children to appear: a. Stellate. b. Small and round. c. As a horizontal irregular slit. d. Everted.

b. Small and round. The cervical os in a nulliparous woman is small and round. In the parous woman, it is a horizontal, irregular slit that also may show healed lacerations on the sides.

A 52-year-old patient states that when she sneezes or coughs she wets herself a little. She is very concerned that something may be wrong with her. The nurse suspects that the problem is: a. Dysuria. b. Stress incontinence. c. Hematuria. d. Urge incontinence.

b. Stress incontinence. Stress incontinence is involuntary urine loss with physical strain, sneezing, or coughing. Dysuria is pain or burning with urination. Hematuria is bleeding with urination. Urge incontinence is involuntary urine loss that occurs as a result of an overactive detrusor muscle in the bladder that contracts and causes an urgent need to void.

When performing a scrotal assessment, the nurse notices that the scrotal contents show a red glow with transillumination. On the basis of this finding the nurse would: a. Assess the patient for the presence of a hernia. b. Suspect the presence of serous fluid in the scrotum. c. Consider this finding normal, and proceed with the examination. d. Refer the patient for evaluation of a mass in the scrotum.

b. Suspect the presence of serous fluid in the scrotum. Normal scrotal contents do not allow light to pass through the scrotum. However, serous fluid does transilluminate and shows as a red glow. Neither a mass nor a hernia would transilluminate.

The nurse suspects that a patient has hyperthyroidism, and the laboratory data indicate that the patient's T4 and T3 hormone levels are elevated. Which of these findings would the nurse most likely find on examination? a. Dyspnea b. Tachycardia c. Constipation d. Atrophied nodular thyroid gland

b. Tachycardia THINK: hyperthyroidism makes everything speed up, if the thyroid is producing more hormone it is likely to be larger, not atrophied.

Which characteristic of the prostate gland would the nurse recognize as an abnormal finding while palpating the prostate gland through the rectum? a. Palpable central groove b. Tenderness to palpation c. Heart shaped d. Elastic and rubbery consistency

b. Tenderness to palpation The normal prostate gland should feel smooth, elastic, and rubbery; slightly movable; heart-shaped with a palpable central groove; and not be tender to palpation.

Which characteristic of the prostate gland would the nurse recognize as an abnormal finding while palpating the prostate gland through the rectum? a. Palpable central groove b. Tenderness to palpation c. Heart shaped d. Elastic and rubbery consistency

b. Tenderness to palpation The normal prostate gland should feel smooth, elastic, and rubbery; slightly movable; heart-shaped with a palpable central groove; and not be tender to palpation.

The nurse is performing an examination of the anus and rectum. Which of these statements is correct and important to remember during this examination? a. The rectum is approximately 8 cm long. b. The anorectal junction cannot be palpated. c. Above the anal canal, the rectum turns anteriorly. d. No sensory nerves are in the anal canal or rectum.

b. The anorectal junction cannot be palpated. The anal columns are folds of mucosa that extend vertically down from the rectum and end in the anorectal junction. This junction is not palpable but is visible on proctoscopy. The rectum is 12 cm long; just above the anal canal, the rectum dilates and turns posteriorly.

Which statement concerning the sphincters is correct? a. The internal sphincter is under voluntary control. b. The external sphincter is under voluntary control. c. Both sphincters remain slightly relaxed at all times. d. The internal sphincter surrounds the external sphincter.

b. The external sphincter is under voluntary control. The external sphincter surrounds the internal sphincter but also has a small section overriding the tip of the internal sphincter at the opening. The external sphincter is under voluntary control. Except for the passing of feces and gas, the sphincters keep the anal canal tightly closed.

The mother of a 10-year-old boy asks the nurse to discuss the recognition of puberty. The nurse should reply by saying: a. Puberty usually begins around 15 years of age. b. The first sign of puberty is an enlargement of the testes. c. The penis size does not increase until about 16 years of age. d. The development of pubic hair precedes testicular or penis enlargement.

b. The first sign of puberty is an enlargement of the testes. Puberty begins sometime between age 9 for African Americans and age 10 for Caucasians and Hispanics. The first sign is an enlargement of the testes. Pubic hair appears next, and then penis size increases.

Which statement concerning the areas of the brain is true? a. The cerebellum is the center for speech and emotions. b. The hypothalamus controls body temperature and regulates sleep. c. The basal ganglia are responsible for controlling voluntary movements. d. Motor pathways of the spinal cord and brainstem synapse in the thalamus.

b. The hypothalamus controls body temperature and regulates sleep. The hypothalamus is a vital area with many important functions: body temperature controller, sleep center, anterior and posterior pituitary gland regulator, and coordinator of autonomic nervous system activity and emotional status. The cerebellum controls motor coordination, equilibrium, and balance. The basal ganglia control autonomic movements of the body. The motor pathways of the spinal cord synapse in various areas of the spinal cord, not in the thalamus.

The nurse is conducting a visual examination. Which of these statements regarding visual pathways and visual fields is true? a. The right side of the brain interprets the vision for the right eye. b. The image formed on the retina is upside down and reversed from its actual appearance in the outside world. c. Light rays are refracted through the transparent media of the eye before striking the pupil. d. Light impulses are conducted through the optic nerve to the temporal lobes of the brain.

b. The image formed on the retina is upside down and reversed from its actual appearance in the outside world. The image formed on the retina is upside down and reversed from its actual appearance in the outside world. The light rays are refracted through the transparent media of the eye before striking the retina, and the nerve impulses are conducted through the optic nerve tract to the visual cortex of the occipital lobe of the brain. The left side of the brain interprets vision for the right eye.

In performing a breast examination, the nurse knows that examining the upper outer quadrant of the breast is especially important. The reason for this is that the upper outer quadrant is: a. The largest quadrant of the breast. b. The location of most breast tumors. c. Where most of the suspensory ligaments attach. d. More prone to injury and calcifications than other locations in the breast.

b. The location of most breast tumors. The upper outer quadrant is the site of most breast tumors. In the upper outer quadrant, the nurse should notice the axillary tail of Spence, the cone-shaped breast tissue that projects up into the axilla, close to the pectoral group of axillary lymph nodes

An 85-year-old patient comments during his annual physical examination that he seems to be getting shorter as he ages. The nurse should explain that decreased height occurs with aging because: a. Long bones tend to shorten with age. b. The vertebral column shortens. c. A significant loss of subcutaneous fat occurs. d. A thickening of the intervertebral disks develops.

b. The vertebral column shortens. Postural changes are evident with aging; decreased height is most noticeable and is due to shortening of the vertebral column. Long bones do not shorten with age. Intervertebral disks actually get thinner with age.

The assessment of a 60-year-old patient has taken longer than anticipated. In testing his pain perception, the nurse decides to complete the test as quickly as possible. When the nurse applies the sharp point of the pin on his arm several times, he is only able to identify these as one very sharp prick. What would be the most accurate explanation for this? a. The patient has hyperesthesia as a result of the aging process. b. This response is most likely the result of the summation effect. c. The nurse was probably not poking hard enough with the pin in the other areas. d. The patient most likely has analgesia in some areas of arm and hyperalgesia in others.

b. This response is most likely the result of the summation effect. At least 2 seconds should be allowed to elapse between each stimulus to avoid summation. With summation, frequent consecutive stimuli are perceived as one strong stimulus. The other responses are incorrect.

A woman who has had rheumatoid arthritis for years is starting to notice that her fingers are drifting to the side. The nurse knows that this condition is commonly referred to as: a. Radial drift. b. Ulnar deviation. c. Swan-neck deformity. d. Dupuytren contracture

b. Ulnar deviation. Fingers drift to the ulnar side because of stretching of the articular capsule and muscle imbalance caused by chronic rheumatoid arthritis. A radial drift is not observed.

During an internal examination, the nurse notices that the cervix bulges outside the introitus when the patient is asked to strain. The nurse will document this as: a. Uterine prolapse, graded first degree. b. Uterine prolapse, graded second degree. c. Uterine prolapse, graded third degree. d. A normal finding.

b. Uterine prolapse, graded second degree. The cervix should not be found to bulge into the vagina. Uterine prolapse is graded as follows: first degreethe cervix appears at the introitus with straining; second degreethe cervix bulges outside the introitus with straining; and third degreethe whole uterus protrudes, even without straining (essentially, the uterus is inside out).

The nurse notices that a patient's palpebral fissures are not symmetric. On examination, the nurse may find that damage has occurred to which cranial nerve (CN)? a. V b. VII c. XI d. XIII

b. VII

In performing a voice test to assess hearing, which of the following would the nurse do? a. Shield the lips so that the sound is muffled. b. Whisper two-syllable words and ask the patient to repeat them. c. Ask the patient to place his finger in his ear to occlude outside noise. d. Stand about 4 feet away to ensure that the patient can really hear at this distance.

b. Whisper two-syllable words and ask the patient to repeat them. With your head 30 to 60 cm (1 to 2 ft) from the person's ear, exhale and whisper slowly some two-syllable words such as Tuesday, armchair, baseball, or fourteen. Normally, the person repeats each word correctly after you say it.

Which of the following groups of individuals need to be tested for the presence of color blindness (deficiency)? a. White girls between the ages of 4 and 8 years b. White boys between the ages of 4 and 8 years c. Asian girls between the ages of 3 and 6 years d. Black boys between the ages of 10 and 15 years

b. White boys between the ages of 4 and 8 years Color blindness is an inherited recessive X-linked trait affecting about 8% of white boys and 4% of black boys. Test only boys for color vision once between the ages of 4 and 8 years.

While performing the otoscopic exam of a 3-year-old boy who has been pulling on his left ear, the nurse finds that his left tympanic membrane is bright red and the light reflex is not visible. The most likely cause is: a. fungal infection. b. acute otitis media. c. rupture of the drum. d. blood behind the drum.

b. acute otitis media. Absent or distorted light reflex and a bright red color of the eardrum are indicative of acute otitis media.

Bleeding into the periosteum during birth is known as a. caput succedaneum. b. cephalhematoma. c. molding. d. craniosynostosis.

b. cephalhematoma. A cephalhematoma is a subperiosteal hemorrhage resulting from birth trauma. A caput succedaneum is edematous swelling and ecchymosis of the presenting part of the head caused by birth trauma. Craniosynostosis is marked asymmetry that is due to a severe deformity caused by premature closure of the sutures and resulting in a long, narrow head. Molding of the cranial bones during passage through the birth canal makes the head asymmetric and ridges more prominent.

The normal color of the optic disc is a. red. b. creamy yellow-orange to pink. c. creamy pink. d. creamy red to yellow-orange.

b. creamy yellow-orange to pink. The color of a normal optic disc ranges from creamy-yellow orange to pink.

Binaural interaction at the level of the brainstem permits a. amplification of sound. b. identification and location of the direction of the sound. c. direction of sound toward the appropriate conduction pathway. d. interpretation of sound.

b. identification and location of the direction of the sound. The function at the brainstem level is binaural interaction, which permits locating the direction of a sound in space as well as identifying the sound.

The nurse is performing a middle ear assessment on a 15-year-old patient who has a history of chronic ear infections. When examining the right tympanic membrane, the nurse sees the presence of dense white patches. The tympanic membrane is otherwise unremarkable. It is pearly, with the light reflex at 5 o'clock and landmarks visible. The nurse should: a. refer the patient for the possibility of a fungal infection. b. know that these are scars caused from frequent ear infections. c. consider that these findings may represent the presence of blood in the middle ear. d. be concerned about the ability to hear because of this abnormality on the tympanic membrane.

b. know that these are scars caused from frequent ear infections. Dense white patches on the tympanic membrane are sequelae of repeated ear infections. They do not necessarily affect hearing.

When examining the ear with an otoscope, the nurse remembers that the tympanic membrane should appear: a. light pink with a slight bulge. b. pearly gray and slightly concave. c. pulled in at the base of the cone of light. d. whitish with a small fleck of light in the superior portion.

b. pearly gray and slightly concave. It is a translucent membrane with a pearly gray color and a prominent cone of light in the anteroinferior quadrant, which is the reflection of the otoscope light. The drum is oval and slightly concave, pulled in at its center by one of the middle ear ossicles, the malleus.

The extraocular muscles consist of four straight or ________ muscles and two slanting or ______ muscles. a. rectilinear/diagonal b. rectus/oblique c. palpebral/conjugate d. superior/inferior

b. rectus/oblique The four straight, or rectus, muscles are the superior, inferior, lateral, and medial rectus muscles. The two slanting, or oblique, muscles are the superior and inferior muscles.

A 17-year-old student is a swimmer on her high school's swim team. She has had three bouts of otitis externa so far this season and wants to know what to do to prevent it. The nurse instructs her to: a. use a cotton-tipped swab to dry the ear canals thoroughly after each swim. b. use rubbing alcohol or 2% acetic acid eardrops after every swim. c. irrigate the ears with warm water and a bulb syringe after each swim. d. rinse the ears with a warmed solution of mineral oil and hydrogen peroxide.

b. use rubbing alcohol or 2% acetic acid eardrops after every swim. With otitis externa (swimmer's ear), swimming causes the external canal to become waterlogged and swell; skinfolds are set up for infection. Prevent by using rubbing alcohol or 2% acetic acid ear drops after every swim.

When an otoscopic examination is performed on an older adult patient, the tympanic membrane may be a. thinner than that of a younger adult. b. whiter than that of a younger adult. c. pinker than that of a younger adult. d. more mobile than that of a younger adult.

b. whiter than that of a younger adult. During otoscopy, the tympanic membrane of an older adult may be whiter in color than that of a younger adult. The tympanic membrane may also appear more opaque and dull. A yellow-amber eardrum color occurs with otitis media with effusion. A red color occurs with acute otitis media. The tympanic membrane of an older adult may be thicker compared with that of a younger adult. Hypomobility is an early sign of acute otitis media.

benign prostatic hyperplasia (BPH)

benign enlargement of the prostate gland

A patient has had three pregnancies and two live births. The nurse would record this information as grav ____, para ____, AB ____. a. 2; 2; 1 b. 3; 2; 0 c. 3; 2; 1 d. 3; 3; 1

c. 3; 2; 1 Gravida (grav) is the number of pregnancies. Para is the number of births. Abortions are interrupted pregnancies, including elective abortions and spontaneous miscarriages.

During an examination of a patient in her third trimester of pregnancy, the nurse notices that the patient's thyroid gland is slightly enlarged. No enlargement had been previously noticed. What does the nurse suspect? a. An iodine deficiency b. Early signs of goiter c. A normal enlargement of the thyroid gland during pregnancy d. Possible thyroid cancer and the need for further evaluation

c. A normal enlargement of the thyroid gland during pregnancy

A 2-year-old boy has been diagnosed with physiologic cryptorchidism. Considering this diagnosis, during assessment the nurse will most likely observe: a. Testes that are hard and painful to palpation. b. Atrophic scrotum and a bilateral absence of the testis. c. Absence of the testis in the scrotum, but the testis can be milked down. d. Testes that migrate into the abdomen when the child squats or sits cross-legged.

c. Absence of the testis in the scrotum, but the testis can be milked down. Migratory testes (physiologic cryptorchidism) are common because of the strength of the cremasteric reflex and the small mass of the prepubertal testes. The affected side has a normally developed scrotum and the testis can be milked down. The other responses are not correct.

While assessing a patient who is hospitalized and bedridden, the nurse notices that the patient has been incontinent of stool. The stool is loose and gray-tan in color. The nurse recognizes that this finding indicates which of the following? a. Occult blood b. Inflammation c. Absent bile pigment d. Ingestion of iron preparations

c. Absent bile pigment The presence of gray-tan stool indicates absent bile pigment, which can occur with obstructive jaundice. The ingestion of iron preparations and the presence of occult blood turns the stools to a black color. Jellylike mucus shreds mixed in the stool would indicate inflammation.

While assessing a patient who is hospitalized and bedridden, the nurse notices that the patient has been incontinent of stool. The stool is loose and gray-tan in color. The nurse recognizes that this finding indicates which of the following? a. Occult blood b. Inflammation c. Absent bile pigment d. Ingestion of iron preparations

c. Absent bile pigment The presence of gray-tan stool indicates absent bile pigment, which can occur with obstructive jaundice. The ingestion of iron preparations and the presence of occult blood turns the stools to a black color. Jellylike mucus shreds mixed in the stool would indicate inflammation

A man who was found wandering in a park at 2 AM has been brought to the emergency department for an examination; he said he fell and hit his head. During the examination, the nurse asks him to use his index finger to touch the nurses finger, then his own nose, then the nurses finger again (which has been moved to a different location). The patient is clumsy, unable to follow the instructions, and overshoots the mark, missing the finger. The nurse should suspect which of the following? a. Cerebral injury b. Cerebrovascular accident c. Acute alcohol intoxication d. Peripheral neuropathy

c. Acute alcohol intoxication During the finger-to-finger test, if the person has clumsy movement with overshooting the mark, either a cerebellar disorder or acute alcohol intoxication should be suspected. The persons movements should be smooth and accurate. The other options are not correct.

A patient is being assessed for range-of-joint movement. The nurse asks him to move his arm in toward the center of his body. This movement is called: a. Flexion. b. Abduction. c. Adduction. d. Extension.

c. Adduction. Moving a limb toward the midline of the body is called adduction; moving a limb away from the midline of the body is called abduction. Flexion is bending a limb at a joint; and extension is straightening a limb at a joint.

A slight protrusion of the eyeballs may be noticed when examining individuals who come from which ethnic/cultural group? a. Hispanic b. American Indian c. African American d. Asian

c. African American African Americans normally may have a slight protrusion of the eyeball beyond the supraorbital ridge.

A patient contacts the office and tells the nurse that she is worried about her 10-year-old daughter having breast cancer. She describes a unilateral enlargement of the right breast with associated tenderness. She is worried because the left breast is not enlarged. What would be the nurses best response? Tell the mother that: a. Breast development is usually fairly symmetric and that the daughter should be examined right away. b. She should bring in her daughter right away because breast cancer is fairly common in preadolescent girls. c. Although an examination of her daughter would rule out a problem, her breast development is most likely normal. d. It is unusual for breasts that are first developing to feel tender because they havent developed much fibrous tissue.

c. Although an examination of her daughter would rule out a problem, her breast development is most likely normal. Occasionally, one breast may grow faster than the other, producing a temporary asymmetry, which may cause some distress; reassurance is necessary. Tenderness is also common.

To palpate the temporomandibular joint, the nurses fingers should be placed in the depression of the ear. a. Distal to the helix b. Proximal to the helix c. Anterior to the tragus d. Posterior to the tragus

c. Anterior to the tragus The temporomandibular joint can be felt in the depression anterior to the tragus of the ear. The other locations are not correct.

During a health history interview, a female patient states that she has noticed a few drops of clear discharge from her right nipple. What should the nurse do next? a. Immediately contact the physician to report the discharge. b. Ask her if she is possibly pregnant. c. Ask the patient some additional questions about the medications she is taking. d. Immediately obtain a sample for culture and sensitivity testing.

c. Ask the patient some additional questions about the medications she is taking. The use of some medications, such as oral contraceptives, phenothiazines, diuretics, digitalis, steroids, methyldopa, and calcium channel blockers, may cause clear nipple discharge. Bloody or blood-tinged discharge from the nipple, not clear, is significant, especially if a lump is also present. In the pregnant female, colostrum would be a thick, yellowish liquid, and it would be normally expressed after the fourth month of pregnancy.

During an examination of a woman, the nurse notices that her left breast is slightly larger than her right breast. Which of these statements is true about this finding? a. Breasts should always be symmetric b. Asymmetry of breast size and shape is probably due to breastfeeding and is nothing to worry about. c. Asymmetry is not unusual, but the nurse should verify that this change is not new. d. Asymmetry of breast size and shape is very unusual and means she may have an inflammation or growth.

c. Asymmetry is not unusual, but the nurse should verify that this change is not new. The nurse should notice symmetry of size and shape. It is common to have a slight asymmetry in size; often the left breast is slightly larger than the right. A sudden increase in the size of one breast signifies inflammation or new growth.

The nurse is preparing to teach a woman about BSE. Which statement by the nurse is correct? a. BSE is more important than ever for you because you have never had any children. b. BSE is so important because one out of nine women will develop breast cancer in her lifetime. c. BSE on a monthly basis will help you become familiar with your own breasts and feel their normal variations. d. BSE will save your life because you are likely to find a cancerous lump between mammograms.

c. BSE on a monthly basis will help you become familiar with your own breasts and feel their normal variations. The nurse should stress that a regular monthly BSE will familiarize the woman with her own breasts and their normal variations. BSE is a positive step that will reassure her of her healthy state. While teaching, the nurse should focus on the positive aspects of BSE and avoid citing frightening mortality statistics about breast cancer, which may generate excessive fear and denial that can obstruct a womans self-care actions.

A woman is in the family planning clinic seeking birth control information. She states that her breasts change all month long and that she is worried that this is unusual. What is the nurses best response? The nurse should tell her that: a. Continual changes in her breasts are unusual. The breasts of nonpregnant women usually stay pretty much the same all month long. b. Breast changes in response to stress are very common and that she should assess her life for stressful events. c. Because of the changing hormones during the monthly menstrual cycle, cyclic breast changes are common. d. Breast changes normally occur only during pregnancy and that a pregnancy test is needed at this time.

c. Because of the changing hormones during the monthly menstrual cycle, cyclic breast changes are common. Breasts of the nonpregnant woman change with the ebb and flow of hormones during the monthly menstrual cycle. During the 3 to 4 days before menstruation, the breasts feel full, tight, heavy, and occasionally sore. The breast volume is smallest on days 4 to 7 of the menstrual cycle.

The nurse is preparing for a class in early detection of breast cancer. Which statement is true with regard to breast cancer in black women in the United States? a. Breast cancer is not a threat to black women. b. Black women have a lower incidence of regional or distant breast cancer than white women. c. Black women are more likely to die of breast cancer at any age. d. Breast cancer incidence in black women is higher than that of white women after age 45.

c. Black women are more likely to die of breast cancer at any age. Black women have a higher incidence of breast cancer before age 45 years than white women and are more likely to die of their disease. In addition, black women are significantly more likely to be diagnosed with regional or distant breast cancer than are white women. These racial differences in mortality rates may be related to an insufficient use of screening measures and a lack of access to health care.

Which of the following cranial nerves is responsible for conducting nerve impulses to the brain from the organ of Corti? a. CN I b. CN III c. CN VIII d. CN XI

c. CN VIII The nerve impulses are conducted by the auditory portion of CN VIII to the brain.

A positive Phalen test and Tinel sign are found in a patient with: a. Torn meniscus b. Temporomandibular joint c. Carpal tunnel syndrome d. Tennis elbow

c. Carpal tunnel syndrome

In performing an assessment of a womans axillary lymph system, the nurse should assess which of these nodes? a. Central, axillary, lateral, and sternal b. Pectoral, lateral, anterior, and sternal c. Central, lateral, pectoral, and subscapular d. Lateral, pectoral, axillary, and suprascapular

c. Central, lateral, pectoral, and subscapular The breast has extensive lymphatic drainage. Four groups of axillary nodes are present: (1) central, (2) pectoral (anterior), (3) subscapular (posterior), and (4) lateral.

The nurse notices that an infant has a large, soft lump on the side of his head and that his mother is very concerned. The mother tells the nurse that she noticed the lump approximately 8 hours after her baby's birth and that it seems to be getting bigger. What is a possible explanation for this? a. Hydrocephalus b. Craniosynostosis c. Cephalohematoma d. Caput succedaneum

c. Cephalohematoma NOTE: A cephalohematoma is an accumulation of blood under the scalp that occurs during the birthing process when forces on the skull and scalp result in the separation of the periosteum from the underlying calvarium resulting in the rupture of blood vessels.

A 30-year-old woman tells the nurse that she has been very unsteady and has had difficulty in maintaining her balance. Which area of the brain that is related to these findings would concern the nurse? a. Thalamus b. Brainstem c. Cerebellum d. Extrapyramidal tract

c. Cerebellum The cerebellar system coordinates movement, maintains equilibrium, and helps maintain posture. The thalamus is the primary relay station where sensory pathways of the spinal cord, cerebellum, and brainstem form synapses on their way to the cerebral cortex. The brainstem consists of the midbrain, pons, and medulla and has various functions, especially concerning autonomic centers. The extrapyramidal tract maintains muscle tone for gross automatic movements, such as walking.

A 46-year-old man requires an assessment of his sigmoid colon. Which instrument or technique is most appropriate for this examination? a. Proctoscope b. Ultrasound c. Colonoscope d. Rectal examination with an examining finger

c. Colonoscope The sigmoid colon is 40 cm long, and the nurse knows that it is accessible to examination only with the colonoscope. The other responses are not appropriate for an examination of the entire sigmoid colon.

A 46-year-old man requires an assessment of his sigmoid colon. Which instrument or technique is most appropriate for this examination? a. Proctoscope b. Ultrasound c. Colonoscope d. Rectal examination with an examining finger

c. Colonoscope The sigmoid colon is 40 cm long, and the nurse knows that it is accessible to examination only with the colonoscope. The other responses are not appropriate for an examination of the entire sigmoid colon.

During an examination of a 3-year-old child, the nurse notices a bruit over the left temporal area. What should the nurse do? a. Check for the bruit again in 1 hour. b. Stop the examination, and notify the physician. c. Continue the examination because a bruit is a normal finding for this age. d. Notify the parents that a bruit has been detected in their child and requires further evaluation.

c. Continue the examination because a bruit is a normal finding for this age.

A patient is unable to differentiate between sharp and dull stimulation to both sides of her face. What does the nurse suspect? a. Bell palsy b. Scleroderma c. Damage to the trigeminal nerve d. Frostbite with resultant paresthesia to the cheeks

c. Damage to the trigeminal nerve

During an examination of an aging man, the nurse recognizes that normal changes to expect would be: a. Enlarged scrotal sac. b. Increased pubic hair. c. Decreased penis size. d. Increased rugae over the scrotum.

c. Decreased penis size. In the aging man, the amount of pubic hair decreases, the penis size decreases, and the rugae over the scrotal sac decreases. The scrotal sac does not enlarge.

A 55-year-old postmenopausal woman is being seen in the clinic for her annual examination. She is concerned about changes in her breasts that she has noticed over the past 5 years. She states that her breasts have decreased in size and that the elasticity has changed so that her breasts seem flat and flabby. The nurses best reply would be: a. This change occurs most often because of long-term use of bras that do not provide enough support to the breast tissues. b. This is a normal change that occurs as women get older and is due to the increased levels of progesterone during the aging process. c. Decreases in hormones after menopause causes atrophy of the glandular tissue in the breast and is a normal process of aging. d. Postural changes in the spine make it appear that your breasts have changed in shape. Exercises to strengthen the muscles of the upper back and chest wall will help prevent the changes in elasticity and size.

c. Decreases in hormones after menopause causes atrophy of the glandular tissue in the breast and is a normal process of aging. The hormonal changes of menopause cause the breast glandular tissue to atrophy, making the breasts more pendulous, flattened, and sagging.

edematous. The nurse is performing a genitourinary assessment on a 50-year-old obese male laborer. On examination, the nurse notices a painless round swelling close to the pubis in the area of the internal inguinal ring that is easily reduced when the individual is supine. These findings are most consistent with a(n) hernia. a. Scrotal b. Femoral c. Direct inguinal d. Indirect inguinal

c. Direct inguinal Direct inguinal hernias occur most often in men over the age of 40 years. It is an acquired weakness brought on by heavy lifting, obesity, chronic cough, or ascites. The direct inguinal hernia is usually a painless, round swelling close to the pubis in the area of the internal inguinal ring that is easily reduced when the individual is supine.

When taking the health history on a patient with a seizure disorder, the nurse assesses whether the patient has an aura. Which of these would be the best question for obtaining this information? a. Does your muscle tone seem tense or limp? b. After the seizure, do you spend a lot of time sleeping? c. Do you have any warning sign before your seizure starts? d. Do you experience any color change or incontinence during the seizure?

c. Do you have any warning sign before your seizure starts? Aura is a subjective sensation that precedes a seizure; it could be auditory, visual, or motor. The other questions do not solicit information about an aura.

The nurse is performing an assessment on a 29-year-old woman who visits the clinic complaining of always dropping things and falling down. While testing rapid alternating movements, the nurse notices that the woman is unable to pat both of her knees. Her response is extremely slow and she frequently misses. What should the nurse suspect? a. Vestibular disease b. Lesion of CN IX c. Dysfunction of the cerebellum d. Inability to understand directions

c. Dysfunction of the cerebellum When a person tries to perform rapid, alternating movements, responses that are slow, clumsy, and sloppy are indicative of cerebellar disease. The other responses are incorrect.

When examining a patients eyes, the nurse recalls that stimulation of the sympathetic branch of the autonomic nervous system: a. Causes pupillary constriction. b. Adjusts the eye for near vision. c. Elevates the eyelid and dilates the pupil. d. Causes contraction of the ciliary body.

c. Elevates the eyelid and dilates the pupil. Stimulation of the sympathetic branch of the autonomic nervous system dilates the pupil and elevates the eyelid. Parasympathetic nervous system stimulation causes the pupil to constrict. The muscle fibers of the iris contract the pupil in bright light to accommodate for near vision. The ciliary body controls the thickness of the lens.

The nurse is explaining the mechanism of the growth of long bones to a mother of a toddler. Where does lengthening of the bones occur? a. Bursa b. Calcaneus c. Epiphyses d. Tuberosities

c. Epiphyses Lengthening occurs at the epiphyses, or growth plates. The other options are not correct.

Generally, the changes normally associated with menopause occur because the cells in the reproductive tract are: a. Aging. b. Becoming fibrous. c. Estrogen dependent. d. Able to respond to estrogen.

c. Estrogen dependent. Because cells in the reproductive tract are estrogen dependent, decreased estrogen levels during menopause bring dramatic physical changes. The other options are not correct.

During a discussion about BSEs with a 30-year-old woman, which of these statements by the nurse is most appropriate? a. The best time to examine your breasts is during ovulation. b. Examine your breasts every month on the same day of the month. c. Examine your breasts shortly after your menstrual period each month. d. The best time to examine your breasts is immediately before menstruation

c. Examine your breasts shortly after your menstrual period each month. The best time to conduct a BSE is shortly after the menstrual period when the breasts are the smallest and least congested.

In examining a 70-year-old male patient, the nurse notices that he has bilateral gynecomastia. Which of the following describes the nurses best course of action? a. Recommend that he make an appointment with his physician for a mammogram. b. Ignore it. Benign breast enlargement in men is not unusual. c. Explain that this condition may be the result of hormonal changes, and recommend that he see his physician. d. Explain that gynecomastia in men is usually associated with prostate enlargement and recommend that he be thoroughly screened.

c. Explain that this condition may be the result of hormonal changes, and recommend that he see his physician. Gynecomastia may reappear in the aging man and may be attributable to a testosterone deficiency.

When the nurse is testing the triceps reflex, what is the expected response? a. Flexion of the hand b. Pronation of the hand c. Extension of the forearm d. Flexion of the forearm

c. Extension of the forearm The normal response of the triceps reflex is extension of the forearm. The normal response of the biceps reflex causes flexion of the forearm. The other responses are incorrect.

During an examination of a female patient, the nurse notes lymphadenopathy and suspects an acute infection. How do acutely infected lymph nodes typically appear? a. Clumped b. Unilateral c. Firm but freely movable d. Soft and nontender

c. Firm but freely movable NOTE: Infections are firm but moveable, if they were immovable cancer would be more likely as it may have metastasized to its surroundings.

During a genital examination, the nurse notices that a male patient has clusters of small vesicles on the glans, surrounded by erythema. The nurse recognizes that these lesions are: a. Peyronie disease. b. Genital warts. c. Genital herpes. d. Syphilitic cancer.

c. Genital herpes. Genital herpes, or herpes simplex virus 2 (HSV-2), infections are indicated with clusters of small vesicles with surrounding erythema, which are often painful and erupt on the glans or foreskin.

During an external genitalia examination of a woman, the nurse notices several lesions around the vulva. The lesions are pink, moist, soft, and pointed papules. The patient states that she is not aware of any problems in that area. The nurse recognizes that these lesions may be: a. Syphilitic chancre. b. Herpes simplex virus type 2 (herpes genitalis). c. HPV or genital warts. d. Pediculosis pubis (crab lice).

c. HPV or genital warts. HPV lesions are painless, warty growths that the woman may not notice. Lesions are pink or flesh colored, soft, pointed, moist, warty papules that occur in single or multiple cauliflower-like patches around the vulva, introitus, anus, vagina, or cervix. Herpetic lesions are painful clusters of small, shallow vesicles with surrounding erythema. Syphilitic chancres begin as a solitary silvery papule that erodes into a red, round or oval superficial ulcer with a yellowish discharge. Pediculosis pubis causes severe perineal itching and excoriations and erythematous areas.

During an examination, the nurse asks a patient to bend forward from the waist and notices that the patient has lateral tilting. When his leg is raised straight up, the patient complains of a pain going down his buttock into his leg. The nurse suspects: a. Scoliosis. b. Meniscus tear. c. Herniated nucleus pulposus. d. Spasm of paravertebral muscles.

c. Herniated nucleus pulposus. Lateral tilting and sciatic pain with straight leg raising are findings that occur with a herniated nucleus pulposus. The other options are not correct.

The nurse suspects that a patient has carpal tunnel syndrome and wants to perform the Phalen test. To perform this test, the nurse should instruct the patient to: a. Dorsiflex the foot. b. Plantarflex the foot. c. Hold both hands back to back while flexing the wrists 90 degrees for 60 seconds. d. Hyperextend the wrists with the palmar surface of both hands touching, and wait for 60 seconds.

c. Hold both hands back to back while flexing the wrists 90 degrees for 60 seconds. For the Phalen test, the nurse should ask the person to hold both hands back to back while flexing the wrists 90 degrees. Acute flexion of the wrist for 60 seconds produces no symptoms in the normal hand. The Phalen test reproduces numbness and burning in a person with carpal tunnel syndrome. The other actions are not correct when testing for carpal tunnel syndrome.

During a well-baby checkup, the nurse notices that a 1-week-old infant's face looks small compared with his cranium, which seems enlarged. On further examination, the nurse also notices dilated scalp veins and downcast or "setting sun" eyes. The nurse suspects which condition? a. Craniotabes b. Microcephaly c. Hydrocephalus d. Caput succedaneum

c. Hydrocephalus

During the interview with a female patient, the nurse gathers data that indicate the patient is perimenopausal. Which of these statements made by this patient leads to this conclusion? a. I have noticed that my muscles ache at night when I go to bed. b. I will be very happy when I can stop worrying about having a period. c. I have been noticing that I sweat a lot more than I used to, especially at night. d. I have only been pregnant twice, but both times I had breast tenderness as my first symptom.

c. I have been noticing that I sweat a lot more than I used to, especially at night. Hormone shifts occur during the perimenopausal period, and associated symptoms of menopause may occur, such as hot flashes, night sweats, numbness and tingling, headache, palpitations, drenching sweats, mood swings, vaginal dryness, and itching. The other responses are not correct.

During the interview, a patient reveals that she has some vaginal discharge. She is worried that it may be a sexually transmitted infection. The nurses most appropriate response to this would be: a. Oh, dont worry. Some cyclic vaginal discharge is normal. b. Have you been engaging in unprotected sexual intercourse? c. Id like some information about the discharge. What color is it? d. Have you had any urinary incontinence associated with the discharge?

c. Id like some information about the discharge. What color is it? Questions that help the patient reveal more information about her symptoms should be asked in a nonthreatening manner. Asking about the amount, color, and odor of the vaginal discharge provides the opportunity for further assessment. Normal vaginal discharge is small, clear or cloudy, and always nonirritating.

The nurse notices that a patient has had a pale, yellow, greasy stool, or steatorrhea, and recalls that this is caused by: a. Occult bleeding. b. Absent bile pigment. c. Increased fat content. d. Ingestion of bismuth preparations

c. Increased fat content. Steatorrhea (pale, yellow, greasy stool) is caused by increased fat content in the stools, as in malabsorption syndrome. Occult bleeding and ingestion of bismuth products cause a black stool, and absent bile pigment causes a gray-tan stool.

The nurse notices that a patient has had a pale, yellow, greasy stool, or steatorrhea, and recalls that this is caused by: a. Occult bleeding. b. Absent bile pigment. c. Increased fat content. d. Ingestion of bismuth preparations

c. Increased fat content. Steatorrhea (pale, yellow, greasy stool) is caused by increased fat content in the stools, as in malabsorption syndrome. Occult bleeding and ingestion of bismuth products cause a black stool, and absent bile pigment causes a gray-tan stool.

The nurse needs to palpate the temporomandibular joint for crepitation. Where is this joint located? a. Just below the hyoid bone and posterior to the tragus b. Just below the vagus nerve and posterior to the mandible c. Just below the temporal artery and anterior to the tragus d. Just below the temporal artery and anterior to the mandible

c. Just below the temporal artery and anterior to the tragus

The nurse is examining only the rectal area of a woman and should place the woman in what position? a. Lithotomy b. Prone c. Left lateral decubitus d. Bending over the table while standing

c. Left lateral decubitus The nurse should place the female patient in the lithotomy position if the genitalia are being examined as well. The left lateral decubitus position is used for the rectal area alone.

The nurse is examining only the rectal area of a woman and should place the woman in what position? a. Lithotomy b. Prone c. Left lateral decubitus d. Bending over the table while standing

c. Left lateral decubitus The nurse should place the female patient in the lithotomy position if the genitalia are being examined as well. The left lateral decubitus position is used for the rectal area alone.

The nurse is caring for a patient who has just had neurosurgery. To assess for increased intracranial pressure, what would the nurse include in the assessment? a. CNs, motor function, and sensory function b. Deep tendon reflexes, vital signs, and coordinated movements c. Level of consciousness, motor function, pupillary response, and vital signs d. Mental status, deep tendon reflexes, sensory function, and pupillary response

c. Level of consciousness, motor function, pupillary response, and vital signs Some hospitalized persons have head trauma or a neurologic deficit from a systemic disease process. These people must be closely monitored for any improvement or deterioration in neurologic status and for any indication of increasing intracranial pressure. The nurse should use an abbreviation of the neurologic examination in the following sequence: level of consciousness, motor function, pupillary response, and vital signs.

A mother brings her newborn baby boy in for a checkup; she tells the nurse that he does not seem to be moving his right arm as much as his left and that he seems to have pain when she lifts him up under the arms. The nurse suspects a fractured clavicle and would observe for: a. Negative Allis test. b. Positive Ortolani sign. c. Limited range of motion during the Moro reflex. d. Limited range of motion during Lasgue test.

c. Limited range of motion during the Moro reflex. For a fractured clavicle, the nurse should observe for limited arm range of motion and unilateral response to the Moro reflex. The other tests are not appropriate for this type of fracture.

Most facial bones articulate at a suture. Which facial bone articulates at a joint? a. Zygomatic bone b. Nasal bone c. Mandible d. Maxilla

c. Mandible The facial bones articulate at sutures (nasal bone, zygomatic bone, and maxilla) except for the mandible. The mandible articulates at the temporomandibular joint.

The nurse is aware of which statement to be true regarding the incidence of testicular cancer? a. Testicular cancer is the most common cancer in men aged 30 to 50 years. b. The early symptoms of testicular cancer are pain and induration. c. Men with a history of cryptorchidism are at the greatest risk for the development of testicular cancer. d. The cure rate for testicular cancer is low.

c. Men with a history of cryptorchidism are at the greatest risk for the development of testicular cancer. Men with undescended testicles (cryptorchidism) are at the greatest risk for the development of testicular cancer. The overall incidence of testicular cancer is rare. Although testicular cancer has no early symptoms, when detected early and treated before metastasizing, the cure rate is almost 100%.

A patient is visiting the clinic for an evaluation of a swollen, painful knuckle. The nurse notices that the knuckle above his ring on the left hand is swollen and that he is unable to remove his wedding ring. This joint is called the joint. a. Interphalangeal b. Tarsometatarsal c. Metacarpophalangeal d. Tibiotalar

c. Metacarpophalangeal The joint located just above the ring on the finger is the metacarpophalangeal joint. The interphalangeal joint is located distal to the metacarpophalangeal joint. The tarsometatarsal and tibiotalar joints are found in the foot and ankle.

A patient, an 85-year-old woman, is complaining about the fact that the bones in her face have become more noticeable. What explanation should the nurse give her? a. Diets low in protein and high in carbohydrates may cause enhanced facial bones. b. Bones can become more noticeable if the person does not use a dermatologically approved moisturizer. c. More noticeable facial bones are probably due to a combination of factors r/t aging, such as decreased elasticity, subcutaneous fat, and moisture in her skin. d. Facial skin becomes more elastic with age. This increased elasticity causes the skin to be more taught, drawing attention to the facial bones.

c. More noticeable facial bones are probably due to a combination of factors r/t aging, such as decreased elasticity, subcutaneous fat, and moisture in her skin.

A 70-year-old patient tells the nurse that he has noticed that he is having trouble hearing, especially in large groups. He says he "can't always tell where the sound is coming from" and the words often sound "mixed up." What might the nurse suspect as the cause for this change? a. Atrophy of the apocrine glands b. Cilia becoming coarse and stiff c. Nerve degeneration in the inner ear d. Scarring of the tympanic membrane

c. Nerve degeneration in the inner ear Presbycusis is a type of hearing loss that occurs with aging, even in people living in a quiet environment. It is a gradual sensorineural loss caused by nerve degeneration in the inner ear or auditory nerve. This makes words sound garbled. The ability to localize sound is impaired also. This communication dysfunction is accentuated when background noise is present.

A mother brings in her newborn infant for an assessment and tells the nurse that she has noticed that whenever her newborns head is turned to the right side, she straightens out the arm and leg on the same side and flexes the opposite arm and leg. After observing this on examination, the nurse tells her that this reflex is: a. Abnormal and is called the atonic neck reflex. b. Normal and should disappear by the first year of life. c. Normal and is called the tonic neck reflex, which should disappear between 3 and 4 months of age. d. Abnormal. The baby should be flexing the arm and leg on the right side of his body when the head is turned to the right.

c. Normal and is called the tonic neck reflex, which should disappear between 3 and 4 months of age. THINK: Picture a baby shooting a bow and arrow, they are looking at their target, the arm is out stretched pointing towards where they want the arrow to go and the arm behind them is flexed pulling back the arrow. After they hit the bullseye 3-4 times they'll go drink their vodka tonic, as babies do.

During an assessment of an 80-year-old patient, the nurse notices the following: an inability to identify vibrations at her ankle and to identify the position of her big toe, a slower and more deliberate gait, and a slightly impaired tactile sensation. All other neurologic findings are normal. The nurse should interpret that these findings indicate: a. CN dysfunction. b. Lesion in the cerebral cortex. c. Normal changes attributable to aging. d. Demyelination of nerves attributable to a lesion.

c. Normal changes attributable to aging. Some aging adults show a slower response to requests, especially for those calling for coordination of movements. The findings listed are normal in the absence of other significant abnormal findings. The other responses are incorrect.

A patient who is visiting the clinic complains of having "stomach pains for 2 weeks" and describes his stools as being "soft and black" for approximately the last 10 days. He denies taking any medications. The nurse is aware that these symptoms are mostly indicative of: a. Excessive fat caused by malabsorption. b. Increased iron intake, resulting from a change in diet. c. Occult blood, resulting from gastrointestinal bleeding. d. Absent bile pigment from liver problems.

c. Occult blood, resulting from gastrointestinal bleeding. Black stools may be tarry as a result of occult blood (melena) from gastrointestinal bleeding or non-tarry from ingestion of iron medications (not diet). Excessive fat causes the stool to become frothy. The absence of bile pigment causes clay-colored stools.

A patient who is visiting the clinic complains of having stomach pains for 2 weeks and describes his stools as being soft and black for approximately the last 10 days. He denies taking any medications. The nurse is aware that these symptoms are mostly indicative of: a. Excessive fat caused by malabsorption. b. Increased iron intake, resulting from a change in diet. c. Occult blood, resulting from gastrointestinal bleeding. d. Absent bile pigment from liver problems.

c. Occult blood, resulting from gastrointestinal bleeding. Black stools may be tarry as a result of occult blood (melena) from gastrointestinal bleeding or nontarry from ingestion of iron medications (not diet). Excessive fat causes the stool to become frothy. The absence of bile pigment causes clay-colored stools.

A 54-year-old man comes to the clinic with a horrible problem. He tells the nurse that he has just discovered a lump on his breast and is fearful of cancer. The nurse knows which statement about breast cancer in men is true? a. Breast masses in men are difficult to detect because of minimal breast tissue. b. Breast cancer in men rarely spreads to the lymph nodes. c. One percent of all breast cancers occurs in men. d. Most breast masses in men are diagnosed as gynecomastia.

c. One percent of all breast cancers occurs in men. One percent of all breast cancers occurs in men. The early spreading to axillary lymph nodes is attributable to minimal breast tissue.

During a bimanual examination, the nurse detects a solid tumor on the ovary that is heavy and fixed, with a poorly defined mass. This finding is suggestive of: a. Ovarian cyst. b. Endometriosis. c. Ovarian cancer. d. Ectopic pregnancy.

c. Ovarian cancer. Ovarian tumors that are solid, heavy, and fixed, with poorly defined mass are suggestive of malignancy. Benign masses may feel mobile and solid. An ovarian cyst may feel smooth, round, fluctuant, mobile, and nontender. With an ectopic pregnancy, the examiner may feel a palpable, tender pelvic mass that is solid, mobile, and unilateral. Endometriosis may have masses (in various locations in the pelvic area) that are small, firm, nodular, and tender to palpation, with enlarged ovaries.

While examining a 75-year-old woman, the nurse notices that the skin over her right breast is thickened and the hair follicles are exaggerated. This condition is known as: a. Dimpling. b. Retraction. c. Peau dorange. d. Benign breast disease.

c. Peau dorange. This condition is known as peau dorange. Lymphatic obstruction produces edema, which thickens the skin and exaggerates the hair follicles. The skin has a pig-skin or orange-peel appearance, and this condition suggests cancer.

The nurse is teaching a class on preventing osteoporosis to a group of perimenopausal women. Which of these actions is the best way to prevent or delay bone loss in this group? a. Taking calcium and vitamin D supplements b. Taking medications to prevent osteoporosis c. Performing physical activity, such as fast walking d. Assessing bone density annually

c. Performing physical activity, such as fast walking Physical activity, such as fast walking, delays or prevents bone loss in perimenopausal women. The faster the pace of walking, the higher the preventive effect is on the risk of hip fracture. The other options are not correct

The nurse is performing a neurologic assessment on a 41-year-old woman with a history of diabetes. When testing her ability to feel the vibrations of a tuning fork, the nurse notices that the patient is unable to feel vibrations on the great toe or ankle bilaterally, but she is able to feel vibrations on both patellae. Given this information, what would the nurse suspect? a. Hyperalgesia b. Hyperesthesia c. Peripheral neuropathy d. Lesion of sensory cortex

c. Peripheral neuropathy Loss of vibration sense occurs with peripheral neuropathy (e.g., diabetes and alcoholism). Peripheral neuropathy is worse at the feet and gradually improves as the examiner moves up the leg, as opposed to a specific nerve lesion, which has a clear zone of deficit for its dermatome. The other responses are incorrect.

The nurse is testing superficial reflexes on an adult patient. When stroking up the lateral side of the sole and across the ball of the foot, the nurse notices the plantar flexion of the toes. How should the nurse document this finding? a. Positive Babinski sign b. Plantar reflex abnormal c. Plantar reflex present d. Plantar reflex 2+ on a scale from 0 to 4+

c. Plantar reflex present With the same instrument, the nurse should draw a light stroke up the lateral side of the sole of the foot and across the ball of the foot, similar to an upside-down J. The normal response is plantar flexion of the toes and sometimes of the entire foot. A positive Babinski sign is abnormal and occurs with the response of dorsiflexion of the big toe and fanning of all toes. The plantar reflex is not graded on a 0 to 4+ scale.

During a neonatal examination, the nurse notices that the newborn infant has six toes. This finding is documented as: a. Unidactyly. b. Syndactyly. c. Polydactyly. d. Multidactyly.

c. Polydactyly. Polydactyly is the presence of extra fingers or toes. Syndactyly is webbing between adjacent fingers or toes. The other terms are not correct.

A patient is unable to perform rapid alternating movements such as rapidly patting her knees. The nurse should document this inability as: a. Ataxia. b. Astereognosis. c. Presence of dysdiadochokinesia. d. Loss of kinesthesia.

c. Presence of dysdiadochokinesia. Slow clumsy movements and the inability to perform rapid alternating movements occur with cerebellar disease. The condition is termed dysdiadochokinesia. Ataxia is an uncoordinated or unsteady gait. Astereognosis is the inability to identify an object by feeling it. Kinesthesia is the persons ability to perceive passive movement of the extremities or the loss of position sense.

An accessory glandular structure for the male genital organs is the: a. Testis. b. Scrotum. c. Prostate. d. Vas deferens.

c. Prostate. Glandular structures accessory to the male genital organs are the prostate, seminal vesicles, and bulbourethral glands

The nurse is performing an otoscopic examination on an adult. Which of the following is true? a. Tilt the person's head forward during the exam. b. Once the speculum is in the ear, release the traction. c. Pull the pinna up and back before inserting the speculum. d. Use the smallest speculum to decrease the amount of discomfort.

c. Pull the pinna up and back before inserting the speculum. Pull the pinna up and back on an adult or older child. This helps straighten the S- shape of the canal.

The physician reports that a patient with a neck tumor has a tracheal shift. The nurse should understand that what is occurring to the patient's trachea? a. Pushed downward b. Pulled to the affected side c. Pushed to the unaffected side d. Pulled downward in a rhythmic pattern

c. Pushed to the unaffected side THINK: The tumor is pushing the trachea out of it's way so it has more room to grow

During an assessment of the newborn, the nurse expects to see which finding when the anal area is slightly stroked? a. Jerking of the legs b. Flexion of the knees c. Quick contraction of the sphincter d. Relaxation of the external sphincter

c. Quick contraction of the sphincter To assess sphincter tone, the nurse should check the anal reflex by gently stroking the anal area and noticing a quick contraction of the sphincter.

During an assessment of the newborn, the nurse expects to see which finding when the anal area is slightly stroked? a. Jerking of the legs b. Flexion of the knees c. Quick contraction of the sphincter d. Relaxation of the external sphincter

c. Quick contraction of the sphincter To assess sphincter tone, the nurse should check the anal reflex by gently stroking the anal area and noticing a quick contraction of the sphincter. The other responses are not correct.

An 18-year-old patient is having her first pelvic examination. Which action by the nurse is appropriate? a. Inviting her mother to be present during the examination b. Avoiding the lithotomy position for this first time because it can be uncomfortable and embarrassing c. Raising the head of the examination table and giving her a mirror so that she can view the examination d. Fully draping her, leaving the drape between her legs elevated to avoid embarrassing her with eye contact

c. Raising the head of the examination table and giving her a mirror so that she can view the examination The techniques of the educational or mirror pelvic examination should be used. This is a routine examination with some modifications in attitude, position, and communication. First, the woman is considered an active participant, one who is interested in learning and in sharing decisions about her own health care. The woman props herself up on one elbow, or the head of the table is raised. Her other hand holds a mirror between her legs, above the examiners hands. The young woman can see all that the examiner is doing and has a full view of her genitalia. The mirror works well for teaching normal anatomy and its relationship to sexual behavior. The examiner can ask her if she would like to have a family member, friend, or chaperone present for the examination. The drape should be pushed down between the patients legs so that the nurse can see her face.

A 2-month-old uncircumcised infant has been brought to the clinic for a well-baby checkup. How would the nurse proceed with the genital examination? a. Eliciting the cremasteric reflex is recommended. b. The glans is assessed for redness or lesions. c. Retracting the foreskin should be avoided until the infant is 3 months old. d. Any dirt or smegma that has collected under the foreskin should be noted.

c. Retracting the foreskin should be avoided until the infant is 3 months old. If uncircumcised, then the foreskin is normally tight during the first 3 months and should not be retracted because of the risk of tearing the membrane attaching the foreskin to the shaft. The other options are not correct.

A patient is complaining of pain in his joints that is worse in the morning, better after he moves around for a while, and then gets worse again if he sits for long periods. The nurse should assess for other signs of what problem? a. Tendinitis b. Osteoarthritis c. Rheumatoid arthritis d. Intermittent claudication

c. Rheumatoid arthritis Rheumatoid arthritis is worse in the morning when a person arises. Movement increases most joint pain, except the pain with rheumatoid arthritis, which decreases with movement. The other options are not correct

A teenage girl has arrived complaining of pain in her left wrist. She was playing basketball when she fell and landed on her left hand. The nurse examines her hand and would expect a fracture if the girl complains of a: a. Dull ache. b. Deep pain in her wrist. c. Sharp pain that increases with movement. d. Dull throbbing pain that increases with rest

c. Sharp pain that increases with movement. A fracture causes sharp pain that increases with movement. The other types of pain do not occur with a fracture.

The nurse has palpated a lump in a female patients right breast. The nurse documents this as a small, round, firm, distinct, lump located at 2 oclock, 2 cm from the nipple. It is nontender and fixed. No associated retraction of the skin or nipple, no erythema, an no axillary lymphadenopathy are observed. What information is missing from the documentation? a. Shape of the lump b. Consistency of the lump c. Size of the lump d. Whether the lump is solitary or multiple

c. Size of the lump If the nurse feels a lump or mass, then he or she should note these characteristics: (1) location, (2) size judge in centimeters in three dimensions: width length thickness, (3) shape, (4) consistency, (5) motility, (6) distinctness, (7) nipple, (8) the skin over the lump, (9) tenderness, and (10) lymphadenopathy.

A patients thyroid gland is enlarged, and the nurse is preparing to auscultate the thyroid gland for the presence of a bruit. A bruit is a __________ sound that is heard best with the __________ of the stethoscope. a. Low gurgling; diaphragm b. Loud, whooshing, blowing; bell c. Soft, whooshing, pulsatile; bell d. High-pitched tinkling; diaphragm

c. Soft, whooshing, pulsatile; bell

The nurse is preparing for an internal genitalia examination of a woman. Which order of the examination is correct? a. Bimanual, speculum, and rectovaginal b. Speculum, rectovaginal, and bimanual c. Speculum, bimanual, and rectovaginal d. Rectovaginal, bimanual, and speculum

c. Speculum, bimanual, and rectovaginal The correct sequence is speculum examination, then bimanual examination after removing the speculum, and then rectovaginal examination. The examiner should change gloves before performing the rectovaginal examination to avoid spreading any possible infection.

The area of the nervous system that is responsible for mediating reflexes is the: a. Medulla. b. Cerebellum. c. Spinal cord. d. Cerebral cortex

c. Spinal cord. The spinal cord is the main highway for ascending and descending fiber tracts that connect the brain to the spinal nerves; it is responsible for mediating reflexes.

A 45-year-old mother of two children is seen at the clinic for complaints of losing my urine when I sneeze. The nurse documents that she is experiencing: a. Urinary frequency. b. Enuresis. c. Stress incontinence. d. Urge incontinence.

c. Stress incontinence. Stress incontinence is involuntary urine loss with physical strain, sneezing, or coughing that occurs as a result to weakness of the pelvic floor. Urinary frequency is urinating more times than usual (more than five to six times per day). Enuresis is involuntary passage of urine at night after age 5 to 6 years (bed wetting). Urge incontinence is involuntary urine loss from overactive detrusor muscle in the bladder. It contracts, causing an urgent need to void.

The nurse is assessing a 1-week-old infant and is testing his muscle strength. The nurse lifts the infant with hands under the axillae and notices that the infant starts to slip between the hands. The nurse should: a. Suspect a fractured clavicle b. Suspect that the infant may have a deformity of the spine. c. Suspect that the infant may have weakness of the shoulder muscles. d. Conclude that this is a normal finding because the musculature of an infant at this age is undeveloped.

c. Suspect that the infant may have weakness of the shoulder muscles. An infant who starts to slip between the nurses hands shows weakness of the shoulder muscles. An infant with normal muscle strength wedges securely between the nurses hands. The other responses are not correct.

A patient who has had rheumatoid arthritis for years comes to the clinic to ask about changes in her fingers. The nurse will assess for signs of what problems? a. Heberden nodes b. Bouchard nodules c. Swan-neck deformities a. Dupuytren contractures

c. Swan-neck deformities Changes in the fingers caused by chronic rheumatoid arthritis include swan-neck and boutonniere deformities. Heberden nodes and Bouchard nodules are associated with osteoarthritis. Dupuytren contractures of the digits occur because of chronic hyperplasia of the palmar fascia.

During an examination, the nurse notices that a male patient has a red, round, superficial ulcer with a yellowish serous discharge on his penis. On palpation, the nurse finds a nontender base that feels like a small button between the thumb and fingers. At this point the nurse suspects that this patient has: a. Genital warts. b. Herpes infection. c. Syphilitic chancre. d. Carcinoma lesion.

c. Syphilitic chancre. This lesion indicates syphilitic chancre, which begins within 2 to 4 weeks of infection.

When a breastfeeding mother is diagnosed with a breast abscess, which of these instructions from the nurse is correct? The mother needs to: a. Continue to nurse on both sides to encourage milk flow. b. Immediately discontinue nursing to allow for healing. c. Temporarily discontinue nursing on the affected breast, and manually express milk and discard it. d. Temporarily discontinue nursing on affected breast, but manually express milk and give it to the baby.

c. Temporarily discontinue nursing on the affected breast, and manually express milk and discard it. With a breast abscess, the patient must temporarily discontinue nursing on the affected breast, manually express the milk, and then discard it. Nursing can continue on the unaffected side.

The articulation of the mandible and the temporal bone is known as the: a. Intervertebral foramen. b. Condyle of the mandible. c. Temporomandibular joint. d. Zygomatic arch of the temporal bone.

c. Temporomandibular joint. The articulation of the mandible and the temporal bone is the temporomandibular joint. The other responses are not correct.

When the nurse is performing a testicular examination on a 25-year-old man, which finding is considered normal? a. Nontender subcutaneous plaques b. Scrotal area that is dry, scaly, and nodular c. Testes that feel oval and movable and are slightly sensitive to compression d. Single, hard, circumscribed, moveable mass, less than 1 cm under the surface of the testes

c. Testes that feel oval and movable and are slightly sensitive to compression Testes normally feel oval, firm and rubbery, smooth, and bilaterally equal and are freely movable and slightly tender to moderate pressure. The scrotal skin should not be dry, scaly, or nodular or contain subcutaneous plaques. Any mass would be an abnormal finding.

A patient has a severed spinal nerve as a result of trauma. Which statement is true in this situation? a. Because there are 31 pairs of spinal nerves, no effect results if only one nerve is severed. b. The dermatome served by this nerve will no longer experience any sensation. c. The adjacent spinal nerves will continue to carry sensations for the dermatome served by the severed nerve. d. A severed spinal nerve will only affect motor function of the patient because spinal nerves have no sensory component.

c. The adjacent spinal nerves will continue to carry sensations for the dermatome served by the severed nerve. A dermatome is a circumscribed skin area that is primarily supplied from one spinal cord segment through a particular spinal nerve. The dermatomes overlap, which is a form of biologic insurance; that is, if one nerve is severed, then most of the sensations can be transmitted by the spinal nerve above and the spinal nerve below the severed nerve.

The nurse is conducting a class on BSE. Which of these statements indicates the proper BSE technique? a. The best time to perform BSE is in the middle of the menstrual cycle. b. The woman needs to perform BSE only bimonthly unless she has fibrocystic breast tissue. c. The best time to perform a BSE is 4 to 7 days after the first day of the menstrual period. d. If she suspects that she is pregnant, then the woman should not perform a BSE until her baby is born.

c. The best time to perform a BSE is 4 to 7 days after the first day of the menstrual period. The nurse should help each woman establish a regular schedule of self-care. The best time to conduct a BSE is right after the menstrual period, or the fourth through seventh day of the menstrual cycle, when the breasts are the smallest and least congested. The pregnant or menopausal woman who is not having menstrual periods should be advised to select a familiar date to examine her breasts each month for example, her birth date or the day the rent is due.

The nurse is examining a 35-year-old female patient. During the health history, the nurse notices that she has had two term pregnancies, and both babies were delivered vaginally. During the internal examination, the nurse observes that the cervical os is a horizontal slit with some healed lacerations and that the cervix has some nabothian cysts that are small, smooth, and yellow. In addition, the nurse notices that the cervical surface is granular and red, especially around the os. Finally, the nurse notices the presence of stringy, opaque, odorless secretions. Which of these findings are abnormal? a. Nabothian cysts are present. b. The cervical os is a horizontal slit. c. The cervical surface is granular and red. d. Stringy and opaque secretions are present.

c. The cervical surface is granular and red. Normal findings: Nabothian cysts may be present on the cervix after childbirth. The cervical os is a horizontal, irregular slit in the parous woman. Secretions vary according to the day of the menstrual cycle, and may be clear and thin or thick, opaque, and stringy. The surface is normally smooth, but cervical eversion, or ectropion, may occur where the endocervical canal is rolled out. Abnormal finding: The cervical surface should not be reddened or granular, which may indicate a lesion

When performing the bimanual examination, the nurse notices that the cervix feels smooth and firm, is round, and is fixed in place (does not move). When cervical palpation is performed, the patient complains of some pain. The nurses interpretation of these results should be which of these? a. These findings are all within normal limits. b. Cervical consistency should be soft and velvety not firm. c. The cervix should move when palpated; an immobile cervix may indicate malignancy. d. Pain may occur during palpation of the cervix.

c. The cervix should move when palpated; an immobile cervix may indicate malignancy. Normally, the cervix feels smooth and firm, similar to the consistency of the tip of the nose. It softens and feels velvety at 5 to 6 weeks of pregnancy (Goodell sign). The cervix should be evenly rounded. With a finger on either side, the examiner should be able to move the cervix gently from side to side, and doing so should produce no pain for the patient. Hardness of the cervix may occur with malignancy. Immobility may occur with malignancy, and pain may occur with inflammation or ectopic pregnancy.

A visitor from Poland who does not speak English seems to be somewhat apprehensive about the nurse examining his neck. How should the nurse proceed that would allow the patient to feel more comfortable with the nurse examining his thyroid gland? a. Behind with the nurse's hands placed firmly around his neck b. The side with the nurse's eyes averted toward the ceiling and thumbs on his neck c. The front with the nurse's thumbs placed on either side of his trachea and his head tilted forward d. The front with the nurse's thumbs placed on either side of his trachea and his head tilted backward

c. The front with the nurse's thumbs placed on either side of his trachea and his head tilted forward

Which of the following statements regarding the results obtained from use of the Snellen chart is true? a. The smaller the denominator, the poorer the vision. b. The smaller the numerator, the poorer the vision. c. The larger the denominator, the poorer the vision. d. The larger the numerator, the better the vision.

c. The larger the denominator, the poorer the vision.

Which of the following would be true regarding otoscopic examination of a newborn? a. Immobility of the drum is a normal finding. b. An injected membrane would indicate infection. c. The normal membrane may appear thick and opaque. d. The appearance of the membrane is identical to that of an adult.

c. The normal membrane may appear thick and opaque. During the first few days, the tympanic membrane often looks thickened and opaque. It may look "injected" and have a mild redness from increased vascularity.

29. During a hearing assessment the nurse finds that sound lateralizes to the patient's left ear with the Weber test. What can the nurse conclude from this? a. The patient has a conductive hearing loss in the right ear. b. Lateralization is a normal finding with the Weber test. c. The patient could have either a sensorineural or a conductive loss. d. A mistake has occurred; the test must be repeated.

c. The patient could have either a sensorineural or a conductive loss. It is necessary to perform the Weber and Rinne tests to determine the type of loss. With conductive loss, sound lateralizes to the "poorer" ear owing to background room noise. With sensorineural loss, sound lateralizes to the "better" ear or unaffected ear.

The nurse is examining a patient's ears and notices cerumen in the external canal. Which of the following statements about cerumen is correct? a. Sticky honey-colored cerumen is a sign of infection. b. The presence of cerumen is indicative of poor hygiene. c. The purpose of cerumen is to protect and lubricate the ear. d. Cerumen is necessary for transmitting sound through the auditory canal.

c. The purpose of cerumen is to protect and lubricate the ear. The ear is lined with glands that secrete cerumen, a yellow waxy material that lubricates and protects the ear.

a. A problem exists with the sensory cortex and its ability to discriminate the location. b. The lack of oxygen in his heart has resulted in decreased amount of oxygen to the areas experiencing the pain. c. The sensory cortex does not have the ability to localize pain in the heart; consequently, the pain is felt elsewhere. d. A lesion has developed in the dorsal root, which is preventing the sensation from being transmitted normally.

c. The sensory cortex does not have the ability to localize pain in the heart; consequently, the pain is felt elsewhere. The sensory cortex is arranged in a specific pattern, forming a corresponding map of the body. Pain in the right hand is perceived at a specific spot on the map. Some organs, such as the heart, liver, and spleen, are absent from the brain map. Pain originating in these organs is referred because no felt image exists in which to have pain. Pain is felt by proxy, that is, by another body part that does have a felt image. The other responses are not correct explanations.

During an internal examination of a womans genitalia, the nurse will use which technique for proper insertion of the speculum? a. The woman is instructed to bear down, the speculum blades are opened and applied in a swift, upward movement. b. The blades of the speculum are inserted on a horizontal plane, turning them to a 30-degree angle while continuing to insert them. The woman is asked to bear down after the speculum is inserted. c. The woman is instructed to bear down, the width of the blades are horizontally turned, and the speculum is inserted downward at a 45-degree angle toward the small of the womans back. d. The blades are locked open by turning the thumbscrew. Once the blades are open, pressure is applied to the introitus and the blades are inserted downward at a 45-degree angle to bring the cervix into view.

c. The woman is instructed to bear down, the width of the blades are horizontally turned, and the speculum is inserted downward at a 45-degree angle toward the small of the womans back. The examiner should instruct the woman to bear down, turn the width of the blades horizontally, and insert the speculum at a 45-degree angle downward toward the small of the womans back. (See the text under Speculum Examination for more detail.)

While performing a well-child assessment on a 5 year old, the nurse notes the presence of palpable, bilateral, cervical, and inguinal lymph nodes. They are approximately 0.5 cm in size, round, mobile, and non-tender. What do these findings lead the nurse to conclude? a. The child has chronic allergies. b. The child likely has an infection. c. These are normal findings for a well child of this age. d. These findings indicate a need for additional evaluation.

c. These are normal findings for a well child of this age. NOTE: Small, soft, non-tender, moveable, non-palpable lymph nodes are normal findings. Large, firm, rubbery, tender, fixed are abnormal findings. (Moveable can be abnormal when combined with other abnormal findings).

During an assessment of a 20-year-old Asian patient, the nurse notices that he has dry, flaky cerumen in his canal. What is the significance of this finding? a. This is probably the result of lesions from eczema in his ear. b. This represents poor hygiene. c. This is a normal finding and no further follow-up is necessary. d. This could be indicative of change in cilia; the nurse should assess for conductive hearing loss.

c. This is a normal finding and no further follow-up is necessary. Asians and American Indians are more likely to have dry cerumen, whereas blacks and whites usually have wet cerumen.

While assessing a 7-month-old infant, the nurse makes a loud noise and notices the following response: abduction and flexion of the arms and legs; fanning of the fingers, and curling of the index finger and thumb in a C position, followed by the infant bringing in the arms and legs to the body. What does the nurse know about this response? a. This response could indicate brachial nerve palsy. b. This reaction is an expected startle response at this age. c. This reflex should have disappeared between 1 and 4 months of age. d. This response is normal as long as the movements are bilaterally symmetric.

c. This reflex should have disappeared between 1 and 4 months of age. The Moro reflex is present at birth and usually disappears at 1 to 4 months. Absence of the Moro reflex in the newborn or its persistence after 5 months of age indicates severe central nervous system injury. The other responses are incorrect.

The nurse needs to palpate the temporomandibular joint for crepitation. This joint is located just below the temporal artery and anterior to the: a. Hyoid bone b. Vagus nerve c. Tragus d. Mandible

c. Tragus

A woman states that 2 weeks ago she had a urinary tract infection that was treated with an antibiotic. As a part of the interview, the nurse should ask, Have you noticed any: a. Changes in your urination patterns? b. Excessive vaginal bleeding? c. Unusual vaginal discharge or itching? d. Changes in your desire for intercourse?

c. Unusual vaginal discharge or itching? Several medications may increase the risk of vaginitis. Broad-spectrum antibiotics alter the balance of normal flora, which may lead to the development of vaginitis. The other questions are not appropriate.

When the nurse is interviewing a preadolescent girl, which opening question would be least threatening? a. Do you have any questions about growing up? b. What has your mother told you about growing up? c. When did you notice that your body was changing? d. I remember being very scared when I got my period. How do you think youll feel?

c. When did you notice that your body was changing? Open-ended questions such as, When did you ? rather than Do you ? should be asked. Open-ended questions are less threatening because they imply that the topic is normal and unexceptional.

During a physical examination, a 45-year-old woman states that she has had a crusty, itchy rash on her breast for approximately 2 weeks. In trying to find the cause of the rash, which question would be important for the nurse to ask? a. Is the rash raised and red? b. Does it appear to be cyclic? c. Where did the rash first appear on the nipple, the areola, or the surrounding skin? d. What was she doing when she first noticed the rash, and do her actions make it worse?

c. Where did the rash first appear on the nipple, the areola, or the surrounding skin? The location where the rash first appeared is important for the nurse to determine. Paget disease starts with a small crust on the nipple apex and then spreads to the areola. Eczema or other dermatitis rarely starts at the nipple unless it is a result of breastfeeding. It usually starts on the areola or surrounding skin and then spreads to the nipple.

During the physical examination, the nurse notices that a female patient has an inverted left nipple. Which statement regarding this is most accurate? a. Normal nipple inversion is usually bilateral. b. Unilateral inversion of a nipple is always a serious sign. c. Whether the inversion is a recent change should be determined. d. Nipple inversion is not significant unless accompanied by an underlying palpable mass.

c. Whether the inversion is a recent change should be determined. The nurse should distinguish between a recently retracted nipple from one that has been inverted for many years or since puberty. Normal nipple inversion may be unilateral or bilateral and usually can be pulled out; that is, if it is not fixed. Recent nipple retraction signifies acquired disease.

A 46-year-old woman is in the clinic for her annual gynecologic examination. She voices a concern about ovarian cancer because her mother and sister died of it. Which statement does the nurse know to be correct regarding ovarian cancer? a. Ovarian cancer rarely has any symptoms. b. The Pap smear detects the presence of ovarian cancer. c. Women at high risk for ovarian cancer should have annual transvaginal ultrasonography for screening. d. Women over age 40 years should have a thorough pelvic examination every 3 years.

c. Women at high risk for ovarian cancer should have annual transvaginal ultrasonography for screening. With ovarian cancer, the patient may have abdominal pain, pelvic pain, increased abdominal size, bloating, and nonspecific gastrointestinal symptoms; or she may be asymptomatic. The Pap smear does not detect the presence of ovarian cancer. Annual transvaginal ultrasonography may detect ovarian cancer at an earlier stage in women who are at high risk for developing it.

Which statement would be most appropriate when the nurse is introducing the topic of sexual relationships during an interview? a. Now, it is time to talk about your sexual history. When did you first have intercourse? b. Women often feel dissatisfied with their sexual relationships. Would it be okay to discuss this now? c. Women often have questions about their sexual relationship and how it affects their health. Do you have any questions? d. Most women your age have had more than one sexual partner. How many would you say you have had?

c. Women often have questions about their sexual relationship and how it affects their health. Do you have any questions? The nurse should begin with an open-ended question to assess individual needs. The nurse should include appropriate questions as a routine part of the health history, because doing so communicates that the nurse accepts the individuals sexual activity and believes it is important. The nurses comfort with the discussion prompts the patients interest and, possibly, relief that the topic has been introduced. The initial discussion establishes a database for comparison with any future sexual activities and provides an opportunity to screen sexual problems.

A patient comes to the clinic complaining of neck and shoulder pain and is unable to turn her head. Which nerve does the nurse suspect is damaged and how should the nurse proceed with the examination? a. XII; assess for a positive Romberg sign. b. XI; palpate the anterior and posterior triangles. c. XI; have patient shrug their shoulders against resistance. d. XII; percuss the sternomastoid and submandibular neck muscles.

c. XI; have patient shrug their shoulders against resistance.

A 14-year-old boy who has been diagnosed with Osgood-Schlatter disease reports painful swelling just below the knee for the past 5 months. Which response by the nurse is appropriate? a. If these symptoms persist, you may need arthroscopic surgery. b. You are experiencing degeneration of your knee, which may not resolve. c. Your disease is due to repeated stress on the patellar tendon. It is usually self-limited, and your symptoms should resolve with rest. d. Increasing your activity and performing knee-strengthening exercises will help decrease the inflammation and maintain mobility in the knee.

c. Your disease is due to repeated stress on the patellar tendon. It is usually self-limited, and your symptoms should resolve with rest. Osgood-Schlatter disease is a painful swelling of the tibial tubercle just below the knee and most likely due to repeated stress on the patellar tendon. It is usually self-limited, occurring during rapid growth and most often in boys. The symptoms resolve with rest. The other responses are not appropriate.

A patient has been shown to have a sensorineural hearing loss. During the assessment, it would be important for the nurse to: a. speak loudly so he can hear the questions. b. assess for middle ear infection as a possible cause. c. ask the patient what medications he is currently taking. d. look for the source of the obstruction in the external ear.

c. ask the patient what medications he is currently taking A simple increase in amplitude may not enable the person to understand words. Sensorineural hearing loss may be caused by presbycusis, a gradual nerve degenera- tion that occurs with aging and by ototoxic drugs, which affect the hair cells in the cochlea.

A patient with a middle ear infection asks the nurse, "What does the middle ear do?" The nurse responds by telling the patient that the middle ear functions to: a. maintain balance. b. interpret sounds as they enter the ear. c. conduct vibrations of sounds to the inner ear. d. increase amplitude of sound for the inner ear to function.

c. conduct vibrations of sounds to the inner ear. Among its other functions, the middle ear conducts sound vibrations from the outer ear to the central hearing apparatus in the inner ear.

The nurse is performing an assessment on a 65-year-old male. He reports a crusty nodule behind the pinna. It bleeds intermittently and has not healed over the past 6 months. On physical assessment, the nurse finds an ulcerated crusted nodule with an indurated base. The preliminary analysis in this situation is that this: a. is most likely a benign sebaceous cyst. b. is most likely a Darwin's tubercle and is not significant. c. could be a potential carcinoma and should be referred. d. is a tophus, which is common in the elderly and is a sign of gout.

c. could be a potential carcinoma and should be referred. An ulcerated crusted nodule with an indurated base that fails to heal is characteristic of a carcinoma. These lesions fail to heal and bleed intermittently. Individuals with such symptoms should be referred for a biopsy.

The labyrinth of the inner ear is responsible for maintaining the body's a. air conduction. b. binaural interaction. c. equilibrium. d. pressure equalization.

c. equilibrium. The labyrinth maintains the body's equilibrium. Binaural interaction is controlled by the brainstem and permits locating the direction of a sound. The normal pathway of hearing is by air conduction. The eustachian tube allows equalization of air pressure on each side of the tympanic membrane.

Narrow palpebral fissures, epicanthal folds, and midfacial hypoplasia are characteristics of a. chronic childhood allergies. b. congenital hypothyroidism. c. fetal alcohol syndrome. d. Down syndrome.

c. fetal alcohol syndrome. Facial characteristics of fetal alcohol syndrome include narrow palpebral fissures, epicanthal folds, and midfacial hypoplasia. Facial characteristics of Down syndrome include upslanting eyes with inner epicanthal folds, flat nasal bridge, small, broad, flat nose, protruding thick tongue, and ear dysplasia. Facial characteristics of chronic allergies include exhausted face, blue shadows below the eyes, double or single crease on the lower eyelids, central facial pallor, open mouth breathing (malocclusion of the teeth and malformed jaw), and a transverse line on the nose. Facial characteristics of congenital hypothyroidism include low hairline, hirsute forehead, swollen eyelids, narrow palpebral fissures, widely spaced eyes, depressed nasal bridge, puffy face, thick tongue protruding through an open mouth, and a dull expression.

The location in the brain where optic nerve fibers from the temporal fields of vision cross over is identified as the a. fovea centralis. b. choroid. c. optic chiasm. d. optic disc.

c. optic chiasm. At the optic chiasm, nasal fibers (from both temporal visual fields) cross over. The fovea centralis is the area of the retina that has the sharpest and keenest vision. The optic disc is the area in which fibers from the retina converge to form the optic nerve. The choroid is the middle vascular layer of the eye, the choroid has dark pigmentation to prevent light from reflecting internally and is heavily vascularized to deliver blood to the retina.

The lens of the eye functions as a a. mediator of light. b. sensory facilitator. c. refracting medium. d. controller of intraocular pressure.

c. refracting medium. The lens serves as a refracting medium, keeping a viewed object in continual focus on the retina. The muscle fibers of the iris function as the mediator of light. The cornea is very sensitive to touch. The intraocular pressure is determined by a balance between the amount of aqueous humor produced and resistance to its outflow at the angle of the anterior chamber.

During an examination, the patient states he is hearing a buzzing sound and says that it is "driving me crazy!" The nurse recognizes that this symptom is: a. vertigo. b. pruritus. c. tinnitus. d. cholesteatoma.

c. tinnitus. Tinnitus is a sound that comes from within a person; it can be a ringing, crackling, or buzzing sound. It accompanies some hearing or ear disorders.

A mother brings her 2-month-old daughter in for an examination and says, "My daughter rolled over against the wall, and now I have noticed that she has this spot that is soft on the top of her head. Is something terribly wrong?" How should the nurse respond? a. "Perhaps that could be a result of your dietary intake during pregnancy." b. "Your baby may have craniosynostosis, a disease of the sutures of the skull." c. "That 'soft spot' may be an indication of cretinism or congenital hypothyroidism." d. "That 'soft spot' is normal, and actually allows for growth of the brain during the first year of your baby's life."

d. "That 'soft spot' is normal, and actually allows for growth of the brain during the first year of your baby's life."

A 60-year-old man has just been told that he has benign prostatic hypertrophy (BPH). He has a friend who just died from cancer of the prostate. He is concerned this will happen to him. How should the nurse respond? a. "The swelling in your prostate is only temporary and will go away." b. "We will treat you with chemotherapy so we can control the cancer." c. "It would be very unusual for a man your age to have cancer of the prostate." d. "The enlargement of your prostate is caused by hormonal changes, and not cancer."

d. "The enlargement of your prostate is caused by hormonal changes, and not cancer." The prostate gland commonly starts to enlarge during the middle adult years. BPH is present in 1 in 10 men at the age of 40 years and increases with age. It is believed that the hypertrophy is caused by hormonal imbalance that leads to the proliferation of benign adenomas. The other responses are not appropriate.

During the assessment of an 18-month-old infant, the mother expresses concern to the nurse about the infant's inability to toilet train. What would be the nurse's best response? a. "Some children are just more difficult to train, so I wouldn't worry about it yet." b. "Have you considered reading any of the books on toilet training? They can be very helpful." c. "This could mean that there is a problem in your baby's development. We'll watch her closely for the next few months." d. "The nerves that will allow your baby to have control over the passing of stools are not developed until at least 18 to 24 months of age."

d. "The nerves that will allow your baby to have control over the passing of stools are not developed until at least 18 to 24 months of age." The infant passes stools by reflex. Voluntary control of the external anal sphincter cannot occur until the nerves supplying the area have become fully myelinated, usually around 1 to 2 years of age. Toilet training usually starts after the age of 2 years.

The nurse is taking the history of a patient who may have a perforated eardrum. What would be an important question in this situation? a. "Do you ever notice ringing or crackling in your ears?" b. "When was the last time you had your hearing checked?" c. "Have you ever been told you have any type of hearing loss?" d. "Was there any relationship between the ear pain and the discharge you mentioned?"

d. "Was there any relationship between the ear pain and the discharge you mentioned?" Typically with perforation, ear pain occurs first, stopping with a popping sensation, and then drainage occurs. A 31-year-old patient tells the nurse that he has noticed pain in his left ear when people speak loudly to him. The nurse knows that this finding: a. is normal for people of that age. b. is a characteristic of recruitment. c. may indicate a middle ear infection. d. indicates that the patient has a cerumen impaction. ANS: b. is a characteristic of recruitment. Recruitment is a marked loss occurring when sound is at low intensity; sound actually may become painful when repeated at a louder volume.

The mother of a 2-year-old is concerned because her son has had three ear infections in the past year. What would be an appropriate response by the nurse? a. "It is unusual for a small child to have frequent ear infections unless there is something else wrong." b. "We need to check the immune system of your son to see why he is having so many ear infections." c. "Ear infections are not uncommon in infants and toddlers because they tend to have more cerumen in the external ear." d. "Your son's eustachian tube is shorter and wider than yours because of his age, which allows for infections to develop more easily."

d. "Your son's eustachian tube is shorter and wider than yours because of his age, which allows for infections to develop more easily." The infant's eustachian tube is relatively shorter and wider, and its position is more horizontal than the adult's, so it is easier for pathogens from the nasopharynx to migrate through to the middle ear.

The nurse is reviewing risk factors for breast cancer. Which of these women have risk factors that place them at a higher risk for breast cancer? a. 37 year old who is slightly overweight b. 42 year old who has had ovarian cancer c. 45 year old who has never been pregnant d. 65 year old whose mother had breast cancer

d. 65 year old whose mother had breast cancer Risk factors for breast cancer include having a first-degree relative with breast cancer (mother, sister, or daughter) and being older than 50 years of age.

Which of these clinical situations would the nurse consider to be outside normal limits? a. A patient has had one pregnancy and states that she believes she may be entering menopause. Her breast examination reveals breasts that are soft and slightly sagging. b. A patient has never been pregnant. Her breast examination reveals large pendulous breasts that have a firm, transverse ridge along the lower quadrant in both breasts. c. A patient has never been pregnant and reports that she should begin her period tomorrow. Her breast examination reveals breast tissue that is nodular and somewhat engorged. She states that the examination was slightly painful. d. A patient has had two pregnancies, and she breastfed both of her children. Her youngest child is now 10 years old. Her breast examination reveals breast tissue that is somewhat soft, and she has a small amount of thick yellow discharge from both nipples.

d. A patient has had two pregnancies, and she breastfed both of her children. Her youngest child is now 10 years old. Her breast examination reveals breast tissue that is somewhat soft, and she has a small amount of thick yellow discharge from both nipples. If any discharge appears, the nurse should note its color and consistency. Except in pregnancy and lactation, any discharge is abnormal. In nulliparous women, normal breast tissue feels firm, smooth, and elastic; after pregnancy, the tissue feels soft and loose. Premenstrual engorgement is normal, and consists of a slight enlargement, tenderness to palpation, and a generalized nodularity. A firm, transverse ridge of compressed tissue in the lower quadrants, known as the inframammary ridge, is especially noticeable in large breasts

A 35-year-old woman is at the clinic for a gynecologic examination. During the examination, she asks the nurse, How often do I need to have this Pap test done? Which reply by the nurse is correct? a. It depends. Do you smoke? b. A Pap test needs to be performed annually until you are 65 years of age. c. If you have two consecutive normal Pap tests, then you can wait 5 years between tests. d. After age 30 years, if you have three consecutive normal Pap tests, then you may be screened every 2 to 3 years.

d. After age 30 years, if you have three consecutive normal Pap tests, then you may be screened every 2 to 3 years. Cervical cancer screening with the Pap test continues annually until age 30 years. After age 21, regardless of sexual history or activity, women should be screened every 3 years until age 30, then every 5 years until age 65.

An assessment of a 23-year-old patient reveals the following: an auricle that is tender and reddish-blue in color with small vesicles. Additional information the nurse would need to know includes which of the following? a. Any change in the ability to hear b. Any recent drainage from the ear c. Recent history of trauma to the ear d. Any prolonged exposure to extreme cold

d. Any prolonged exposure to extreme cold Frostbite causes reddish-blue discoloration and swelling of the auricle after exposure to extreme cold. Vesicles or bullae may develop, and the person feels pain and tenderness.

The nurse notices that a patient's submental lymph nodes are enlarged. In an effort to identify the cause of the node enlargement, what should the nurse assess? a. Infraclavicular area b. Supraclavicular area c. Area distal to the enlarged node d. Area proximal to the enlarged node

d. Area proximal to the enlarged node NOTE: Proximal is "upstream", opposite of distal.

During the taking of the health history of a 78-year-old man, his wife states that he occasionally has problems with short-term memory loss and confusion: He cant even remember how to button his shirt. When assessing his sensory system, which action by the nurse is most appropriate? a. The nurse would not test the sensory system as part of the examination because the results would not be valid. b. The nurse would perform the tests, knowing that mental status does not affect sensory ability. c. The nurse would proceed with an explanation of each test, making certain that the wife understands. d. Before testing, the nurse would assess the patients mental status and ability to follow directions.

d. Before testing, the nurse would assess the patients mental status and ability to follow directions. The nurse should ensure the validity of the sensory system testing by making certain that the patient is alert, cooperative, comfortable, and has an adequate attention span. Otherwise, the nurse may obtain misleading and invalid results.

When reviewing the musculoskeletal system, the nurse recalls that hematopoiesis takes place in the: a. Liver. b. Spleen. c. Kidneys. d. Bone marrow.

d. Bone marrow. The musculoskeletal system functions to encase and protect the inner vital organs, to support the body, to produce red blood cells in the bone marrow (hematopoiesis), and to store minerals. The other options are not correct.

A 32-year-old woman tells the nurse that she has noticed very sudden, jerky movements mainly in her hands and arms. She says, They seem to come and go, primarily when I am trying to do something. I havent noticed them when Im sleeping. This description suggests: a. Tics. b. Athetosis. c. Myoclonus. d. Chorea.

d. Chorea. Chorea is characterized by sudden, rapid, jerky, purposeless movements that involve the limbs, trunk, or face. Chorea occurs at irregular intervals, and the movements are all accentuated by voluntary actions.

The nurse notices that a woman in an exercise class is unable to jump rope. The nurse is aware that to jump rope, ones shoulder has to be capable of: a. Inversion. b. Supination. c. Protraction. d. Circumduction.

d. Circumduction. Circumduction is defined as moving the arm in a circle around the shoulder. The other options are not correct.

During an admission assessment, the nurse notices that a male patient has an enlarged and rather thick skull. The nurse suspects acromegaly. What additional finding would the nurse assess for to confirm this suspicion? a. Exophthalmos b. Bowed long bones c. Acorn-shaped cranium d. Coarse facial features

d. Coarse facial features

A 50-year-old woman is in the clinic for weakness in her left arm and leg that she has noticed for the past week. The nurse should perform which type of neurologic examination? a. Glasgow Coma Scale b. Neurologic recheck examination c. Screening neurologic examination d. Complete neurologic examination

d. Complete neurologic examination The nurse should perform a complete neurologic examination on an individual who has neurologic concerns (e.g., headache, weakness, loss of coordination) or who is showing signs of neurologic dysfunction. The Glasgow Coma Scale is used to define a persons level of consciousness. The neurologic recheck examination is appropriate for those who are demonstrating neurologic deficits. The screening neurologic examination is performed on seemingly well individuals who have no significant subjective findings from the health history.

The nurse is performing a genital examination on a male patient and notices urethral drainage. When collecting urethral discharge for microscopic examination and culture, the nurse should: a. Ask the patient to urinate into a sterile cup. b. Ask the patient to obtain a specimen of semen. c. Insert a cotton-tipped applicator into the urethra. d. Compress the glans between the examiners thumb and forefinger, and collect any discharge.

d. Compress the glans between the examiners thumb and forefinger, and collect any discharge. If urethral discharge is noticed, then the examiner should collect a smear for microscopic examination and culture by compressing the glans anteroposteriorly between the thumb and forefinger. The other options are no correct actions.

While the nurse is taking the history of a 68-year-old patient who sustained a head injury 3 days earlier, he tells the nurse that he is on a cruise ship and is 30 years old. The nurse knows that this finding is indicative of a(n): a. Great sense of humor. b. Uncooperative behavior. c. Inability to understand questions. d. Decreased level of consciousness

d. Decreased level of consciousness A change in consciousness may be subtle. The nurse should notice any decreasing level of consciousness, disorientation, memory loss, uncooperative behavior, or even complacency in a previously combative person. The other responses are incorrect.

acid. A young swimmer comes to the sports clinic complaining of a very sore shoulder. He was running at the pool, slipped on some wet concrete, and tried to catch himself with his outstretched hand. He landed on his outstretched hand and has not been able to move his shoulder since. The nurse suspects: a. Joint effusion. b. Tear of rotator cuff. c. Adhesive capsulitis. d. Dislocated shoulder

d. Dislocated shoulder A dislocated shoulder occurs with trauma involving abduction, extension, and external rotation (e.g., falling on an outstretched arm or diving into a pool).

In an individual with otitis externa, which of the following signs would the nurse expect to find on assessment? a. Rhinorrhea b. Periorbital edema c. Pain over the maxillary sinuses d. Enlarged superficial cervical nodes

d. Enlarged superficial cervical nodes The lymphatic drainage of the external ear flows to the parotid, mastoid, and superficial cervical nodes. The signs are severe swelling of the canal, inflammation, and tenderness.

A patient visits the clinic because he has recently noticed that the left side of his mouth is paralyzed. He states that he cannot whistle but the nurse notes he can still raise his eyebrows. What does the nurse suspect? a. Bell palsy b. Cushing syndrome c. Parkinson syndrome d. Experienced a cerebrovascular accident (CVA) or stroke

d. Experienced a cerebrovascular accident (CVA) or stroke NOTE: Lesions that damage the motor cortex, such as strokes, result in contralateral facial weakness of the lower face only. Lesions that damage the facial nerve in the brainstem or after it exits the brainstem, such as with Bell's Palsy, result in ipsilateral facial weakness involving both the upper and lower face.

A 62-year-old man states that his physician told him that he has an inguinal hernia. He asks the nurse to explain what a hernia is. The nurse should: a. Tell him not to worry and that most men his age develop hernias. b. Explain that a hernia is often the result of prenatal growth abnormalities. c. Refer him to his physician for additional consultation because the physician made the initial diagnosis. d. Explain that a hernia is a loop of bowel protruding through a weak spot in the abdominal muscles.

d. Explain that a hernia is a loop of bowel protruding through a weak spot in the abdominal muscles. A hernia is a loop of bowel protruding through a weak spot in the musculature. The other options are not correct responses to the patients question.

During the taking of a health history, the patient states, It really hurts back there, and sometimes it itches, too. I have even seen blood on the tissue when I have a bowel movement. Is there something there? The nurse should expect to see which of these upon examination of the anus? a. Rectal prolapse b. Internal hemorrhoid c. External hemorrhoid that has resolved d. External hemorrhoid that is thrombosed

d. External hemorrhoid that is thrombosed These symptoms are consistent with an external hemorrhoid. An external hemorrhoid, when thrombosed, contains clotted blood and becomes a painful, swollen, shiny blue mass that itches and bleeds with defecation. When the external hemorrhoid resolves, it leaves a flabby, painless skin sac around the anal orifice. An internal hemorrhoid is not palpable but may appear as a red mucosal mass when the person performs a Valsalva maneuver. A rectal prolapse appears as a moist, red doughnut with radiating lines.

Which of the following behaviors demonstrated by an individual may be indicative of hearing loss? a. Not looking at the examiner when being questioned b. Talking in a high pitched voice c. Speaking slowly with well articulated consonants d. Frequently asking for the question to be repeated

d. Frequently asking for the question to be repeated Hearing loss is indicated when a person frequently asks to have statements repeated. Hearing loss is indicated when a person lip reads or watches faces and lips closely. Hearing loss is indicated when a person has a flat, monotonous tone of voice. Hearing loss is indicated when speech sounds are garbled, vowel sounds are distorted, and the person uses an inappropriately loud voice.

When performing a genital assessment on a middle-aged man, the nurse notices multiple soft, moist, painless papules in the shape of cauliflower-like patches scattered across the shaft of the penis. These lesions are characteristic of: a. Carcinoma. b. Syphilitic chancres. c. Genital herpes. d. Genital warts.

d. Genital warts. The lesions of genital warts are soft, pointed, moist, fleshy, painless papules that may be single or multiple in a cauliflower-like patch. They occur on the shaft of the penis, behind the corona, or around the anus, where they may grow into large grapelike clusters.

The nurse is providing patient education for a man who has been diagnosed with a rotator cuff injury. The nurse knows that a rotator cuff injury involves the: a. Nucleus pulposus. b. Articular processes. c. Medial epicondyle. d. Glenohumeral joint.

d. Glenohumeral joint. A rotator cuff injury involves the glenohumeral joint, which is enclosed by a group of four powerful muscles and tendons that support and stabilize it. The nucleus pulposus is located in the center of each intervertebral disk. The articular processes are projections in each vertebral disk that lock onto the next vertebra, thereby stabilizing the spinal column. The medial epicondyle is located at the elbow.

A woman who is 8 weeks pregnant is in the clinic for a checkup. The nurse reads on her chart that her cervix is softened and looks cyanotic. The nurse knows that the woman is exhibiting _______ sign and ________ sign. a. Tanner; Hegar b. Hegar; Goodell c. Chadwick; Hegar d. Goodell; Chadwick

d. Goodell; Chadwick Shortly after the first missed menstrual period, the female genitalia show signs of the growing fetus. The cervix softens (Goodell sign) at 4 to 6 weeks, and the vaginal mucosa and cervix look cyanotic (Chadwick sign) at 8 to 12 weeks. These changes occur because of increased vascularity and edema of the cervix and hypertrophy and hyperplasia of the cervical glands. Hegar sign occurs when the isthmus of the uterus softens at 6 to 8 weeks. Tanner sign is not a correct response.

When the nurse is performing a genital examination on a male patient, which action is correct? a. Auscultating for the presence of a bruit over the scrotum b. Palpating for the vertical chain of lymph nodes along the groin, inferior to the inguinal ligament c. Palpating the inguinal canal only if a bulge is present in the inguinal region during inspection d. Having the patient shift his weight onto the left (unexamined) leg when palpating for a hernia on the right side

d. Having the patient shift his weight onto the left (unexamined) leg when palpating for a hernia on the right side When palpating for the presence of a hernia on the right side, the male patient is asked to shift his weight onto the left (unexamined) leg. Auscultating for a bruit over the scrotum is not appropriate. When palpating for lymph nodes, the horizontal chain is palpated. The inguinal canal should be palpated whether a bulge is present or not.

The nurse is explaining to a student nurse the four areas in the body where lymph nodes are accessible. Which areas should the nurse include in her explanation to the student? a. Head, breasts, groin, and abdomen b. Arms, breasts, inguinal area, and legs c. Head and neck, arms, breasts, and axillae d. Head and neck, arms, inguinal area, and axillae

d. Head and neck, arms, inguinal area, and axillae

A 22-year-old woman is being seen at the clinic for problems with vulvar pain, dysuria, and fever. On physical examination, the nurse notices clusters of small, shallow vesicles with surrounding erythema on the labia. Inguinal lymphadenopathy present is also present. The most likely cause of these lesions is: a. Pediculosis pubis. b. Contact dermatitis. c. HPV. d. Herpes simplex virus type 2.

d. Herpes simplex virus type 2. Herpes simplex virus type 2 exhibits clusters of small, shallow vesicles with surrounding erythema that erupt on the genital areas. Inguinal lymphadenopathy is also present. The woman reports local pain, dysuria, and fever.

During a health history, a patient tells the nurse that he has trouble in starting his urine stream. This problem is known as: a. Urgency. b. Dribbling. c. Frequency d. Hesitancy

d. Hesitancy Hesitancy is trouble in starting the urine stream. Urgency is the feeling that one cannot wait to urinate. Dribbling is the last of the urine before or after the main act of urination. Frequency is urinating more often than usual.

The nurse is examining a 2-month-old infant and notices asymmetry of the infants gluteal folds. The nurse should assess for other signs of what disorder? a. Fractured clavicle b. Down syndrome c. Spina bifida d. Hip dislocation

d. Hip dislocation Unequal gluteal folds may accompany hip dislocation after 2 to 3 months of age, but some asymmetry may occur in healthy children. Further assessment is needed. The other responses are not correct.

In assessing a 70-year-old patient who has had a recent cerebrovascular accident, the nurse notices right- sided weakness. What might the nurse expect to find when testing his reflexes on the right side? a. Lack of reflexes b. Normal reflexes c. Diminished reflexes d. Hyperactive reflexes

d. Hyperactive reflexes Hyperreflexia is the exaggerated reflex observed when the monosynaptic reflex arc is released from the influence of higher cortical levels. This response occurs with upper motor neuron lesions (e.g., a cerebrovascular accident). The other responses are incorrect.

A 9-year-old girl is in the clinic for a sport physical examination. After some initial shyness she finally asks, Am I normal? I dont seem to need a bra yet, but I have some friends who do. What if I never get breasts? The nurses best response would be: a. Dont worry, you still have plenty of time to develop. b. I know just how you feel, I was a late bloomer myself. Just be patient, and they will grow. c. You will probably get your periods before you notice any significant growth in your breasts. d. I understand that it is hard to feel different from your friends. Breasts usually develop between 8 and 10 years of age.

d. I understand that it is hard to feel different from your friends. Breasts usually develop between 8 and 10 years of age. Adolescent breast development usually begins between 8 and 10 years of age. The nurse should not belittle the girls feelings by using statements like dont worry or by sharing personal experiences. The beginning of breast development precedes menarche by approximately 2 years.

A 22-year-old woman has been considering using oral contraceptives. As a part of her health history, the nurse should ask: a. Do you have a history of heart murmurs? b. Will you be in a monogamous relationship? c. Have you carefully thought this choice through? d. If you smoke, how many cigarettes do you smoke per day?

d. If you smoke, how many cigarettes do you smoke per day? Oral contraceptives, together with cigarette smoking, increase the risk for cardiovascular side effects. If cigarettes are used, then the nurse should assess the patients smoking history. The other questions are not appropriate.

When performing a genitourinary assessment on a 16-year-old male adolescent, the nurse notices a swelling in the scrotum that increases with increased intra-abdominal pressure and decreases when he is lying down. The patient complains of pain when straining. The nurse knows that this description is most consistent with a(n) hernia. a. Femoral b. Incisional c. Direct inguinal d. Indirect inguinal

d. Indirect inguinal With indirect inguinal hernias, pain occurs with straining and a soft swelling increases with increased intra- abdominal pressure, which may decrease when the patient lies down. These findings do not describe the other hernias.

A 13-year-old girl is visiting the clinic for a sports physical examination. The nurse should remember to include which of these tests in the examination? a. Testing for occult blood b. Valsalva maneuver c. Internal palpation of the anus d. Inspection of the perianal area

d. Inspection of the perianal area The perianal region of the school-aged child and adolescent should be inspected during the examination of the genitalia. Internal palpation is not routinely performed at this age. Testing for occult blood and performing the Valsalva maneuver are also not necessary.

A 13-year-old girl is visiting the clinic for a sports physical examination. The nurse should remember to include which of these tests in the examination? a. Testing for occult blood b. Valsalva maneuver c. Internal palpation of the anus d. Inspection of the perianal area

d. Inspection of the perianal area The perianal region of the school-aged child and adolescent should be inspected during the examination of the genitalia. Internal palpation is not routinely performed at this age. Testing for occult blood and performing the Valsalva maneuver are also not necessary.

The nurse is explaining to a patient that there are shock absorbers in his back to cushion the spine and to help it move. The nurse is referring to his: a. Vertebral column. b. Nucleus pulposus. c. Vertebral foramen. d. Intervertebral disks.

d. Intervertebral disks. Intervertebral disks are elastic fibrocartilaginous plates that cushion the spine similar to shock absorbers and help it move. The vertebral column is the spinal column itself. The nucleus pulposus is located in the center of each disk. The vertebral foramen is the channel, or opening, for the spinal cord in the vertebrae.

During an assessment of a 32-year-old patient with a recent head injury, the nurse notices that the patient responds to pain by extending, adducting, and internally rotating his arms. His palms pronate, and his lower extremities extend with plantar flexion. Which statement concerning these findings is most accurate? This patients response: a. Indicates a lesion of the cerebral cortex. b. Indicates a completely nonfunctional brainstem. c. Is normal and will go away in 24 to 48 hours. d. Is a very ominous sign and may indicate brainstem injury.

d. Is a very ominous sign and may indicate brainstem injury. These findings are all indicative of decerebrate rigidity, which is a very ominous condition and may indicate a brainstem injury.

Which statement concerning the anal canal is true? The anal canal: a. Is approximately 2 cm long in the adult. b. Slants backward toward the sacrum. c. Contains hair and sebaceous glands. d. Is the outlet for the gastrointestinal tract.

d. Is the outlet for the gastrointestinal tract. The anal canal is the outlet for the gastrointestinal tract and is approximately 3.8 cm long in the adult. It is lined with a modified skin that does not contain hair or sebaceous glands, and it slants forward toward the umbilicus.

Which of the following statements concerning the eustachian tube is true? a. It is responsible for the production of cerumen. b. It remains open except when swallowing or yawning. c. It allows passage of air between the middle and outer ear. d. It helps equalize air pressure on both sides of the tympanic membrane.

d. It helps equalize air pressure on both sides of the tympanic membrane. The eustachian tube allows equalization of air pressure on each side of the tympanic membrane so that the membrane does not rupture (e.g., during altitude changes in an airplane). The tube is normally closed, but it opens with swallowing or yawning.

Fibrous bands running directly from one bone to another that strengthen the joint and help prevent movement in undesirable directions are called: a. Bursa. b. Tendons. c. Cartilage. d. Ligaments.

d. Ligaments. Fibrous bands running directly from one bone to another that strengthen the joint and help prevent movement in undesirable directions are called ligaments. The other options are not correct

A professional tennis player comes into the clinic complaining of a sore elbow. The nurse will assess for tenderness at the: a. Olecranon bursa. b. Annular ligament. c. Base of the radius. d. Medial and lateral epicondyle.

d. Medial and lateral epicondyle. The epicondyles, the head of the radius, and the tendons are common sites of inflammation and local tenderness, commonly referred to as tennis elbow. The other locations are not affected.

A 19-year-old college student is brought to the emergency department with a severe headache he describes as, "Like nothing I've ever had before." His temperature is 40° C, and he has a stiff neck. The nurse looks for other signs and symptoms of which problem? a. Head injury b. Cluster headache c. Migraine headache d. Meningeal inflammation

d. Meningeal inflammation

The nurse is testing the function of CN XI. Which statement best describes the response the nurse should expect if this nerve is intact? The patient: a. Demonstrates the ability to hear normal conversation. b. Sticks out the tongue midline without tremors or deviation. c. Follows an object with his or her eyes without nystagmus or strabismus. d. Moves the head and shoulders against resistance with equal strength

d. Moves the head and shoulders against resistance with equal strength The following normal findings are expected when testing the spinal accessory nerve (CN XI): The patients sternomastoid and trapezius muscles are equal in size; the person can forcibly rotate the head both ways against resistance applied to the side of the chin with equal strength; and the patient can shrug the shoulders against resistance with equal strength on both sides. Checking the patients ability to hear normal conversation checks the function of CN VIII. Having the patient stick out the tongue checks the function of CN XII. Testing the eyes for nystagmus or strabismus is performed to check CNs III, IV, and VI.

The nurse is examining a 3-month-old infant. While the nurse holds his or her thumbs on the infants inner mid thighs and the fingers on the outside of the infants hips, touching the greater trochanter, the nurse adducts the legs until the his or her thumbs touch and then abducts the legs until the infants knees touch the table. The nurse does not notice any clunking sounds and is confident to record a: a. Positive Allis test. b. Negative Allis test. c. Positive Ortolani sign. d. Negative Ortolani sign.

d. Negative Ortolani sign. Normally, this maneuver feels smooth and has no sound. With a positive Ortolani sign, however, the nurse will feel and hear a clunk, as the head of the femur pops back into place. A positive Ortolani sign also reflects hip instability. The Allis test also tests for hip dislocation but is performed by comparing leg lengths.

When the nurse is conducting sexual history from a male adolescent, which statement would be most appropriate to use at the beginning of the interview? a. Do you use condoms? b. You dont masturbate, do you? c. Have you had sex in the last 6 months? d. Often adolescents your age have questions about sexual activity.

d. Often adolescents your age have questions about sexual activity. The interview should begin with a permission statement, which conveys that it is normal and acceptable to think or feel a certain way. Sounding judgmental should be avoided.

What disease is characterized by a flat, expressionless, or masklike face, a staring gaze, oily skin, and elevated eyebrows? a. Cushing syndrome b. Scleroderma c. Acromegaly d. Parkinson disease

d. Parkinson disease Facial characteristics of Parkinson disease include a flat, expressionless face that is masklike with elevated eyebrows, a staring gaze, oily skin, and drooling. Facial characteristics of acromegaly include an elongated head, a massive face, a prominent nose and lower jaw, a heavy eyebrow ridge, and coarse facial features. Facial characteristics of scleroderma include hard, shiny skin on the forehead and cheeks, thin, pursed lips with radial furrowing, absent skinfolds, muscle atrophy of the face and neck, and absence of expression. Facial characteristics of Cushing syndrome include a plethoric, rounded, "moonlike" face, prominent jowls, red cheeks, and hirsutism on the upper lip, lower cheeks, and chin.

A 25-year-old woman comes to the emergency department with a sudden fever of 38.3 C and abdominal pain. Upon examination, the nurse notices that she has rigid, boardlike lower abdominal musculature. When the nurse tries to perform a vaginal examination, the patient has severe pain when the uterus and cervix are moved. The nurse knows that these signs and symptoms are suggestive of: a. Endometriosis. b. Uterine fibroids. c. Ectopic pregnancy. d. Pelvic inflammatory disease.

d. Pelvic inflammatory disease. These signs and symptoms are suggestive of acute pelvic inflammatory disease, also known as acute salpingitis.

During an assessment of a 20-year-old man, the nurse finds a small palpable lesion with a tuft of hair located directly over the coccyx. The nurse knows that this lesion would most likely be a: a. Rectal polyp. b. Pruritus ani. c. Carcinoma. d. Pilonidal cyst

d. Pilonidal cyst A pilonidal cyst or sinus is a hair-containing cyst or sinus located in the midline over the coccyx or lower sacrum. It often opens as a dimple with a visible tuft of hair and, possibly, an erythematous halo.

During an assessment of a 20-year-old man, the nurse finds a small palpable lesion with a tuft of hair located directly over the coccyx. The nurse knows that this lesion would most likely be a: a. Rectal polyp. b. Pruritus ani. c. Carcinoma. d. Pilonidal cyst.

d. Pilonidal cyst. A pilonidal cyst or sinus is a hair-containing cyst or sinus located in the midline over the coccyx or lower sacrum. It often opens as a dimple with a visible tuft of hair and, possibly, an erythematous halo. (See Table 25-1 for more information, and also for the description of a pruritus ani. .)

When the nurse asks a 68-year-old patient to stand with his feet together and arms at his side with his eyes closed, he starts to sway and moves his feet farther apart. The nurse would document this finding as: a. Ataxia. b. Lack of coordination. c. Negative Homans sign. d. Positive Romberg sign.

d. Positive Romberg sign. Abnormal findings for the Romberg test include swaying, falling, and a widening base of the feet to avoid falling. A positive Romberg sign is a loss of balance that is increased by the closing of the eyes. Ataxia is an uncoordinated or unsteady gait. Homans sign is used to test the legs for deep-vein thrombosis.

The nurse knows that a common assessment finding in a boy younger than 2 years old is: a. Inflamed and tender spermatic cord. b. Presence of a hernia in the scrotum. c. Penis that looks large in relation to the scrotum. d. Presence of a hydrocele, or fluid in the scrotum.

d. Presence of a hydrocele, or fluid in the scrotum. A common scrotal finding in boys younger than 2 years of age is a hydrocele, or fluid in the scrotum. The othe options are not correct.

The nurse is providing patient teaching about an erectile dysfunction drug. One of the drugs potential side effects is prolonged, painful erection of the penis without sexual stimulation, which is known as: a. Orchitis. b. Stricture. c. Phimosis. d. Priapism.

d. Priapism. Priapism is prolonged, painful erection of the penis without sexual desire. Orchitis is inflammation of the testes. Stricture is a narrowing of the opening of the urethral meatus. Phimosis is the inability to retract the foreskin.

When the nurse is performing a genital examination on a male patient, the patient has an erection. The nurses most appropriate action or response is to: a. Ask the patient if he would like someone else to examine him. b. Continue with the examination as though nothing has happened. c. Stop the examination, leave the room while stating that the examination will resume at a later time. d. Reassure the patient that this is a normal response and continue with the examination.

d. Reassure the patient that this is a normal response and continue with the examination. When the male patient has an erection, the nurse should reassure the patient that this is a normal physiologic response to touch and proceed with the rest of the examination. The other responses are not correct and may be perceived as judgmental.

During an examination, the nurse asks the patient to perform the Valsalva maneuver and notices that the patient has a moist, red, doughnut-shaped protrusion from the anus. The nurse knows that this finding is consistent with a: a. Rectal polyp. b. Hemorrhoid. c. Rectal fissure. d. Rectal prolapse.

d. Rectal prolapse. In rectal prolapse, the rectal mucous membrane protrudes through the anus, appearing as a moist red doughnut with radiating lines. It occurs after a Valsalva maneuver, such as straining at passing stool or with exercising

During an examination, the nurse asks the patient to perform the Valsalva maneuver and notices that the patient has a moist, red, doughnut-shaped protrusion from the anus. The nurse knows that this finding is consistent with a: a. Rectal polyp. b. Hemorrhoid. c. Rectal fissure. d. Rectal prolapse.

d. Rectal prolapse. In rectal prolapse, the rectal mucous membrane protrudes through the anus, appearing as a moist red doughnut with radiating lines. It occurs after a Valsalva maneuver, such as straining at passing stool or with exercising.

While performing a rectal examination, the nurse notices a firm, irregularly shaped mass. What should the nurse do next? a. Continue with the examination, and document the finding in the chart. b. Instruct the patient to return for a repeat assessment in 1 month. c. Tell the patient that a mass was felt, but it is nothing to worry about. d. Report the finding, and refer the patient to a specialist for further examination.

d. Report the finding, and refer the patient to a specialist for further examination. A firm or hard mass with an irregular shape or rolled edges may signify carcinoma. Any mass that is discovered should be promptly reported for further examination.

While performing a rectal examination, the nurse notices a firm, irregularly shaped mass. What should the nurse do next? a. Continue with the examination, and document the finding in the chart. b. Instruct the patient to return for a repeat assessment in 1 month. c. Tell the patient that a mass was felt, but it is nothing to worry about. d. Report the finding, and refer the patient to a specialist for further examination.

d. Report the finding, and refer the patient to a specialist for further examination. A firm or hard mass with an irregular shape or rolled edges may signify carcinoma. Any mass that is discovered should be promptly reported for further examination. The other responses are not correct.

Which of the following is a risk factor for ear infections in young children? a. Family history b. Air conditioning c. Excessive cerumen d. Secondhand cigarette smoke

d. Secondhand cigarette smoke Passive or second hand smoke is a risk factor for ear infections.

The nurse is performing a breast examination. Which of these statements best describes the correct procedure to use when screening for nipple and skin retraction during a breast examination? Have the woman: a. Bend over and touch her toes. b. Lie down on her left side and notice any retraction. c. Shift from a supine position to a standing position, and note any lag or retraction. d. Slowly lift her arms above her head, and note any retraction or lag in movement.

d. Slowly lift her arms above her head, and note any retraction or lag in movement. The woman should be directed to change position while checking the breasts for signs of skin retraction. Initially, she should be asked to lift her arms slowly over her head. Both breasts should move up symmetrically. Retraction signs are due to fibrosis in the breast tissue, usually caused by growing neoplasms. The nurse should notice whether movement of one breast is lagging.

The nurse is examining the glans and knows which finding is normal for this area? a. The meatus may have a slight discharge when the glans is compressed. b. Hair is without pest inhabitants. c. The skin is wrinkled and without lesions. d. Smegma may be present under the foreskin of an uncircumcised male.

d. Smegma may be present under the foreskin of an uncircumcised male. The glans looks smooth and without lesions and does not have hair. The meatus should not have any discharge when the glans is compressed. Some cheesy smegma may have collected under the foreskin of an uncircumcised male.

A 78-year-old man has a history of a cerebrovascular accident. The nurse notes that when he walks, his left arm is immobile against the body with flexion of the shoulder, elbow, wrist, and fingers and adduction of the shoulder. His left leg is stiff and extended and circumducts with each step. What type of gait disturbance is this individual experiencing? a. Scissors gait b. Cerebellar ataxia c. Parkinsonian gait d. Spastic hemiparesis

d. Spastic hemiparesis With spastic hemiparesis, the arm is immobile against the body. Flexion of the shoulder, elbow, wrist, and fingers occurs, and adduction of the shoulder, which does not swing freely, is observed. The leg is stiff and extended and circumducts with each step. Causes of this type of gait include cerebrovascular accident.

During ocular examinations, the nurse keeps in mind that movement of the extraocular muscles is: a. decreased int he older adult b impaired in a patient with cataracts c. Stimulated by cranial nerves (CNs) I and II d. Stimulated by CNs III, IV, and VI

d. Stimulated by CNs III, IV, and VI Movement of the extraocular muscles is stimulated by three CNs: III, IV, and VI. THINK: CNs III, IV, and VI help the eyes do tricks

A 68-year-old woman has come in for an assessment of her rheumatoid arthritis, and the nurse notices raised, firm, nontender nodules at the olecranon bursa and along the ulna. These nodules are most commonly diagnosed as: a. Epicondylitis. b. Gouty arthritis. c. Olecranon bursitis. d. Subcutaneous nodules

d. Subcutaneous nodules Subcutaneous nodules are raised, firm, and nontender and occur with rheumatoid arthritis in the olecranon bursa and along the extensor surface of the ulna.

The nurse has completed the musculoskeletal examination of a patients knee and has found a positive bulge sign. The nurse interprets this finding to indicate: a. Irregular bony margins. b. Soft-tissue swelling in the joint. c. Swelling from fluid in the epicondyle. d. Swelling from fluid in the suprapatellar pouch.

d. Swelling from fluid in the suprapatellar pouch. A positive bulge sign confirms the presence of swelling caused by fluid in the suprapatellar pouch. The other options are not correct.

During an examination, the nurse finds that a patient's left temporal artery is tortuous and feels hardened and tender, compared with the right temporal artery. The nurse suspects which condition? a. Bell palsy b. Crepitation c. Mastoiditis d. Temporal arteritis

d. Temporal arteritis

Which of these statements is true regarding the penis? a. The urethral meatus is located on the ventral side of the penis. b. The prepuce is the fold of foreskin covering the shaft of the penis. c. The penis is made up of two cylindrical columns of erectile tissue. d. The corpus spongiosum expands into a cone of erectile tissue called the glans.

d. The corpus spongiosum expands into a cone of erectile tissue called the glans. At the distal end of the shaft, the corpus spongiosum expands into a cone of erectile tissue, the glans. The penis is made up of three cylindrical columns of erectile tissue. The skin that covers the glans of the penis is the prepuce. The urethral meatus forms at the tip of the glans.

Which of the following is an expected response on the cover test? a. The uncovered eye is unable to maintain its gaze on a fixed object. b. The covered eye jumps to reestablish fixation when it is uncovered. c. The covered eye moves into a relaxed position. d. The covered eye maintains its position when uncovered.

d. The covered eye maintains its position when uncovered. A normal response to the cover test is a steady fixed gaze. If muscle weakness is present, the covered eye will drift into a relaxed position. A normal response to the cover test is a steady fixed gaze. When the eye is uncovered, if it jumps to reestablish fixation, eye muscle weakness exists.

Which statement concerning the testes is true? a. The lymphatic vessels of the testes drain into the abdominal lymph nodes. b. The vas deferens is located along the inferior portion of each testis. c. The right testis is lower than the left because the right spermatic cord is longer. d. The cremaster muscle contracts in response to cold and draws the testicles closer to the body.

d. The cremaster muscle contracts in response to cold and draws the testicles closer to the body. When it is cold, the cremaster muscle contracts, which raises the scrotal sac and brings the testes closer to the body to absorb heat necessary for sperm viability. The lymphatic vessels of the testes drain into the inguinal lymph nodes. The vas deferens is located along the upper portion of each testis. The left testis is lower than the right because the left spermatic cord is longer.

A 60-year-old man has just been told that he has benign prostatic hypertrophy (BPH). He has a friend who just died from cancer of the prostate. He is concerned this will happen to him. How should the nurse respond? a. The swelling in your prostate is only temporary and will go away. b. We will treat you with chemotherapy so we can control the cancer. c. It would be very unusual for a man your age to have cancer of the prostate d. The enlargement of your prostate is caused by hormonal changes, and not cancer.

d. The enlargement of your prostate is caused by hormonal changes, and not cancer. The prostate gland commonly starts to enlarge during the middle adult years. BPH is present in 1 in 10 men at the age of 40 years and increases with age. It is believed that the hypertrophy is caused by hormonal imbalance that leads to the proliferation of benign adenomas. The other responses are not appropriate.

A 65-year-old patient remarks that she just cannot believe that her breasts sag so much. She states it must be from a lack of exercise. What explanation should the nurse offer her? After menopause: a. Only women with large breasts experience sagging. b. Sagging is usually due to decreased muscle mass within the breast. c. A diet that is high in protein will help maintain muscle mass, which keeps the breasts from sagging. d. The glandular and fat tissue atrophies, causing breast size and elasticity to diminish, resulting in breasts that sag.

d. The glandular and fat tissue atrophies, causing breast size and elasticity to diminish, resulting in breasts that sag. After menopause, the glandular tissue atrophies and is replaced with connective tissue. The fat envelope also atrophies, beginning in the middle years and becoming significant in the eighth and ninth decades of life. These changes decrease breast size and elasticity; consequently, the breasts droop and sag, looking flattened and flabby.

During the assessment of an 18-month-old infant, the mother expresses concern to the nurse about the infants inability to toilet train. What would be the nurses best response? a. Some children are just more difficult to train, so I wouldnt worry about it yet. b. Have you considered reading any of the books on toilet training? They can be very helpful. c. This could mean that there is a problem in your babys development. Well watch her closely for the next few months. d. The nerves that will allow your baby to have control over the passing of stools are not developed until at least 18 to 24 months of age.

d. The nerves that will allow your baby to have control over the passing of stools are not developed until at least 18 to 24 months of age. The infant passes stools by reflex. Voluntary control of the external anal sphincter cannot occur until the nerves supplying the area have become fully myelinated, usually around 1 to 2 years of age. Toilet training usually starts after the age of 2 years.

A 65-year-old woman is in the office for routine gynecologic care. She had a complete hysterectomy 3 months ago after cervical cancer was detected. Which statement does the nurse know to be true regarding this visit? a. Her cervical mucosa will be red and dry looking. b. She will not need to have a Pap smear performed c. The nurse can expect to find that her uterus will be somewhat enlarged and her ovaries small and hard. d. The nurse should plan to lubricate the instruments and the examining hand adequately to avoid a painful examination.

d. The nurse should plan to lubricate the instruments and the examining hand adequately to avoid a painful examination. In the aging adult woman, natural lubrication is decreased; therefore, to avoid a painful examination, the nurse should take care to lubricate the instruments and the examining hand adequately. Menopause, with the resulting decrease in estrogen production, shows numerous physical changes. The cervix shrinks and looks pale and glistening. With the bimanual examination, the uterus feels smaller and firmer and the ovaries are not normally palpable. Women should continue cervical cancer screening up to age 65 years if they have an intact cervix and are in good health. Women who have had a total hysterectomy do not need cervical cancer screening if they have 3 consecutive negative Pap tests or 2 or more consecutive negative HIV and Pap tests within the last 10 years.

Which of these statements about the peripheral nervous system is correct? a. The CNs enter the brain through the spinal cord. b. Efferent fibers carry sensory input to the central nervous system through the spinal cord. c. The peripheral nerves are inside the central nervous system and carry impulses through their motor fibers. d. The peripheral nerves carry input to the central nervous system by afferent fibers and away from the central nervous system by efferent fibers.

d. The peripheral nerves carry input to the central nervous system by afferent fibers and away from the central nervous system by efferent fibers. A nerve is a bundle of fibers outside of the central nervous system. The peripheral nerves carry input to the central nervous system by their sensory afferent fibers and deliver output from the central nervous system by their efferent fibers. The other responses are not related to the peripheral nervous system.

The mother of a 2-year-old is concerned about the upcoming placement of tympanostomy tubes in her son's ears. The nurse would include which of the following in the teaching plan? a. The tubes are placed in the inner ear. b. The tubes are used in children with sensorineural loss. c. The tubes are permanently inserted during a surgical procedure. d. The purpose of the tubes is to decrease the pressure and allow for drainage.

d. The purpose of the tubes is to decrease the pressure and allow for drainage. Polyethylene tubes are inserted surgically into the eardrum to relieve middle ear pressure and promote drainage of chronic or recurrent middle ear infections. Tubes extrude spontaneously in 6 months to 1 year.

During a health history, a 22-year old woman asks, Can I get that vaccine for human papilloma virus (HPV)? I have genital warts and Id like them to go away! What is the nurses best response? a. The HPV vaccine is for girls and women ages 9 to 26 years, so we can start that today. b. This vaccine is only for girls who have not yet started to become sexually active. c. Lets check with the physician to see if you are a candidate for this vaccine. d. The vaccine cannot protect you if you already have an HPV infection.

d. The vaccine cannot protect you if you already have an HPV infection. The HPV vaccine is appropriate for girls and women age 9 to 26 years and is administered to prevent cervical cancer by preventing HPV infections before girls become sexually active. However, it cannot protect the woman if an HPV infection is already present.

While performing an assessment of the perianal area of a patient, the nurse notices that the pigmentation of anus is darker than the surrounding skin, the anal opening is closed, and a skin sac that is shiny and blue is noted. The patient mentioned that he has had pain with bowel movements and has occasionally noted some spots of blood. What would this assessment and history most likely indicate? a. Anal fistula b. Pilonidal cyst c. Rectal prolapse d. Thrombosed hemorrhoid

d. Thrombosed hemorrhoid The anus normally looks moist and hairless, with coarse folded skin that is more pigmented than the perianal skin, and the anal opening is tightly closed. The shiny blue skin sac indicates a thrombosed hemorrhoid.

While performing an assessment of the perianal area of a patient, the nurse notices that the pigmentation of anus is darker than the surrounding skin, the anal opening is closed, and a skin sac that is shiny and blue is noted. The patient mentioned that he has had pain with bowel movements and has occasionally noted some spots of blood. What would this assessment and history most likely indicate? a. Anal fistula b. Pilonidal cyst c. Rectal prolapse d. Thrombosed hemorrhoid

d. Thrombosed hemorrhoid The anus normally looks moist and hairless, with coarse folded skin that is more pigmented than the perianal skin, and the anal opening is tightly closed. The shiny blue skin sac indicates a thrombosed hemorrhoid.

A man who has had gout for several years comes to the clinic with a problem with his toe. On examination, the nurse notices the presence of hard, painless nodules over the great toe; one has burst open with a chalky discharge. This finding is known as: a. Callus. b. Plantar wart. c. Bunion. d. Tophi.

d. Tophi. Tophi are collections of monosodium urate crystals resulting from chronic gout in and around the joint that cause extreme swelling and joint deformity. They appear as hard, painless nodules (tophi) over the metatarsophalangeal joint of the first toe and they sometimes burst with a chalky discharge.

A 15-year-old boy is seen in the clinic for complaints of dull pain and pulling in the scrotal area. On examination, the nurse palpates a soft, irregular mass posterior to and above the testis on the left. This mass collapses when the patient is supine and refills when he is upright. This description is consistent with: a. Epididymitis. b. Spermatocele. c. Testicular torsion. d. Varicocele

d. Varicocele A varicocele consists of dilated, tortuous varicose veins in the spermatic cord caused by incompetent valves within the vein. Symptoms include dull pain or a constant pulling or dragging feeling, or the individual may be asymptomatic. When palpating the mass, the examiner will feel a soft, irregular mass posterior to and above the testis that collapses when the individual is supine and refills when the individual is upright.

During an otoscopic examination, the nurse notes an area of black and white dots on the tympanic membrane and ear canal wall. What does this finding suggest? a. Malignancy b. Viral infection c. Blood in the middle ear d. Yeast or fungal infection

d. Yeast or fungal infection A colony of black or white dots on the drum or canal wall suggests a yeast or fungal infection (otomycosis).

The nurse is teaching a pregnant woman about breast milk. Which statement by the nurse is correct? a. Your breast milk is immediately present after the delivery of your baby. b. Breast milk is rich in protein and sugars (lactose) but has very little fat. c. The colostrum, which is present right after birth, does not contain the same nutrients as breast milk. d. You may notice a thick, yellow fluid expressed from your breasts as early as the fourth month of pregnancy.

d. You may notice a thick, yellow fluid expressed from your breasts as early as the fourth month of pregnancy. After the fourth month, colostrum may be expressed. This thick yellow fluid is the precursor of milk, and it contains the same amount of protein and lactose but practically no fat. The breasts produce colostrum for the first few days after delivery. It is rich with antibodies that protect the newborn against infection; therefore, breastfeeding is important.

A 70-year-old woman tells the nurse that every time she gets up in the morning or after shes been sitting, she gets really dizzy and feels like she is going to fall over. The nurses best response would be: a. Have you been extremely tired lately? b. You probably just need to drink more liquids. c. Ill refer you for a complete neurologic examination. d. You need to get up slowly when youve been lying down or sitting.

d. You need to get up slowly when youve been lying down or sitting. Aging is accompanied by a progressive decrease in cerebral blood flow. In some people, this decrease causes dizziness and a loss of balance with a position change. These individuals need to be taught to get up slowly. The other responses are incorrect.

A patient complains that while studying for an examination he began to notice a severe headache in the frontotemporal area of his head that is throbbing and is somewhat relieved when he lies down. He tells the nurse that his mother also had these headaches. What should the nurse suspect? a. Hypertension b. Cluster headaches c. Tension headaches d. migraine headaches

d. migraine headaches NOTE: See Table 14.1 for comparisons of headaches

A severe deficiency of thyroid hormone leading to nonpitting edema, coarse facial features, dry skin, and dry coarse hair is known as a. congenital hypothyroidism. b. scleroderma. c. Hashimoto thyroiditis. d. myxedema.

d. myxedema. Myxedema (hypothyroidism) is a deficiency of thyroid hormone. If severe, the symptoms include nonpitting edema or myxedema, a puffy edematous face, especially around the eyes (periorbital edema), coarse facial features, dry skin, and dry coarse hair and eyebrows. Congenital hypothyroidism is a thyroid deficiency that occurs at an early age, characteristics include low hairline, hirsute forehead, swollen eyelids, narrow palpebral fissures, widely spaced eyes, depressed nasal bridge, puffy face, thick tongue protruding through an open mouth, and a dull expression. Scleroderma is a rare connective tissue disease characterized by chronic hardening and shrinking degenerative changes in the skin blood vessels, synovium, and skeletal muscles. Hashimoto thyroiditis is a condition with excess thyroid hormone production, symptoms include goiter, nervousness, fatigue, weight loss, muscle cramps, heat intolerance, tachycardia, shortness of breath, excessive sweating, fine muscle tremor, thin silky hair and skin, infrequent blinking, and a staring appearance.

In performing an examination of a 3-year-old with a suspected ear infection, the nurse would: a. omit the otoscopic exam if the child has a fever. b. pull the ear up and back before inserting the speculum. c. ask the mother to leave the room while examining the child. d. perform the otoscopic examination at the end of the assessment.

d. perform the otoscopic examination at the end of the assessment. In addition to its place in the complete examination, eardrum assessment is manda- tory for any infant or child requiring care for illness or fever. For the infant or young child, the timing of the otoscopic examination is best toward the end of the complete examination.

Craniosynostosis is a severe deformity caused by a. increased intracranial pressure. b. excess growth hormone or a deficit in thyroid hormone. c. a localized bone disease that softens, thickens, and deforms bone. d. premature closure of the sutures.

d. premature closure of the sutures. Craniosynostosis is marked asymmetry that is due to a severe deformity caused by premature closure of the sutures and resulting in a long, narrow head. Hydrocephalus (obstruction of drainage of cerebrospinal fluid) results in excessive accumulation of cerebrospinal fluid, increasing intracranial pressure, and enlargement of the head. Paget disease (osteitis deformans) is a localized bone disease of unknown etiology that softens, thickens, and deforms bone. Acromegaly results from excessive secretion of growth hormone from the pituitary after puberty. Congenital hypothyroidism and myxedema are caused by thyroid hormone deficiency.

A patient is admitted to the emergency department after a motor vehicle accident. The trachea is deviated to the left side. This finding is characteristic of: a. right sided atelectasis. b. right pleural adhesion. c. aortic arch aneurysm. d. right pneumothorax.

d. right pneumothorax. The trachea is normally midline, with a right pneumothorax, the trachea is deviated to the unaffected side (left). The trachea is pulled downward with systole of an aortic arch aneurysm. With a large right sided pleural adhesion, the trachea is deviated to the affected side (right). With a large right-sided atelectasis, the trachea is deviated to the affected side (right).

During an interview, the patient states he has the sensation that "everything around him is spinning." The nurse recognizes that the portion of the ear responsible for this sensation is: a. the cochlea. b. cranial nerve VIII. c. the organ of Corti. d. the bony labyrinth.

d. the bony labyrinth If the labyrinth ever becomes inflamed, it feeds the wrong information to the brain, creating a staggering gait and a strong, spinning, whirling sensation called vertigo.

When assessing muscle strength, the nurse observes that a patient has complete range of motion against gravity with full resistance. What grade of muscle strength should the nurse record using a 0- to 5-point scale? a. 2 b.3 c. 4 d.5

d.5 Complete range of motion against gravity is normal muscle strength and is recorded as grade 5 muscle strength. The other options are not correct.

During the taking of a health history, the patient states, "It really hurts back there, and sometimes it itches, too. I have even seen blood on the tissue when I have a bowel movement. Is there something there?" The nurse should expect to see which of these upon examination of the anus? a. Rectal prolapse b. Internal hemorrhoid c.External hemorrhoid that has resolved d.External hemorrhoid that is thrombosed

d.External hemorrhoid that is thrombosed These symptoms are consistent with an external hemorrhoid. An external hemorrhoid, when thrombosed, contains clotted blood and becomes a painful, swollen, shiny blue mass that itches and bleeds with defecation. When the external hemorrhoid resolves, it leaves a flabby, painless skin sac around the anal orifice. An internal hemorrhoid is not palpable but may appear as a red mucosal mass when the person performs a Valsalva maneuver. A rectal prolapse appears as a moist, red doughnut with radiating lines.

hydrocele

scrotal swelling caused by a collection of fluid, more common in newborns

What kind of messages does the peripheral nervous system carry?

sensory and motor

hemorrhoids

swollen, twisted, varicose veins in the rectal region

cryptorchidism

undescended testicles


Related study sets

ATI Fundamental practice questions pt.3

View Set

Module 2, Unit 3 - Identification and Authentication

View Set

Introduction to Medical Terminology 'O' Prefixes, Word Roots, and Suffixes

View Set

Health and Wellness Final Review Questions

View Set

Social Studies (World History) 107 Self Test

View Set

Davis Ch. 56: Oral & Esophageal Disorders

View Set